Matrix and vector algebra - problems

You might also like

Download as pdf or txt
Download as pdf or txt
You are on page 1of 127

PART II: Matrix and vector algebra

Chapter 7: Matrix algebra, 1


Chapter 8: Determinants, 9
Chapter 9: Elementary operations with vectors, 14
Chapter 10: The scalar product, 22
Chapter 11: Vector product, 31
Chapter 12: Linear algebraic equations, 38
Chapter 13: Eigenvalues and eigenvectors, 49

Chapter 7: Matrix algebra

7.1. The elements are a13 = 3 and a31 = 2.


7.2. Comparison of A and B shows that A = B if x = −2 and y − x = 3. It follows that y = 1.
7.3. The answers are
· ¸ · ¸
3 1 0 −1 3 −6
A+B = , A−B = ,
3 1 6 −5 −1 2
· ¸
−4 7 −15
2A − 3B = .
−14 −3 2

7.4. The distributive law is satisfied since, as follows:


     
1 0 2 1 3 1
B+C = 2 1  +  −1 1  =  1 2  .
−1 −1 0 1 −1 0

Hence  
· ¸ 3 1 · ¸
1 3 0  1 2 = 6 7
A(B + C) = .
2 1 1 6 4
−1 0
Also
   
· ¸ 1 0 · ¸ 2 1
1 3 0   1 3 0 
AB + AC = 2 1 + −1 1 
2 1 1 2 1 1
−1 −1 0 1
· ¸ · ¸ · ¸
7 3 −1 4 6 7
= + = = A(B + C).
3 0 3 4 6 4

7.5. Verify the left- and right-hand sides:


  
· ¸ −1 0 · ¸
−1 2 −1  1 1 1
A(BC) = 2  
2 3 1 −1 2
3 −1
 
· ¸ −1 −1
−1 2 −1 
= −1 5 
2 3 1
4 1
· ¸
−5 10
=
−1 14

1
Also
  
· ¸ −1 0 · ¸
 −1 2 −1  1 1 1
(AB)C = 2 
2 3 1 −1 2
3 −1
· ¸· ¸
0 5 1 1
=
4 5 −1 2
· ¸
−5 10
=
−1 14
= A(BC),

as required.
7.6. Multiplication gives
· ¸· ¸ · ¸
4 2 −2 −1 0 0
AB = = ,
2 1 4 2 0 0

and · ¸· ¸ · ¸
−2 −1 4 2 −10 −5
BA = = 6= AB.
4 2 2 1 20 10

7.7. Since A + C = I3 ,
     
1 0 0 2 1 3 −1 −1 3
C = I3 − A =  0 1 0 − 1 −1 2  =  −1 2 −2  .
0 0 1 −2 1 1 2 −1 0

Hence
AC = A(I3 − A) = A − A2 and CA = (I3 − A)A = A − A2 = AC.
Thus     
2 1 3 −1 −1 3 3 −3 −8
AC =  1 −1 2   −1 2 −2  =  4 −5 −1  .
−2 1 1 2 −1 0 3 3 4
Since A + C = I3 , multiplication by A and C give

A2 + AC = A,

and
CA + C 2 = C.
Adding:
A2 + AC + CA + C 2 = A + C = I3 .
Finally, since AC = CA,
 
−5 6 16
2 2 
A + C = I3 − 2AC = −8 11 2 .
−6 −6 −7

7.8. Using the transpose rules in Section 7.3 (following Example 7.7),

(A + AT )T = AT + (AT )T = AT + A = A + AT ,

which means that A + AT is a symmetric matrix (see Section 7.3).


Similarly
(A − AT )T = AT − (AT )T = AT − A = −(A − AT ),

2
which means that A − AT is skew-symmetric.
Use the result that
1 1
A = (A + AT ) + (A − AT ).
2 2
For the given A  
2 −2 3
AT =  1 0 1 ,
3 1 2
so that    
2 − 12 3 0 3
2 0
A =  − 12 0 1  +  − 32 0 0 .
3 1 2 0 0 0

7.9. Given  
1 3 · ¸
1 −1 0
A =  −1 2  , then T
A = .
3 2 1
0 1
Also  
10 5 3 · ¸
 2 1
T
AA = 5 5 2 , T
A A= .
1 14
3 2 1

7.10. The equation Ax = d becomes


      
1 −1 2 x x − y + 2z 2
 3 0 1  y  =  3x + z  =  0 .
−1 2 −3 z −x + 2y − 3z −1

Hence
x − y + 2z = 2
3x + z = 0 .
−x + 2y − 3z = −1
Also
 
£ ¤ 1 3 −1
xT AT = x y z  −1 0 2 
2 1 −3
£ ¤ £ ¤
= x − y + 2z 3x + z −x + 2y − 3z = 2 0 −1 .

The verification follows.


7.11.     
1 0 0 1 0 0 1 0 0
A2 =  a −1 0   a −1 0  =  0 1 0 .
b c 1 b c 1 2b + ac 0 1
If 2b + ac = 0, then A2 = I3 . Hence A−1 = A: in other words A is its own inverse. In this case,
eliminating, say, b:  
1 0 0
A−1 = A =  a −1 0  ,
− 21 ac c 0
for any a and c.
It follows that
A2n−1 A2n−1 = A4n−2 = (A2 )2n−1 = (I3 )2n−1 = I3 .
Hence the inverse of A2n−1 is A2n−1 .

3
7.12. The two matrix products are
  1 1
  
2 0 1 0 2 4 1 0 0
AB =  2 −2 2  1 −1 − 14 = 0 1 0  = I3 .
0 4 −4 1 −1 − 12 0 0 1

Similarly     
1 1
0 2 4 2 0 1 1 0 0
BA =  1 −1 − 14  2 −2 2 = 0 1 0  = I3 .
1 −1 − 12 0 4 −4 0 0 1
By Section 7.4, B must be the inverse of A.
7.13. The powers of A are
    
2 0 1 2 0 1 4 4 3
A2 =  2 −2 2   2 −2 2  =  0 12 0  ,
0 4 1 0 4 1 8 −4 9

and     
2 0 1 4 4 3 16 4 15
A3 = AA2 =  2 −2 2   0 12 0  =  24 −24 24  .
0 4 1 8 −4 9 8 44 9
Hence
     
16 4 15 4 4 3 2 0 1
A3 − A2 − 12A =  24 −24 24  −  0 12 0  − 12  2 −2 2 
8 44 9 8 −4 9 0 4 1
 
1 0 0
= −12  0 1 0  .
0 0 1

Multiplying both sides by A−1 (A is nonsingular):

A−1 A3 − A−1 A2 − 12A−1 A = −12A−1 I3 ,

that is,
A2 − A − 12I3 = −12A−1 .
Hence  
5 −2 −1
1 2 1
A−1 = [A − A − 12I3 ] =  1 −1 1 ,
12 6
−4 4 2
using A2 previously found.
7.14. Let A be the matrix in each case, and use rule (7.8).
(a) Then ¯ ¸
¯ 1 1
det(A) = ¯¯ = −3.
2 −1
Hence · ¸
1 1
−1 3 3
A = 2 .
3 − 13
(b) ¯ ¯ · ¸
¯ 2 3 ¯ 11 3
− 43
det(A) = ¯¯ ¯ = 43, and A−1 =
¯
43
7 2 .
−7 11 43 43

4
(c) ¯ ¸ · ¸
¯ 1 0 1 0
det(A) = ¯¯ = −2, and A−1 = .
0 −2 0 − 21
(d) ¯ ¯ · ¸
¯ 10 −7 ¯¯ 0 1
det(A) = ¯¯ ¯ = 56, and A −1
= 1 5
8
.
8 0 −7 28

(e) ¯ ¯ · ¸
¯ −99 100 ¯ 49 50
¯
det(A) = ¯ ¯ = −19402, and A−1 = − 9701 9701
.
97 98 ¯ 97
19402
99
19402

7.15. The inverse of A must satisfy AA−1 = I4 . Note that A has just one element in any row or
column. Hence to achieve the zeros in the correct positions in I4 , we must have
 
0 0 1 0
 1 0 0 0 
A−1 =   0 1 0 0 .

0 0 0 1

This suggests the form of inverse appropriate to the more general matrix:
 
0 0 c−1 0
 a−1 0 0 0 
A−1 = 
,
0 b−1 0 0 
0 0 0 d−1

assuming that a, b, c, d are all non-zero.


7.16. The equation Ax = d becomes
    
0 1 1 x 6
 1 −2 2   y  =  3  ,
1 0 1 z −9
or
y + z = 6
x − 2y + 2z = 3 .
x + z = −9
To find the inverse first calculate
¯ ¯
¯ 0 1 1 ¯¯
¯
det(A) = ¯¯ 1 −2 2 ¯¯ = 3.
¯ 1 0 1 ¯

Using (7.10)  
− 23 − 13 4
3
 
A−1 =  1
3 − 13 1
3 .
2 1
3 3 − 13
Multiplying Ax = d on the right by A−1 :

A−1 Ax = I3 Ax = x = A−1 d.

Hence    2    
x −3 − 13 4
3 6 −17
 
x =  y  =  31 − 13 1
3  3  =  −2  ,
z 2 1
− 13 −9 8
3 3

5
which is the solution of the set of equations.
7.17. Thus,

(A−1 BA)2 = (A−1 BA)(A−1 BA) = A−1 BAA−1 BA = A−1 BIn BA = A−1 B 2 A.

Using this result, observe that

A−1 B 4 A = (A−1 B 2 A)2 = (A−1 BA)4 .

Using rule (7.8)


· 1 ¸· ¸· ¸ · ¸
−1 3 − 23 1 2 1 2 1
3
8
3
A BA = 1 1 = .
3 3
−1 0 −1 1 − 23 2
3

Finally
· 1 8 ¸4 · ¸
−1 4 −1 4 3 3 1 −8
A B A = (A BA) = = .
− 23 2
3
2 0

7.18. Since the parabola must pass through the points (x1 , y1 ), (x2 , y2 ) and (x3 , y3 ),

y1 = a + bx1 + cx21 ,

y2 = a + bx2 + cx22 ,
y3 = a + bx3 + cx23 .
These can be viewed as three linear equations in a, b and c, which can be expressed in the matrix
form     
1 x1 x21 a y1
 1 x2 x22   b  =  y2  .
1 x3 x23 c y3
Let A be the 3 × 3 matrix on the left. By (7.11)
¯ ¯
¯ 1 x1 x21 ¯¯
¯
det(A) = ¯¯ 1 x2 x22 ¯¯ = (x2 x23 − x22 x3 ) − x1 (x23 − x22 ) + x21 (x3 − x2 )
¯ 1 x3 x23 ¯
= (x1 − x2 )(x2 − x3 )(x3 − x1 ).

The determinant will be non-zero if x1 , x2 and x3 are all different. By (7.10) the inverse of A
is given by ¯ 
¯ x2 x23 − x3 x22 −(x1 x23 − x3 x21 ) x1 x22 − x2 x21
1 ¯
A−1 = ¯ −(x23 − x22 ) x23 − x21 −(x22 − x23 )  ,
det(A) ¯¯
x3 − x2 −(x3 − x1 ) x2 − x1
which equals the answer given in the question after some factorization. It follows that
   x2 x3 y1 x 3 x 1 y2 x 1 x 2 y3 
y1 (x2 −x3 )(x3 −x1 ) (x3 −x2 )(x1 −x2 ) (x1 −x3 )(x2 −x3 )
 y2  =  (x +x )y1
 − (x2 −x2 3 )(x3 3 −x − (x3(x 3 +x1 )y2
− (x1(x 1 +x2 )y3 
1) −x2 )(x1 −x2 ) −x3 )(x2 −x3 )  .
y3 y1 y2 y3
(x2 −x3 )(x3 −x1 ) (x3 −x2 )(x1 −x2 ) (x1 −x3 )(x2 −x3 )

Note that if, for example, x1 = x2 then there will no parabola of the form given if y1 6= y2 , but if
y1 = y2 (that is, two points coincide) there will be an infinite set of such parabolas.
For the three points given x1 = −2, x2 = 1 and x3 = 3,
   
1
1 −2 4 5 1 − 15
 1 
A= 1 1 1  and A−1 =  − 15 4 1
6 10  .
1 3 9 1
− 16 1
15 10

6
Hence    14 
a −5
 b =
 151 
.
c 11
15

The required parabola through the given points is

y = − 14
5 +
1
15 x + 11 2
15 x .

7.19. If aij = (−j)i − ij, then  


−2 −4 −6
A =  −1 0 3 .
−4 −14 −36
Also ¯ ¯  
7
¯ −2 −4 −6 ¯ − 25 − 12
¯ ¯ 
4

det A = ¯¯ −1 0 3 ¯ = 24,
¯ A−1 =  −2 2 1
2 .
¯ −4 −14 −36 ¯ 7
−2 1
− 16
12

7.20. Calculate the powers of A:


 
8 7 11
A2 = AA =  3 6 3 ,
5 1 8
    
2 1 3 8 7 11 34 23 49
A3 = AA2 =  1 −1 2   3 6 3  =  15 3 24  .
1 2 1 5 1 8 19 20 25
Hence
     
34 23 49 8 7 11 2 1 3
A3 − 2A2 − 9A =  15 3 24  − 2  3 6 3  − 9 1 −1 2 
19 20 25 5 1 8 1 2 1
= 0.

Also
     
8 7 11 2 1 3 1 0 0
A2 − 2A − 9I3 =  3 6 3  − 2  1 −1 2  −  0 1 0 
5 1 8 1 2 1 0 0 1
 
−5 5 5
=  1 −1 −1  6= 0.
3 −3 −3

Suppose that A−1 exists. Then

A−1 (A3 − 2A2 − 9A) = A2 − 2A − 9I3 = 0,

which contradicts the result above. We conclude that the inverse of A does not exist.
7.21. Given A2 = A and A 6= In .
(a) Suppose that A−1 exists. Then

A−1 A2 = A−1 A or A = In ,

which contradicts the assumption that A is not the identity matrix. We conclude that the inverse
of A does not exist.

7
(b) Verify the result:

(In + A)(In − 12 A) = I2n + A − 12 A − 12 A2 = In + A − 21 A − 12 A = In .

Hence In − 12 A must be the inverse of In + A.


(c) Use proof by induction. For m = 2,

(In + A)2 = (In + A)(In + A) = In + 2A + A2 = In + 3A,

since A2 = A. Hence the result is certainly true for m = 2. Suppose we know that the result is
true for some value of m, say m = r; that is that

(In + A)r = In + (2r − 1)A.

Then

(In + A)r+1 = (In + A)(In + A)r = (In + A)(In + (2r − 1)A


= I2n + A + (2r − 1)A + (2r − 1)A2
= In + A + (2r − 1)A + (2r − 1)A
= In + (2r+1 − 1)A.

Hence if the conjecture is true for m = r, then it is true for m = r + 1. Hence by induction it is
true for m = 3, 4, . . ..
7.22.
· ¸ · ¸
x1 + x2 y1 + y2 x1 x2 − y1 y2 x2 y1 + x1 y2
A1 + A2 = , A1 A2 = .
−y1 − y2 x1 + x2 −x2 y1 − x1 y2 x1 x2 − y1 y2

Note that A2 A1 = A1 A2 . The inverse


· ¸
1 x1 −y1
A−1
1 = 2 .
x1 + y12 y1 x1

These results parallel the rules for complex numbers. For the complex numbers:

z1 + z2 = (x1 + x2 ) + i(y1 + y2 ), z1 z2 = x1 x2 − y1 y2 + i(x1 y2 + x2 y1 ) = z2 z1 ,

1 x1 + iy1
= 2 .
z1 x1 + y12
The top row in the matrix operations gives the real and imaginary parts of the corresponding
complex ones.
Also |z1 |2 = x21 + y12 , corresponding to det A1 = x21 + y12 .
For the exponential function
1 2
ez1 = 1 + z1 + z + ···.
2! 1
Interpret the exponential of the matrix as
1 2
eA1 = In + A1 + A + ···.
2! 1
The real and imaginary parts of ez1 are given by the elements on the top rows of the terms of eA1 ,
since z n corresponds to An .

8
Chapter 8: Determinants

8.1. (a) ¯ ¯
¯ 1 2 ¯¯
¯ = (1) × (3) − (2) × (−1) = 5.
¯ −1 3 ¯
(b) ¯ ¯
¯ 1 0 1 ¯ ¯ ¯ ¯ ¯ ¯ ¯
¯ ¯ ¯ 0 ¯¯ ¯ 0 0 ¯ ¯ ¯
¯ 0 1 0 ¯ = (1) × ¯ 1 − (0) × ¯¯ ¯ + (1) × ¯ 0 1 ¯ = 1.
¯ ¯ ¯ 0 1 ¯ 1 1 ¯ ¯ 1 0 ¯
¯ 1 0 1 ¯

(c) ¯ ¯
¯ 1 −1 2 ¯
¯ ¯
¯ 3 1 −1 ¯ = 1.
¯ ¯
¯ 2 1 −1 ¯

(d) ¯ ¯
¯ 2 1 0 −1 ¯
¯ ¯
¯ 0 0 2 0 ¯
¯ ¯ = 20.
¯ 3 1 2 1 ¯
¯ ¯
¯ 0 1 −1 1 ¯

(e) Expanding by the top row at each step


¯ ¯
¯ 0 1 0 0 0 ¯¯ ¯ ¯
¯ ¯ 1 0 0 0 ¯ ¯ ¯
¯ 1 0 0 0 0 ¯¯ ¯ ¯ ¯ 0 0 1 ¯ ¯ ¯
¯ ¯ 0 0 0 1 ¯ ¯ ¯ ¯ 0 ¯¯
¯ 0 0 0 0 1 ¯¯ = − ¯¯ ¯ = −¯ 1 0 0 ¯ = −¯ 1 = −1.
¯ ¯ ¯ ¯ ¯ 0 1 ¯
¯ 0 0 1 0 0 ¯¯ ¯ 0 1 0 0 ¯ ¯ 0 1 0 ¯
¯ ¯ 0 0 1 0 ¯
¯ 0 0 0 1 0 ¯

(f) After repeated expansion by the top rows


¯ ¯
¯ 2 1 0 0 0 ¯ ¯ ¯ ¯ ¯
¯ ¯ ¯ 2 1 0 0 ¯ ¯ 1 1 0 0 ¯
¯ 1 2 1 0 0 ¯ ¯ ¯ ¯ ¯
¯ ¯ ¯ ¯ ¯ ¯
¯ 0 1 2 1 0 ¯ = 2¯ 1 2 1 0 ¯−¯ 0 2 1 0 ¯ = 2 × 5 − 4 = 6.
¯ ¯ ¯ 0 1 2 1 ¯ ¯ 0 1 2 1 ¯
¯ 0 0 1 2 1 ¯ ¯ ¯ ¯ ¯
¯ ¯ ¯ 0 0 1 2 ¯ ¯ 0 0 1 2 ¯
¯ 0 0 0 1 2 ¯

8.2. (a) The elements of row 1 are twice the elements in row 2 (Rule 5).
(b) Add the elements in row 2 to row 1 to give (Rule 6)
¯ ¯ ¯ ¯
¯ −1 2 3 ¯¯ ¯¯ 2 3 1 ¯¯
¯
¯ 3 1 −2 ¯¯ = ¯¯ 3 1 2 ¯¯ .
¯
¯ −2 −3 −1 ¯ ¯ −2 − − 3 −1 ¯

The determinant is zero since the elements in row 1 are (-1) times the elements in row 3.
(c) The determinant is unchanged if the elements in row 1 become the elements in row 1 minus
the elements in row 2 (Rule 6). The determinant is therefore zero since the first and third rows
have the same elements (Rule 5).
(d) The determinant is zero since the elements in rows 2 and 3 are in the same ratio (3/5) (Rule
6).
8.3. Since a is a factor of each element in row 1 and also a factor of each element in column 1,
and c is factor of each element in column 3, by Rule 2,
¯ 3 ¯ ¯ ¯
¯ a ab ac2 ¯ ¯ a b c ¯
¯ ¯ ¯ ¯
¯ a c ac ¯¯ = a2 c ¯¯ b c a ¯¯ = a2 c∆.
¯
¯ bac a bc ¯ ¯ c a b ¯

9
8.4. There are many ways of simplifying determinants using the rules in Section 8.2. The usual
aim is to introduce zero elements and to reduce the size of numbers. These methods are illustrated
in the following solutions. Note that operations between columns (ci ) are employed as well as
operations between rows (ri ).
(a)
¯ ¯ ¯ ¯
¯ 99 100 200 ¯ ¯ 1 −2 1 ¯¯
¯ ¯ ¯
¯ 98 102 199 ¯ = ¯ 98 102 199 ¯ (r10 = r1 − r2 )
¯ ¯ ¯ ¯
¯ −1 2 3 ¯ ¯ −1 2 3 ¯
¯ ¯
¯ 1 −2 0 ¯¯
¯
= ¯¯ 98 102 101 ¯¯ (c03 = c3 − c1 )
¯ −1 2 3 ¯
¯ ¯
¯ 1 0 0 ¯¯
¯
= ¯¯ 98 298 101 ¯¯ (c02 = c2 − 2c1 )
¯ −1 0 4 ¯
= 298 × 4 = 1192.

(b)
¯ ¯ ¯ ¯
¯ 77 84 55 ¯¯ ¯ 2 −3 −2 ¯
¯ ¯ ¯
¯ 75 87 57 ¯¯ = ¯ 75 87 57 ¯ (r10 = r1 − r2 )
¯ ¯ ¯
¯ 1 −2 3 ¯ ¯ 1 −2 3 ¯
¯ ¯
¯ 25 29 19 ¯ 0
= 3 ¯¯ ¯ (r = r1 − 2r2 )
1 −2 3 ¯ 1
¯ ¯
¯ 0 1 −8 ¯¯
¯
= 3 ¯¯ 25 3 −6 ¯¯ (c02 = c2 − c1 , c03 = c3 − c1 )
¯ 1 −3 2 ¯
¯ ¯ ¯ ¯
¯ 25 −6 ¯ ¯ 4 ¯¯
= −3 ¯¯ ¯ − 24 ¯ 25
1 2 ¯ ¯ 1 −3 ¯
= −3 × 56 − 24 × (−79) = 1728

(c)
¯ ¯ ¯ ¯
¯ 2 −1 1 ¯ ¯ 3 ¯ −1 1
¯ ¯ ¯ ¯ 0
¯ 99 98 55 ¯ = ¯ 1 ¯ (c1 = c1 − c2 )
98 55
¯ ¯ ¯ ¯
¯ 200 197 111 ¯ ¯ 3 ¯ 197 111
¯ ¯
¯0 −198 −110 ¯¯
¯
= ¯1 98 55 ¯¯ (r10 = r1 − r3 )
¯
¯3 197 111 ¯
¯ ¯
¯0 −198 −110 ¯¯
¯
= ¯1 −1 0 ¯¯ (r20 = r2 − 21 r1 , r30 = r3 + r1 )
¯
¯3 −1 1 ¯
¯ ¯
¯0 −198 −110 ¯¯
¯
= ¯1 −1 0 ¯¯
¯
¯2 0 1 ¯
= −22

(d)
¯ ¯ ¯ ¯
¯ 87 84 83 81 ¯ ¯ 4 1 2 81 ¯
¯ ¯ ¯ ¯ (c01 = c1 − c3 )
¯ 77 76 77 75 ¯ ¯ 0 −1 2 75 ¯
¯ ¯ = ¯ ¯ (c02 = c2 − c3 )
¯ 54 53 52 54 ¯ ¯ 2 1 −2 54 ¯
¯ ¯ ¯ ¯ (c03 = c3 − c4 )
¯ −43 −44 −46 −4 ¯ ¯ 3 2 −42 −4 ¯

10
¯ ¯
¯ 4 0 4 156 ¯¯
¯ (r10 = r1 + r2 )
¯ 0 −1 2 75 ¯¯
= ¯ (r30 = r3 + r2 )
¯ 2 0 0 129 ¯¯
¯ (r40 = r4 + 2r2 )
¯ 3 0 −38 146 ¯
¯ ¯
¯ 4 4 156 ¯¯
¯
= − ¯¯ 2 0 129 ¯¯ (expanding by column 2)
¯ 3 −38 146 ¯
¯ ¯
¯ 0 4 156 ¯¯
¯
= − ¯¯ 2 0 129 ¯¯ (c10 = c1 − c2 )
¯ 41 −38 146 ¯
¯ ¯ ¯ ¯
¯ 2 129 ¯ ¯ 0 ¯¯
= 4 ¯¯ ¯ − 156 ¯ 2
41 146 ¯ ¯ 41 −38 ¯ = −8132

8.5. If b = a, then the first two columns have the same elements which means that the determinant
will be zero (Rule 5). We conclude that the determinant has a factor (a − b). Similarly the
determinant has factors (b − c) and (c − a). The determinant is of degree three in a, b and c which
implies that
∆ = k(b − c)(c − a)(a − b),
where k is a number. Compare the leading diagonal term 1×b×c2 = bc2 in ∆ with the corresponding
term in the expansion which is kbc2 . Hence k = 1, and
∆ = (b − c)(c − a)(a − b).
8.6. Operations between columns give
¯ ¯
¯ 1 1 1 ¯
¯ ¯
∆ = ¯¯ a b c ¯¯
¯ a3 b3 c3 ¯
¯ ¯
¯ 1 0 0 ¯
¯ ¯ (c02 = c2 − c1 )
= ¯¯ a b−a c−a ¯
¯
¯ a3 b3 − a3 c3 − a3 ¯ (c03 = c3 − c1 )

= (b − a)(c3 − a3 ) − (c − a)(b3 − a3 ).
Taking out factors
¯ ¯
¯ 1 1 ¯
∆ = (b − a)(c − a) ¯¯ 2 ¯
b + a2 + ba c + a + ac ¯
2 2
¯ ¯
¯ 1 0 ¯ 0
= (b − a)(c − a) ¯¯ 2 ¯ (c = c2 − c1 )
b + a2 + ba (c − b ) + a(c − b) ¯ 2
2 2

= (b − a)(c − a)(c − b)(a + b + c) = (b − c)(c − a)(a − b)(a + b + c)

8.7. The determinant equation is linear equation in x and y, and therefore represents the equation
of a straight line. Also the determinant is zero if x = a1 , y = b1 since two rows have the same
elements. Hence the line passes through the point (a1 , b1 ). Similarly the line also passes through
the point (a2 , b2 ).
The cofactors of x and y are X1 = b1 −b2 and X2 = a2 −a1 , and the slope is −X1 /X2 (a1 6= a2 ).
(a) The required line is y = 4x − 5.
(b) The required line is 5y = −x − 1.
8.8.
¯ ¯ ¯ ¯
¯ 1 1 −1 ¯¯ ¯ 1 0 0 ¯¯
¯ ¯ (c03 = c3 − c1 )
¯ 1 a 2 ¯¯ = ¯¯ 1 a − 1 3 ¯¯
¯ (c02 = c2 − c1 )
¯ −1 1 2 ¯ ¯ −1 2 1 ¯
= (a − 1) − 6 = a − 7.

11
The determinant is zero if a = 7.
8.9. Each term in the expansion contains three elements each from a different row and column.
Since there are x’s are in different rows and columns, terms in x3 will appear in the expansion,
although it is possible that terms could cancel. In the second determinant there is no x in row 1,
the expansion will be of degree 2, at most, in x.
¯ ¯ ¯ ¯
¯ x 2 −2 ¯ ¯ 2x + 2 x + 1 x + 1 ¯
¯ ¯ ¯ ¯ 0
¯ 2 x 3 ¯ = ¯ 2 x 3 ¯ (r1 = r1 + r2 + r3 )
¯ ¯ ¯ ¯
¯ x −1 x ¯ ¯ x −1 x ¯
¯ ¯
¯ 2 1 1 ¯¯
¯
= (x + 1) ¯¯ 2 x 3 ¯¯
¯ x −1 x ¯
= x3 + x2 + 5x + 4
Hence the determinant is zero if x = −1 or x2 + x + 4 = 0. The solutions are

x = −1, x = 12 (−1 ± i 15).
The expansion of the second determinant is
¯ ¯
¯ 1 1 2 ¯
¯ ¯
¯ 3 x 2 ¯ = 4 − x − x2 .
¯ ¯
¯ x 1 x ¯

Hence the determinant is zero where x = 21 (−1 ± 17).
8.10. By the expansion (8.3)
¯ ¯
¯ a11 + b11 a12 + b12 a13 + b13 ¯
¯ ¯
¯ a21 a22 a23 ¯
¯ ¯
¯ a31 a32 a33 ¯
= (a11 + b11 )(a22 a33 − a32 a23 ) − (a12 + b12 )(a21 a33 − a31 a23 ) +
(a13 + b13 )(a21 a32 − a31 a22 )
= a11 (a22 a33 − a32 a23 ) − a12 (a21 a33 − a31 a23 ) + a13 (a21 a32 − a31 a22 ) +
b (a a − a32 a23 ) − b12 (a21 a33 − a31 a23 ) + b13 (a21 a32 − a31 a22 )
¯ 11 22 33 ¯ ¯ ¯
¯ a11 a12 a13 ¯ ¯ b11 b12 b13 ¯
¯ ¯ ¯ ¯
= ¯¯ a21 a22 a23 ¯¯ + ¯¯ a21 a22 a23 ¯¯ .
¯ a31 a32 a33 ¯ ¯ a31 a32 a33 ¯

8.11. Expansion by row 1, as in the previous problem, will lead to 2 determinants. Then expansion
by row 2 for each of these 2 determinants will result in 22 = 4 determinants: the number doubles
for each row. Therefore there will be 23 = 8 determinants.
For an n × n determinant with the sum of two terms in each element, the expansion will lead
to 2n determinants.
8.12.
¯ ¯ ¯ ¯
¯ 1 a1 − b1 a1 + b1 ¯ ¯ 1 2a1 a1 + b1 ¯¯
¯ ¯ ¯
¯ 1 a2 − b2 a2 + b2 ¯ = ¯¯ 1 2a2 a2 + b2 ¯¯ (c02 = c2 − c3 )
¯ ¯
¯ 1 a3 − b3 a3 + b3 ¯ ¯ 1 2a3 a3 + b3 ¯
¯ ¯
¯ 1 2a1 b1 ¯¯
¯
= ¯¯ 1 2a2 b2 ¯¯ (c03 = c3 − 12 c2 )
¯ 1 2a3 b3 ¯
¯ ¯
¯ 1 a1 b1 ¯¯
¯
= 2 ¯¯ 1 a2 b2 ¯¯
¯ 1 a3 b3 ¯

12
8.13. Easy to verify that ¯ ¯
¯ 2 1 ¯¯
D1 = 2, D2 = ¯¯ = 3.
1 2 ¯
Expanding Dn by the top row

Dn = 2Dn−1 − 1 × 1 × Dn−2 = 2Dn−1 − Dn−2 ,

or, equivalently,
Dn − Dn−1 = Dn−1 − Dn−2 .
It follows that

Qn = Dn − Dn−1 = Qn−1 = Qn−2 = . . . = Q2 = D2 − D1 = 2 − 1 = 1.

Also
Dn = Dn−1 + Qn = Dn−1 + 1 = Dn−2 + 2 = · · · = D2 + (n − 1) = n + 1.

8.14. First observe that the determinant is a quartic in x, that is, a polynomial of the fourth
degree. If x = a, then rows 1 and 2 have the same elements, which mean that the determinant
is zero. Similarly, if x = b and x = c, two rows have identical elements. There will be one more
solution. Then
¯ ¯
¯ x a b c ¯
¯ ¯
¯ a x b c ¯
¯ ¯
¯ a b x c ¯
¯ ¯
¯ a b c x ¯
¯ ¯
¯ x+a+b+c x+a+b+c x+a+b+c x+a+b+c ¯
¯ ¯
¯ a x b c ¯
= ¯¯ ¯
¯
¯ a b x c ¯
¯ a b c x ¯
(r10 = r1 + r2 + r3 + r4 )
¯ ¯
¯ 1 1 1 1 ¯
¯ ¯
¯ a x b c ¯
= (x + a + b + c) ¯¯ ¯,
¯
¯ a b x c ¯
¯ a b c x ¯

showing a factor (x + a + b + c). Hence, the solutions are

x = a, b, c, −(a + b + c).

8.15. The determinants of A and B are


¯ ¯ ¯ ¯
¯ a a12 ¯¯ ¯ b b12 ¯¯
det A = ¯¯ 11 = a11 a22 − a12 a21 , det B = ¯¯ 11 = b11 b22 − b12 b21 .
a21 a22 ¯ b21 b22 ¯

(a) The product AB is given by


· ¸
a11 b11 + a12 b21 a11 b12 + a12 b22
AB = .
a21 b11 + a22 b21 a21 b12 + a22 b22

Then it can be verified that

det(AB) = (a11 b11 + a12 b21 )(a21 b12 + a22 b22 ) −


(a21 b11 + a22 b21 )(a11 b12 + a12 b22 )
= (a11 a22 − a12 a21 )(b11 b22 − b12 b21 ) = det A det B.

13
Put B = A to obtain det(A2 ) = (det A)2 .
(b) ¯ ¯
¯ a a21 ¯
det(A ) = ¯¯ 11
T ¯ = a11 a21 − a12 a21 = det A.
¯
a12 a22

(c) By (7.8) ¯ ¯
−1 1 ¯¯ a22 −a12 ¯¯
A = .
det A ¯ −a21 a11 ¯
Since det A effectively divides each element in the matrix,
1 1 1
det(A−1 ) = (a22 a11 − a21 a12 ) = det A = .
(det A)2 (det A)2 det A

(d) Result follows from (c) since A−1 = adj A/ det A.


8.16. The numerical answers are:
det A = −2; det B = 18; det AB = −36; det(AT ) = −2;
det(adj A) = −2. Since  
7 1 −5
1
A−1 =  −2 0 2 ,
2
1 1 −1
then det(A−1 ) = − 12
8.17. The matrix A in full is
 
α − 2 2α − 4 3α − 8
A =  α + 2 2α + 4 3α + 8  = 0,
α − 2 2α − 4 3α − 8

and its determinant is zero since rows 1 and 3 have the same elements.

Chapter 9: Elementary operations with vectors

9.1. (a) P Q = −QP = (5, −3); (b) P Q = −QP = (−1, −3); (c) P Q = −QP = (−1, −3);
(d) P Q = −QP = (1, 1).
y
4 HbL
HaL
3

1 HcL

x
-2 -1 1 2 3

-1
HdL

-2

Figure 1: Problem 9.1


√ √ √ √ √
9.2. (a) 3, 0; (b) 2, 90◦ ; (c) √ 2, 135◦ ; (d) 2, 45◦ ; (e) 2, −135◦ ;
(f) 5, 127◦ ; (g) 5, −126◦ ; (h) 5, 153◦ .
√ √ √ √ √
9.3. (a) ( 2, 2); (b) (− 32 , 3 2 3 ); (c) ( 32 , 3 2 3 ); (d) (− 3 2 2 , − 32 ).
9.4. BE = QE − QB = (3, 3) + (1, 1) + (2, 3) − [(2, 4) + (4, 1)] = (6, 1) − (6, 5) = (0, −4). The
bearing is due south.
√ 2 2 2 √
9.5. (a) Distance between (0, 0, 0) and (1, 2, 3)
√ is (1 2 + 2 + 3 )2 = 14. 2 √
(b) Distance between (1, 2, 3) and (3, 2, 1) is [(1 − 3) + (2 − 2) + (3 − 1) ] = 2 2.

14
(c) Distance between (1, 0, −1) and (−1, 1, 0) is
√ √
[(1 − (−1))2 + (0 − 1)2 + (−1 − 0)2 ] = 6.

9.6. The vector P Q = (2, 3, 3) − (1, 2, 1) = (1, 1, 2). The projections respectively are the compo-
nents of P Q, namely 1, 1, 2.
9.7. (a) 2a = 2(1, 2, 1) = (2, 4, 2); 3b = 3(2, 1, 2) = (6, 3, 6); 2a − 3b = (2, 4, 2) − (6, 3, 6) =
(−4, 1, −4).
(b) 2a = (6, 4, 6); 3b = (3, 3, 6); 2a − 3b = (3, 1, 0).
(c) 2a = (12, 6, 2); 3b = (12, 6, 3); 2a − 3b = (0, 0, −1).
The vector 2a − 3b is parallel to the (x, y) plane in (b) because the z component is zero, and
parallel to the z axis in (c) since both its x and y components are zero.
9.8. Figure 2 shows that AB + BC + CA = 0 by the triangle law.
B

Figure 2: Problem 9.8

9.9. Figure 3 shows that CD = CB + BA + AD.

B B D

C C
A A

D E

Figure 3: Problem 9.9

9.10. The vector OP = (5, 2, −3), and the vector giving the origin Q in terms of O is OQ =
(2, −1, 3). Hence QP = OP − OQ = (3, 3, −6). In QXY Z, the point P has coordinates (3, 3, −6).
The relation between coordinates is x = X + 2, y = Y − 1, z = Z + 3. Hence the equation of
the sphere x2 + y 2 + z 2 = 1 becomes the sphere

(X + 2)2 + (Y − 1)2 + (Z + 3)2 = 1.

9.11. Let AB = a, BC = b, CD = c. By repeated application of the triangle rule, AD = a+b+c.


Since P and Q are midpoints
1 1 1
P Q = P B + BQ = AB + BC = (a + b).
2 2 2
Simlarly
1 1 1 1 1
SR = SD + DR = AD + DC = (a + b + c) − c = (a + b) = P Q.
2 2 2 2 2

15
The quadrilateral P QRS has opposite sides which are parallel and equal in length, and is therefore
a parallelogram.
9.12. AP is the median vector from A; F is a representative point, position vector r, on AP . The
parametric equation for AP is

r = rA + λAP = rA + λ(b + 21 a) (i)


1
= rA + λ[rC − rA + 2 (rB − rC )]
= (1 − λ)rA + 12 λrB + 12 λrC , (ii)

where a = CB, b = AC, c = AC and λ is a parameter. By permuting the suffixes, A → B,


B → C, C → A, we obtain for the other medians:

r = (1 − µ)rB + 12 µrC + 12 µrA , (iii)

and
r = (1 − ν)rC + 12 νrA + 12 νrB , (iv)
where µ, ν are parameters. These lines meet at a single point C if values of λ, µ, ν can be found
that make the right-hand sides (ii), (iii), (iv) equal. If we put λ = µ = ν = 32 we obtain the
common point G, with
OG = 13 (rA + rB + rC ).
Also the term λAP = 23 AP in (i) shows that G is two-thirds of the way along AP .
9.13. Can one vector, say OC, be expressed in terms of the other two? We require constants α
and β such that, from eqn (9.12)

OC = αOA + βOB, or (5, 5, 7) = α(1, 1, 2) + β(1, 1, 1).

This will be possible if α + β = 5 and 2α + β = 7. Hence α = 2 and β = 3, and the three vectors
drawn from the origin lie in the same plane.
Similarly, for OA = (a, a, p), OB = (b, b, q), OC = (c, c, r), we require α and β such that

c = αa + βb, r = αp + βq.

Hence, the lines lie in the same plane since solutions for α and β can be found:
cq − rb ra − cp
α= , β= ,
aq − pb aq − bp

provided aq − pb 6= 0. If aq − pb = 0, the vectors OA and OB lie in the same direction, and the
three vectors will still lie in a plane. The vectors all lie in a plane through the z axis, at 45◦ to the
x and y axes.
9.14. If vE is the velocity of the glider relative to the earth, then

vE = v − w = (40, 30, 10) − (5, −10, 0) = (35, 40, 10).

9.15. Let vBS be the velocity of the boat relative to the sea, vB be the velocity of the boat and
vS the velocity of the sea. Then vBS = vB − vS , so that

vS = vB − vBS = (4, 1) − (5, 4) = (−1, −3).

9.16. Let vW C be the velocity of the wind relative to the cyclist, vW the velocity of the wind,
and vC the velocity of the cyclist. Suppose that vW = (a, b). Then, in case (i):

vW C = (a, b) − (0, 10) = (a, b − 10).

16
Since vW C has zero component in the northerly direction, it follows that b = 10. In case (ii)

vW C = (a, b) − (0, 20) = (a, b − 20) = (a, −10).

Since the wind appears to come from the north-west, a = 10. Its speed is
√ √
|vW | = [102 + 102 ] = 10 2km h−1 .

9.17. Let vS be the velocity of the ship, vW the velocity of the wind and vW S be the velocity of
the wind relative to the ships. In both cases vW S = vW − vS . Let vW = (a, b). Then

(a, b) − (0, −u) = (a, b − u) has direction −î.

Hence b = u, and a is negative. Secondly


µ ¶ µ ¶
2u 2u
(a, b) − − √ ,0 = a + √ ,b
3 3
has direction à !
b
tan 2u = −120◦ .
a+ √ 3

Therefore √
u
2u = − 3,
a + √3
so that √ √
u = − 3a − 2u, or a = − 3u.

The true velocity of the wind is vW = ( 3u, u).
9.18. The terminal point

R = a + 2r = (2, 3, 1) + 2(1, 1, 2) = (4, 5, 5).

9.19. (a) 0◦ , 90◦ , 90◦ ; (b) 45◦ , 45 ◦ ◦


√ , 90 ; ◦
(c) 90◦ , 90◦ , 180


; (d) arctan(1/ 3) = 54.7 , 54.7◦ , 54.7◦ ;
(e) arctan(1/ 3) = 54.7◦ , 54.7◦ , 125.3◦ .
9.20. (a) The position vector of S is given by (a sketch of the points is helpful)

OS = OP + (P Q + P R) = (1, 1, 0) + (0, 0, 1) + (0, 1, 1) = (1, 2, 2).

(b) The diagonal P S = P Q + P R = (0, 1, 2). Hence the coordinates of the midpoint are

OP + 12 P S = (1, 1, 0) + 12 (0, 1, 2) = (1, 32 , 1).

(c) As in (b) the midpoint of QR has coordinates

OQ + 12 QR = OQ + 12 (QS + QP ) = (1, 1, 1) + 12 [(0, 1, 1) + (0, 0, −1)] = (1, 23 , 1),

which has the same coordinates as the midpoint of P S.


The coordinates of the midpoints are:
A has coordinates OP + 12 P R = (1, 1, 0) + 12 (0, 1, 1) = (1, 32 , 12 );
B has coordinates OP + P R + 12 RS = (1, 1, 0) + (0, 1, 1) + 12 (0, 0, 1) = (1, 2, 23 );
C has coordinates OP + P Q + 12 QS = (1, 1, 0) + (0, 0, 1) + 12 (0, 1, 1) = (1, 23 , 23 );
D has coordinates OP + 12 P Q = (1, 1, 0) + 12 (0, 0, 1) = (1, 1, 21 ).
Hence ABCD is a parallelogram since AB = (0, 21 , 1) = DC.

17
9.21. Since AB = (2, 1, −1) and AC = (−4, −2, 2), then AC = −2AB.
(a) Since AC is of the opposite sign to AB, then A is between C and B.
(b) If P is a point on the line, then OP = OA + AP = OA + λAB, where λ is a parameter.
(c) As in (b)

(x, y, z) = (2λ + 1, λ + 2, −λ − 1) = λ(2, 1, −1) + (1, 2, −1) = λAB + OA,

which is the equation of the straight line.


9.22. (a) By the triangle law AB = b − a. Then
1 1 1
OC = OA + AB = a + (b − a) = (a + b).
2 2 2

(b) As in (a)
1 3 1
OU = OA + (b − a) = a + b.
4 4 4
(c) As in (b)
1 1 3 1
OV = OA + AV = OA + BA = a + (a − b) = a − b.
2 2 2 2

9.23. Let OA = a and OB = b.


(a) We are given that AU/U B = λ where 0 < λ < 1. Hence AU = λU B, and

OU = OA + AU = a + λU B = a + λ(b − OU ).

Hence
1
OU = (a + λb).
1−λ
(b) Since V does not lie between A and B, AV = λBA, where 0 < λ < 1. Thus

OV = OA + AV = a + λ(OV − b).

Therefore
1
OV = (a − λb).
1+λ
(c) If λ > 1, then B is outside AB, in the direction AB.
9.24. (a) The vector (2, 3, 4) − (1, 2, 3) = (1, 1, 1) is a vector in the direction of the line. If r is the
position vector a general point on the line, then

r = (x, y, z) = (1, 4, 2) + λ(1, 1, 1), or x = 1 + λ, y = 4 + λ, z = 2 + λ.

(b) From (a) λ = x − 1 = y − 4 = z − 2. The equation of the line can be represented in the form
of two simultaneous equations
x − 1 = y − 4 = z − 2.

(c) Using the representation in (b), the line intersects the (x, y) plane where

x − 1 = y − 4 = −2, that is, at (−1, 2, 0).

Similarly, the line meets the (y, z) plane where

−1 = y − 4 = z − 2, that is, at (0, 3, 1).

(d) Using these points the equation can be expressed in the form

r = (−1, 2, 0) + ν[(−1, 2, 0) − (0, 3, 1)] = (−1, 2, 0) + ν(−1, −1, −1).

18
Two more equations for the line could be

x + 1 = y − 2 = z.

9.25. The position vector of P can be expressed as

r = λa + (1 − λ)b = b + λ(a − b).

Hence P describes a straight line through A and B.


Looking at the line through AB, if λ < 0 then P lies on the extension of AB, if 0 < λ < 1, P
lies between A and B, and if λ > 1, P lies on the extension of BA.
9.26. The equation of a plane through the points a, b and c can be expressed in the from

r − a = λ(b − a) + µ(c − a).

(a) The plane is


r = (1, 0, 1) + λ(−1, 1, −1) + µ(−1, 0, 0).
Hence
x = 1 − λ − µ, y = λ, z = 1 − λ.
Eliminate λ between the second two equations gives the plane y + z = 1: x can take any value.
(b) The plane is
r = (0, 0, 0) + λ(1, 2, −1) + µ(2, 2, 2).
Hence
x = λ + 2µ, y = 2λ + 2µ, z = −λ + 2µ.
Eliminate λ giving
x + z = 4µ, y + 2z = 6µ.
Finally eliminate µ:
3x − 2y − z = 0
which is the equation of the plane.
9.27. The coordinates of any point on the line are (1 + t, 2 + t, 3 + t). The square of its distance
from the origin is

r2 = x2 + y 2 + z 2 = (1 + t)2 + (2 + t)2 + (3 + t)2 = 14 + 12t + 3t2 .

Then
d(r2 )
= 12 + 6t,
dt
which is zero where t = −2. The distance is a√stationary minimum
√ since the second derivative is
6 which is positive. The minimum distance is (14 − 24 + 12) = 2.
9.28. The unit vectors required are ±a/|a|.
(a) ±( √334 , √434 , √334 ); (b) ±( 27 , 37 , 67 );
(c) ±(− √16 , √16 , √26 ); (d) ±( √16 , − √26 , √16 );
(e) ±( √16 , − √26 , √16 ).

9.29. (a) The vector is (−2î + 3ĵ + k̂) − (î + ĵ + k̂) = −3î + 2ĵ + 4k̂ with length 29.

(b) 2î − 3ĵ − k̂, and length 14.
9.30. The vector equation of the line is

r = (î − ĵ + 2k̂) + λ(2î − 2ĵ − 3k̂).

19
If x = 0, then 1 + 2λ = 0 or λ = − 21 , in which case r = 72 k̂. The î and ĵ components are both zero
which means that the line cuts the z axis.
9.31. A square bounded by the 4 lines ±x ± y = 1.
9.32. An octahedron bounded by the 8 triangular faces ±x ± y ± z = 1.
9.33. The centre of mass is at

r̃ = 16 [1(1, 1, 2) + 2(−2, 3, 5) + 3(0, 3, 2)] = 16 (−3, 16, −2) = − 12 î + 83 ĵ − 13 k̂.

9.34. The vector equation of the line is r = (1, 1, 1) + λ(1, 2, −1). The line meets the plane where

−2 = x − y + z = (1 + λ) − (1 + 2λ) + (1 − λ0 = 1 − 2λ.

Hence λ = 32 . Therefore they intersect at 52 î + 4ĵ − 12 k̂.


9.35. The aircraft’s path is given by the position vector

r = î[P + R cos(V t/R)] + ĵ sin(V t/R) + H k̂.

9.36. Let OA and OB be unit vectors in the directions of a and b. Thus


a a
OA = â = , OB = b̂ = .
|a| |b|
If C is the midpoint of AB, then OC bisects the angle AOB. Hence
µ ¶
1 1 1 a b
OC = OA + AB = a + (b̂ − â) = + .
2 2 2 |a| |b|

9.37. (a) For A: its path is given by

rA = 0.41î + 148tĵ + 0.99k̂,

which describes a straight line since the components are linear in t. The velocity of A is
dr
vA = = 148ĵ.
dt
Hence its speed is 148 km.hr−1 . For B: its path is given by

rB = 100tî + 250tĵ + 250tk̂,

which also describes a straight line. The velocity of B is


dr
vB =
= 100î + 250ĵ + 250k̂.
dt
√ √
Its speed is [1002 + 2502 + 2502 ] = 150 6 ≈ 367 km.hr−1 .
(b) Let S(t) be the distance between A and B at time t. Then

S(t) = [(100t − 0.41)2 + (250t − 148t)2 + (250t − 0.99)2 ].

We require the minimum value of S(t), which is the same as that of [S(t)]2 . Differentiate [S(t)]2 :

d[S(t)]2
= 200(100t − 0.41) + 2 × 102 + 500(250t − 0.99),
dt
which is zero when t = 0.0035 hr=12.6 s. The minimum distance is S(0.0035) = 0.38 km, which is
a close encounter for aircraft.

20
9.38. (a) The position vector of B relative to A is rB − rA . The velocity of B relative A is
d drB drA
[rB − rA ] = − .
dt dt dt

(b) For the given points A : (t, −t2 , t) and B : (t3 , 2t2 , 1 + 3t), the velocity of B relative to A is
d
[(t − t3 )î + (−t2 − 2t2 )ĵ + (t − 1 − 3t)k̂] = (1 − 3t2 )î − 6tĵ − 2k̂.
dt
The velocity of A relative to B is the vector in the opposite direction, namely
drA drB
− = −(1 − 3t2 )î + 6tĵ + 2k̂.
dt dt

(c) The relative speed is v(t) = [(1 − 3t2 )2 + 36t2 + 4]. Since v(t) is never zero, any stationary
point of v(t) occurs at the same time as any stationary point of [v(t)]2 . Hence
d
[v(t)]2 = 12t(5 + 3t2 ),
dt
which is zero only when t = 0. The stationary value is a minimum since the second derivative is
obviously positive.
9.39. Given r = îa cos ωt + ĵ sin ωt, the velocity and acceleration are given by
dr
v= = −îaω sin ωt + ĵbω cos ωt,
dt
dv
a== −îaω 2 cos ωt − ĵbω 2 sin ωt = −ω 2 r.
dt
Hence the acceleration vector is in the opposite direction to r at the particle, and must, therefore,
be directed towards the origin.
9.40. The first and second derivatives of r = sec t and θ = t are

ṙ = sec t tan t, r̈ = 2 sec3 t − sec t, θ̇ = 1, θ̈ = 0.

From Example 9.15, the radial and transverse components of acceleration are given by

d2 r
= (r̈ − rθ̇2 )êr + (rθ̈ + 2ṙθ̇)êθ = 2 sec t tan2 t êr + 2 sec t tan t êθ .
dt2

9.41. Given r = îa cos ωt sin νt + ĵa sin ωt sin νt + k̂ cos νt,

|r|2 = a2 cos2 ωt sin2 νt + a2 sin2 ωt sin2 νt + a2 cos2 νt,


= a2 sin2 νt + a2 cos2 νt = a2

The velocity is given by


dr
v= = îa(−ω sin ωt sin νt + ν cos ωt cos νt)
dt
+ĵa(ω cos ωt sin νt + ν sin ωt cos νt) − k̂aν sin νt

Its magnitude is

|v| = a[(−ω sin ωt sin νt + ν cos ωt cos νt)2


1
+(ω cos ωt sin νt + ν sin ωt cos νt)2 + ν 2 sin2 νt] 2
1
= a(ν 2 + ω 2 sin2 νt) 2 = v(t), say.

21
To find the minimum and maximum speeds differentiate v(t) with respect to t:

dv(t) aω 2 ν sin νt cos νt


= 1 .
dt (ν 2 + ω 2 sin2 νt) 2
π π 3π
This zero when sin νt cos νt = 0, that is when t = 0, 2ν , ν , 2ν . At t = 0 and t = πν , the sign of
dv/dt changes from negative to positive as t increases through these values of t. Hence they are
both minima, and they occur at r = ±k̂a, the highest and lowest points of the sphere. Similarly,
π
maxima occur at the times t = 2ν and t = 3π
2ν . At these times the k̂ component of r is zero which
means that the maximum speeds occur on the equator of the sphere.

Chapter 10: The scalar product

10.1. (a) (2, 2, 1) · (3, 1, 2) = 10; (b) (2, −3, 2) · (−2, 3, −1) = −15;
(c) (2, 2, −3) · (−1, 1, −2) = 6; (d) (2, 3, 4) · (1, −2, 1) = 0;
(e) p − q, p + q, p) · (p + q, q, −p − q) = (p2 − q 2 ) + q(p + q) + p(−p − q) = 0.
10.2. (a) (2, 3) · (3, 4) = 18; (b) (1, 0) · (0, 1) = 0; (c) (5, 6) · (0, −4) = −24; (d) (2, 3) · (3, −2) = 0.

10.3. By (10.1c)

|a + b|2 + |a − b|2 = (a + b) · (a + b) + (a − b) · (a − b)
= a·a+a·b+b·a+b·b+a·a−a·b−b·a+b·b
= 2a · a + 2b · b = 2(|a|2 + |b|2 ),

using (10.1c) again.


10.4. (a) (2, −3, 4) · (−1, −2, 3) = 16.
(b) a · b = (2î − 3ĵ + 4k̂) · (−1î − 2ĵ + 3k̂) = −2 + 6 + 12 = 16,
since î · î = 1, î · ĵ = 0, and so on.
10.5. Given that a = î + 2ĵ − k̂, b = î + 3ĵ + k̂,
(a) a · b = (1, 2, −1) · (1, 3, 1) = 6;
(b) (a − b) · (a + b) = (0, −1, −2) · (2, 5, 0) = −5;
(c) (a − b) · (a − b) = (0, −1, −2) · (0, −1, −2) = 5;
(d)

a · a + 2a · b + b · b = (1, 2, −1) · (1, 2, −1) + 2(1, 2, −1) · (1, 3, 1) + (1, 3, 1) · (1, 3, 1)


= 6 + 12 + 11 = 29,

or note that it is the same as |a + b|2 .


(e) (a · a)b − (b · b)b = 6(1, 2, −1) − 11(1, 3, 1) = (6, 12, −6) − (11, 33, 11) = (−5, −21, −17)
10.6. Use (10.4) which states that the angle θ between a and b, in the range 0 ≤ θ ≤ 180◦ , is
given by
a·b
θ = arccos .
|a||b|
√ √
(a) θ = arccos[2/( 3 2) = 35.3◦ ;
(b) θ = arccos 0 = 90◦ ; √ √
(c) θ = arccos[(2 − 3 + 6)/( 14 14)] = arccos[5/14] = 69.1◦ .
10.7. (a) arccos 0 = 90◦√; √
(b) θ = arccos[(2 + 2)/( 5 5)] = arccos[4/5] = 36.9◦ ;
(c) θ = arccos 0 = 90◦ .
10.8. Imagine a cube placed with the origin of coordinate axes at one corner, and with three edges
coincident with the positive directions of the axes. Assume that these three edges are represented

22
by the vectors aî, aĵ and ak̂ (the cube has side-length a). The diagonal joins the origin (0, 0, 0)
to (a, a, a) which can be represented by the vector r = aî + aĵ + ak̂. The angle α between this
diagonal and, say, the x axis is given by
√ √
α = arccos[aî · r/(a.a 3)] = arccos[1/ 3] = 54.7◦ .

10.9. The position vector r of any point on the cone will be in the cone, and always make an angle
α with the axial unit vector â. Hence, by (10.4a) the equation of the cone is

â · r = |r| cos α.

Let r = xî + y ĵ + z k̂, â = 27 î − 73 ĵ − 67 k̂ and α = 60◦ . Then the cartesian equation of the cone is

( 27 , − 37 , − 67 ) · (x, y, z) = 12 (x2 + y 2 + z 2 ),

or, after squaring both sides,

4(2x − 3y − 6z)2 = 49(x2 + y 2 + z 2 ),

or
33x2 + 13y 2 − 95z 2 + 48xy − 144yz + 96zx = 0.

10.10. Let BC = a, CA = b, and AB = c. By the triangle law,

a = (−1, 1, −1), b = (3, 0, −1), c = (−2, −1, 2).

In each case the scalar product gives the supplement of the corresponding internal angle. Thus,
d β = CBA,
α = BAC, d γ = ACB, d then

b·c −8 8
cos(π − α) = = √ √ =− √ ,
|b||c| 10 9 3 10
c·a −1 1
cos(π − β) = = √ √ =− √ ,
|c||a| 9 3 3 3
a·b −2 2
cos(π − γ) = = √ √ = −√ .
|a||b| 3 10 30
We can now find the internal angles;

α = 32.5◦ , β = 78.9◦ , γ = 68.6◦ .

10.11. Using (10.1c)


1 1
(|a + b|2 − |a − b|2 ) = [(a + b) · (a + b) − (a − b) · (a − b)]
4 4
= a·b

10.12. From the definition of the scalar product

F·a |F||a| cos θ


= = |F| cos θ,
|a| |a|

which is the component of F in the direction of a. Due regard to sign occurs since 0 ≤ θ ≤ π. For
each of the vectors
F·a
= F · â = (8, 15, 9) · (2, 3, 6)/7 = (16 + 45 + 54)/7 = 115/7;
|a|

23
F · b̂ = (8, 15, 9) · (0, 3, 4)/7 = (45 + 36)/7 = 81/7,
F · ĉ = (8, 15, 9) · (2, 2, 1) = (16 + 30 + 9)/7 = 55/7.
Looking at the components in F = λa + µb + νc, we have

8 = 2λ + 2ν,

15 = 3λ + 3µ + 2ν,
9 = 6λ + 4µ + ν.
Now solve these equations by elimination: the solution is

λ = −13/11, µ = 310/11, ν = 57/11.

Hence
13 30 57
F = − 11 a+ 11 b + 11 c.

10.13. Form the scalar product of a and b:

a · b = (1, 3, 4) · (−2, 6, −4) = 0,

which means that the vectors are perpendicular.


The vector c = c1 î + c2 ĵ + c3 k̂ will be perpendicular to both a and b if c · a = 0 and c · b = 0.
This will be the case if
c1 + 3c2 + 4c3 = 0,
and
−2c1 + 6c2 − 4c3 = 0.
We have two equations in three unknowns, so specify one of them: put, say, c3 = 1. Then

c1 + 3c2 = −4, and − 2c1 + 6c2 = 4.

Hence c1 = −3 and c2 = − 13 . The solution is any (nonzero) multiple of (−9, −1, 3). Unit vectors
in the directions of c and −C are
(−9, −1, 3) 1
ĉ = ± √ =√ .
(81 + 1 + 9) 91

10.14. By (10.4), the angle is given by

1, 1, −1) · (2, −1, 2) √


arccos √ = arccos[1/ 3] = 101.1◦ .
3 3

Let c = c1 î + c2 ĵ + c3 k̂ be a vector perpendicular to both a and b. Then we must have


c · a = c · b = 0, which in component form become:

c1 + c2 − c3 = 0, and 2c1 − c2 + 2c3 = 0.

These are two equations in three unknowns, so specify one component, c1 = 1, and solve the two
equations for c2 and c3 . The result is c2 = −4, c3 = −3. Hence any vector which is perpendicular
to both a and b is any multiple of î − 4ĵ − 3k̂.
10.15. The two vectors are perpendicular if

(λ, 2, −1) · (1, 1, −3λ) = λ + 2 + 3λ = 2 + 4λ = 0.

Hence λ = − 12 .

24
10.16. The three vectors are mutually perpendicular if b · c = c · a = a · b = 0. Hence α, β, and
γ must satisfy
−2 + 2β − 6γ = 0, 2α + 2 + 9γ = 0, −α + 4β − 6 = 0.
By elimination the solutions are

α = − 25 , β = 75 , γ= 2
15 .

10.17. The vectors giving relevant edges of the tetrahedron are

AB = (−1, 1, 0), AD = (−1, y, z), BD = (0, y − 1, z),

BC = (0, 0, 1), DC = (0, 1 − y, 1 − z).


d is a right angle if BC · DC = (0, 0, 1) · (0, 1 − y, 1 − z) = 1 − z = 0. Hence z = 1. The
Then BCD
triangle ABD is equilateral if AB = BD = DA, that is, if
√ √ √
2 = [1 + (y − 1)2 ] = (y 2 + 2).

Obviously, y = 0. Hence D is the point (0, 0, 1).


10.18. The axes are shown in the figure. For a general point (x, y) whose coordinates are (X, Y )
in axes rotated through an angle α anticlockwise
y
Y 3 X

x
-2 -1 1 2

-1

Figure 4: Problem 10.18

x = X cos α − Y sin α, y = X sin α + Y cos α,


or solving for X and Y

X = x cos α + y sin α, Y = −x sin α + y cos α.



(a) Given α = 45◦ and P : (x, y) = (2, 2), X = 2 √12 + 2 √12 = 2 2, Y = −2 √12 + 2 √12 = 0.

(b) If Q : (X, Y ) = (1, −1), then x = √12 + √12 = 2, y = √12 − √12 = 0.
(c) Express x and y in terms of X and Y in the equation of the circle. It becomes
µ ¶2 µ ¶2
1 1 1 1
√ X − √ Y −1 + √ X+√ Y = 1,
2 2 2 2
which can be simplified to
µ ¶2 µ ¶2
1 1
X−√ + Y +√ = 1.
2 2

10.19. The lengths and direction cosines are (a) 1, (0, 1, 0); (b) 3, ( √13 , √13 , √13 );

(c) 3, ( 13 , − 23 , − 32 ); (d) 3, ( √13 , − √13 , √13 );

(e) 3, ( √13 , − √13 , − √13 ); (f) 7, ( 27 , 37 , 67 );
(g) 3, ( 13 , − 23 , 23 ); (h) 3, (0, 0, 1); (i) 3, (0, 0, −1).

25
10.20. (a) Check the scalar products:

19Y · 19Z = (15, −10, 6) · (10, 6, −15) = 150 − 60 − 90 = 0,

19Z · 19X = (10, 6, −15) · (6, 15, 10) = 60 + 90 − 150 = 0,


19X · 19Y = (6, 15, 10) · (15, −10, 6) = 90 − 150 + 60 = 0.
They are all zero: hence the vectors are mutually perpendicular. They are also all unit vectors
since for each 62 + 152 + 102 = 361 = 192 .
(b) The base vectors for OXY Z are Î = (6, 15, 10)/19, Ĵ = (15, −10, 6)/19, K̂ = (10, 6, −15)/19.
Hence using (10.13a)     
X 6 15 10 x
 Y  = 1  15 −10 6  y .
19
Z 10 6 −15 z
Similarly     
x 6 15 10 X
 y  = 1  15 −10 6  Y .
19
z 10 6 −15 Z
(c) From (b)       
X 6 15 10 1 56
 Y = 1  15 1
−10 6  2  =  7 .
19 19
Z 10 6 −15 2 −8
(d) The plane x + y + z = 0 in the new coordinates is

[(6X + 15Y + 10Z) + (15X − 10Y + 6Z) + (10X + 6Y − 15Z)]/19 = 0,

or
31X + 11Y + Z = 0.

10.21. The direction cosines are


3 4 12 6
(a) ±( 13 , 13 , 13 ); (b) ±( 19 , − 10 15
19 , 19 ).

10.22. We are given that for each particle with position vector r(t), its velocity is v = f (t)r. Let
s(t) be the position vector of another particle. The velocity of the particle with position vector r
relative to that with position vector s is
d
vs = (r − s) = f (t)r − f (t)s = f (t)(r − s).
dt
Hence the relative velocity obeys the same rule.
10.23. If α1 , α2 and α3 are the angles the vector makes with the axes then

cos2 α1 + cos2 α2 + cos2 α3 = 1.



In this problem cos α1 = cos 45◦ = 1/ 2, cos α3 = cos 60◦ = 12 , so that
1 1
2 + cos2 α2 + 4 = 1, or cos2 α2 = 14 .

Therefore cos α2 = ± 12 . Hence a makes either 60◦ or 120◦ with y axis.


10.24. The direction cosines are
3 4 12
(a) ±( 13 , 13 , 13 ).
6 −10 15
(b) ±( 19 , 19 , 19 ).
10.25. (a) The coefficients of λ give a vector in the direction of the line, namely (−1, 3, 1).
(b) The normal n to the plane is in the direction of the line. We can put n = (−1, 3, 1). Since the
plane passes through the origin, its equation is n · r = 0, that is, −x + 3y + z = 0.

26
(c) The plane passes through the origin. Therefore its equation takes the form px + qy + rz = 0.
The plane must also pass through two points on the line. Choose obvious points, such as λ = 1
giving the point (0, 5, 2) and λ = −1 giving (2, −1, 0). Therefore p, q, r must satisfy

5q + 2r = 0, 2p − q = 0.

Put q = 2 (or any nonzero value). Then p = 1 and r = −5.The equation of the is

x + 2y − 5z = 0.

10.26. The angle between the planes is the same as the angle between the normals. For any plane
ax + by + cz = p, the direction of its normal is the vector n = (p, q, r).
(a) From (10.22), the normals to the planes 2x − 3y + z = 2 and x − y = 0 are, respectively,
n1 = (2, −3, 1) and n2 = (1, −1, 0). If θ is the angle between the planes then
n1 · n2 5 5
cos θ = =√ √ = √ .
|n1 ||n2 | 14 2 2 7

Hence θ = 19.1◦ .
(b) The normals to the planes x + y + z = 0 and z = 0 are, respectively,
√ n1 = (1, 1, 1) and
n2 = (0, 0, 1). If θ is the angle between the planes then cos θ = 1/ 3. Hence θ = 54.7◦ .
10.27. Since the directions of the normals to the planes are given by a and b, the planes are
perpendicular if a · b = 0.
The direction of the normal to the plane x+y+z = 0 is (1, 1, 1), from(10.22). Any vector (p, q, r)
is perpendicular to (1, 1, 1) if p + q + r = 0. The set of all vectors perpendicular to the normal is
(p, q, −p − q) for any p and q. The set of all perpendicular planes is given by px + qy − (p + q)z = d
for any p, q and d.
10.28. (a) The normal vector of the plane ax + by + cz = d is (a, b, c) (see (10.22)) which is a fixed
vector.
(b) The normal to the plane 2x + y − z = 2 is n = (2, 1, −1). Since the line passes through the
origin, the position vector must be in the direction of n. Therefore r = λ(2, 1, −1).
(c) The point λ(2, 1, −1) must lie on the √ plane. Hence 4λ + λ√+ λ = 2, so that λ = 13 and the point
2 1 1
of intersection ( 3 , 3 , − 3 ). The length is [(4 + 1 + 1)/9] = (2/3).
(d) The planes are parallel. The line meets this plane where 4λ1 + λ1 + λ1 = 1 so that λ1 = 16 .
The point of intersection is ( 13 , 16 , − 16 ). The distance between the planes is the distance between
the two points of intersection, namely
"µ ¶ µ ¶2 µ ¶2 # · ¸
√ 2 1 2 1 1 1 1 √ 1 1 1 1
− + − + − + = + + =√ .
3 3 3 6 3 6 9 36 36 6

10.29. The parametric equation for the straight line through q perpendicular to the plane is
r − q = λp. The line intersects the plane p · r = d where

p · (λp + q) = d, so that λ = (d − p · q)/|p|2 .

The line meets the plane at the point


p
r1 = q + (d − p · q).
|p|2

The distance of Q from the plane is

|r1 − q| = |d − p · q|/p| = |p · q − d|/|p|.

27
For the given point and plane q = (1, 1, 2), p = (1, 2, −4) and d = −3. Using the formula above
the perpendicular distance is

|p · q − d| |(1, 2, −4) · (1, 1, 2) + 3| | − 5 + 3| 2


= √ = √ =√ .
|p| 21 21 21

10.30. (a) For the plane P1 through A, −ĵ + k̂ will be the direction of its normal. Hence (see
(10.22)) its equation is
0x − y + z = 0 + 1 + 3, or − y + z = 4.
Let the equation of P2 be ax + by + cz = d. It passes through A : (0, −1, 3), B : (1, 0, 3) and
C : (0, 0, 5). Hence
−b + 3c = d, a + 3c = d, 5c = d.
Let d = 5. Then c = 1, a = d − 3c = 2, and b = 3c − d = −2. The equation of P2 is

2x − 2y + z = 5.

(b) The normals of P1 and P2 are n1 = (0, −1, 1) and n2 = (2, −2, 1) so the angle θ between the
planes is
n1 · n2 2+1 1
cos θ = =√ √ =√ .
|n1 ||n2 | 2 9 2
Hence θ = 45◦ .
(c) The equation of the normal to P1 which passes through the origin is (x, y, z) = λn1 = λ(0, −1, 1).
This meets P1 where λ + λ √ = 4. Hence λ = √2, so that the line meets the plane at (0, −2, 2). The
perpendicular distance is [(−2)2 + 22 ] = 2 2. (The result can also be found using the formula
in Problem 10.29.)
(d) The line OD is given by
x = λ, y = 4λ, z = −4λ.
This meets P1 where −4λ − 4λ = 4, that is where λ = − 12 . Similarly the line meets P2 where
2λ − 8λ − 4λ = 5, from which it follows that λ = − 21 again. Since λ takes the same values on both
planes the points of intersection coincide, and the point must therefore lie on the line of intersection
of the planes.
(e) From (d), λ = − 12 at the point of intersection of L with OD. Therefore the point of intersection
has coordinates (− 12 , −2, 2).
10.31. Since F · n̂ = |F||n̂| cos θ = |F| cos θ (see (10.4)), it follows that the right-hand side is the
component in the direction n̂. Hence

F = Fs + (F · n̂)n̂.
√ √
For the straight line 2x − 3y = 1, we can choose ŝ = (3, 2)/ 13 and n̂ = (2, −3)/ 13. Then

Fn = (F · n̂)n̂ = ( √213 + √9 )( √2 , − √3 )
13 13 13
= ( 22 33
13 , − 13 ).

The other component Fs is given by

Fs = F − Fn = (1, −3) − ( 22 33 9 6
13 , − 13 ) = (− 13 , − 13 ).

10.32. (a) The component of û in the direction ŝ is

ûs = (û · ŝ)ŝ

so that
ûn = û − (û · ŝ)ŝ.

28
`
s2

`
u
`
u2
2Θ-60
`
u1
`
s1
60 Θ Θ

Figure 5: Problem 10.32

The vector û1 is û with the n̂ component reversed. Thus

û1 = ûs − ûn = −û + 2(û · ŝ)ŝ.

(b) In this example, ŝ = î, n̂ = ĵ, and û = √1 î + √1 ĵ. Therefore, by (a),


2 2
µ ¶ µ ¶
1 1 1 1 1
û1 = √ î + √ ĵ + 2 − √ î = − √ î + √ ĵ.
2 2 2 2 2

(c) For the first reflection, ŝ = î, n̂ = −ĵ, and

û = −î cos θ − ĵ sin θ.

Using (a)
û1 = î cos θ + ĵ sin θ + 2(− cos θ)î = −î cos θ + ĵ sin θ.
√ √
3 3
For the second reflection, ŝ2 = 12 î + 2 ĵ, n̂2 = 2 î − 12 ĵ. Therefore
√ √
3 3
û = î cos θ − ĵ sin θ + 2(− 12 cos θ + 2 sin θ)( 12 î + 2 ĵ)
√ √
3
= î( 12 cos θ + 2 sin θ) + ĵ(− 23 cos θ + 1
2 sin θ)
◦ ◦
= î cos(θ − 60 ) + ĵ sin(θ − 60 ).

10.33. For one point choose x = 0. Then y + z = 2 and y − 2z = 1. Solving these equations
y = 53 and z = 13 . For the other point choose z = 0. Then x + y = 2 and 2x + y = 1. solving
these equations x = −1 and y = 3. Two points on the line of intersection are a = (0, 35 , 31 ) and
b = (−1, 3, 0).
(b) A vector equation of the line of intersection (see Example 9.9a) is

r = b + λ(a − b) = (−1, 3, 0) + λ(1, − 34 , 31 ).

(c) A cartesian equation can be written down from (b):


x+1 y−3 z
= = 1.
1 − 43 3

10.34. Select any two points on the line of intersection of 2x + 3y − z = 1 and x + y + z = 0; say
a = (0, 14 , − 14 ) and b = ( 13 , 0, − 13 ). From(10.25) the line of intersection can be expressed as
1 1
x y− 4 z+ 4
1 = 1 − 1 .
−3 4 12

(Note: an infinite number of alternative expressions may be obtained.)

29
10.35. Let (p, q, r) be a vector in the direction of the line of intersection. If the vector is parallel to
both planes it must be parallel to the line of intersection, and it must also therefore be perpendicular
to both normals of the planes. Hence

2p + 3q − 2r = 0, p − 3q + 2r = 0.

One obvious solution is p = 0, q = 2, z = 3. Hence direction ratios in the direction of the line of
intersection are 0, 2, 3 or any non-zero multiple of these numbers.
10.36. The plane P1 with normal vector n1 = ĵ + k̂ is y + z = d. This plane passes through
B : (−1, −2, 0) if −2 = d. Hence P1 has the equation y + z = −2. The plane P2 with normal vector
n2 = 2î − ĵ + 3k̂ is 2x − y + 3z = h. This plane passes through C : (−1, 0, 3) if −2 + 9 = h. Hence
h = 7, and P2 has the equation 2x−y +3z = 7. The vector AC = (−1, 0, 3)−(−1, −2, 1) = (0, 2, 2),
which is a multiple of n1 , the normal of P1 . Hence AC is perpendicular to P1 .
10.37. The vector (p, q, r) is perpendicular to both (a1 , b1 , c1 ) an (a2 , b2 , c2 ), the normals to the
two planes, and therefore must be parallel to the line of intersection. The components p, q, r are
possible direction ratios for this line.
10.38. Let the line EQ meet the screen at the point (X, Y, Z). Then, comparing directions,

X −1 Y −1 Z −1
= = = λ.
x−1 y−1 z−1

The point (X, Y, Z) lies on the plane 1.1x + 1.1y + z = 1 if

1.1[1 + λ(x − 1)] + 1.1[1 + λ(y − 1)] + 1 + λ[1 + λ(z − 1)] = 1,

Hence
1.2
λ=− .
1.1x + 1.1y + z − 3.2
The apparent position is
(1 + (x − 1)λ, 1 + (y − 1)λ, 1 + zλ).

10.39. Pythagoras’ theorem for an increment of arc-length gives

(δs)2 ≈ (δx)2 + (δy)2 .

Divide through by (δt)2 and let δt → 0 (and take the square root):
"µ ¶ µ ¶2 #
2
ds √ dx dy √
= + = [a2 sin2 t + b2 cos2 t].
dt dt dt

The unit tangent vector is

dr dr dt (−a sin t, b cos t)


t̂ = = =√ 2 2 .
dt dt ds [a sin t + b2 cos2 t]

A vector normal to t̂ is
dt̂ ab(−b cos t, −a sin t)
n= = 3 .
dt (a2 sin2 t + b2 cos2 t) 2
A unit normal is
(−b cos t, −a sin t)
n̂ = √ .
(a2 sin2 t + b2 cos2 t)
The curvature κ is given by
dt̂
= κn̂.
ds

30
Hence
ab
κ= 2 3 .
(a2 sin t + b2 cos2 t) 2
The radius of curvature
3
ρ = 1/|κ| = (a2 sin2 t + b2 cos2 t) 2 /(ab).
At t = 0,
t̂ = (0, 1); n̂ = (−1, 0); κ = a/b2 ; ρ = b2 /a.
At t = 14 π,
3 3
(−a, b) (−b, −a) 2 2 ab (a2 + b2 ) 2
t̂ = √ ; n̂ = √ 2 ; κ= 3 ; ρ= 3 .
(a2 + b2 ) (a + b2 ) (a2 + b2 ) 2 2 ab
2

At t = 12 π,
t̂ = (−1, 0); n̂ = (0, −1); κ = b/a2 ; ρ = a2 /b.

10.40. With x as the parameter, the arc-length satisfies


ds √
= [1 + f 0 (x)2 ]
dx
(since δs2 = δx2 + δy 2 ). The unit tangent vector is

dr dr dx î + f 0 (x)ĵ
t̂ = = =√ .
dt dx ds [1 + f 0 (x)2 ]

A normal vector n is given by

dt̂ −f 0 (x)f 00 (x)î + f 00 (x)ĵ


n= = 3 .
dx [1 + f 0 (x)2 ] 2

Further
dt̂ −f 0 (x)f 00 (x)î + f 00 (x)ĵ n̂
= 0 2 2
=− .
ds [1 + f (x) ] [1 + f 0 (x)2 ]2
Hence
3
κ = −f 00 (x)/[1 + f 0 (x)] 2 ,
(the sign of κ depends on the direction of the unit normal).
(a) For the parabola y = x2 , f (x) = x2 . Hence
3
κ = 2/(1 + 4x2 ) 2 .

(b) For the cosine curve y = cos x, f (x) = cos x. Hence


3
κ = − cos x/(1 + sin2 x) 2 .

Chapter 11: Vector product

11.1. Given a = (1, −2, 2), b = (3, −1, −1), and c = (−1, 0, −1):
(a) a × b = (4, 7, 5); (b) b × a = (−4, −7, −5); (c) a × a = 0;
(d) a · (b × c) = −9; (e) c · (a × b) = −9; (f) b · a × c) = 9;
(g) (a × b) · b = 0; (h) a × (a × b) = (−24, 3, 15);
(i) (c × b) × a = (−6, 3, 6).

31
11.2. The vector n1 × n2 is a vector perpendicular to both normals n1 and n2 , and therefore must
be in the direction of the line of intersection of the two planes. Hence the required plane must
have this vector as its normal, and since it passes through the origin it can be represented by

r · (n1 × n2 ) = 0.

11.3. (a) r = a + λ(b × c), since (r − a) · b = λb · (b × c = 0, and similarly (r − a) · c = 0.


(b) r = (1, 2, 1) + λ(1, −1, 0) × (0, 1, 1) = (1, 2, 1) + λ(−1, −1, 1).
11.4. The vector a is perpendicular to the plane r · a = d, and a × u is perpendicular to a.
Therefore a × u is parallel to the plane.
In this example a = (2, −3, −1). Choose two simple vectors for u, say, u1 = î and u2 = ĵ, in
which case two parallel vectors are

a × u1 = (2, −3, −1) × (1, 0, 0) = (0, 1, −3),

a × u2 = (2, −3, −1) × (0, 1, 0) = (−1, 0, −2).

11.5. Since, by (11.8),


|a × b| = |a||b| sin θ,
the vector product will be zero if a = 0 or b = 0 or θ = 0 or θ = 180◦ .
11.6. The vectors a = 2î + 3ĵ + 6k̂ and b = 6î + 2ĵ − 3k̂ are perpendicular since

a · b = (2, 3, 6) · (6, 2, −3) = 12 + 6 − 18 = 0.

The vectors a, b, c form a right-handed system if

c = a × b = (2, 3, 6) × (6, 2, −3) = (−21, 42, −14).

11.7. (a) Two sides are AB = b − a and CA = c − a. As in Example 11.2 the area of the triangle
ABC is
1 1
|(b − a) × (c − a)| = |b × c − a × c − b × a + a × a|
2 2
1
= |c × c + c × a + a × b|
2
(b) The first vertex a is moved to a + λa(b − c) which is the same distance as a from the side BC.
In other words the height of the triangle on the same base is the same as that of ABC. Hence the
area will be the same as that of ABC.
(c) In (a) take a = (1, −2, −1), b = (1, −1, 2) and b = (1, −1, 2) and c = (1, 2, −1). Then
1
Area = |(1, −1, 2) × (1, 2, −1) + (1, 2 − 1) × (1, −2, −1) +
2
(1, −2, −1) × (1, −1, 2)|.
= (6, 0, 0)

11.8. Let E be the foot of the perpendicular from D on to the plane, and let θ be the angle
between AD and DE. The vector b × c is a vector in the direction of ED. From a property of the
scalar product
d · (b × c) = |d||b × c| cos θ.
Since ED = |d| cos θ, the required perpendicular distance is (taking account of the possibility that
cos θ < 0)
ED = |d · (b × c)|/|b × c|.

32
11.9. The answer is essentially given by equation 11.11. Let

a = QA = (x1 − x0 , y1 − y0 , z1 − z0 ), b = QB = (x2 − x0 , y2 − y0 , z2 − z0 ),

c = QB = (x3 − x0 , y3 − y0 , z3 − z0 ).
By (11.1) and (11.10), the volume V of the parallelepiped is given by

V = |a · (b × c)| = | det A1 |,

where ¯ ¯
¯ x1 − x0 x2 − x0 x3 − x0 ¯
¯ ¯
A1 = ¯¯ y1 − y0 y2 − y0 y3 − y0 ¯,
¯
¯ z1 − z0 z2 − z0 z3 − z0 ¯
after a transpose between rows and columns in the determinant: this does not affect the answer.
11.10. (a) Let a, b, c be any three given vectors, no two of which are parallel, and let v be an
arbitrary vector. We may express v in the form

v = Xa + Y b + Zc,

by choosing the coefficients X, Y , Z suitably. (X, Y , Z may be called the components of v in


terms of the oblique axes a, b, c, which need not always be unit vectors.) The components can
be worked out as follows.
Form the scalar product of v with b × c. Since b · (b × c) and c · (b × c) are zero by (11.10d),
we obtain
v · (b × c) = Xa · (b × c).
Therefore
v · (b × c)
X= .
a · (b × c)
Similarly
v · (c × a) v · (a × b)
Y = , Z= .
a · (c × a) a · (a × b)
(b) For the given vectors a = (1, 1, 0), b = (0, 1, 1), c = (1, 0, 1),

D = a · (b × c) = 2, v · (b × c) = 1, v · (c × a) = 1,

v · (a × b) = 1.
1
Hence X = Y = Z = 2.

11.11. Let v = (v1 , v2 , v3 ), a = (a1 , a2 , a3 ), etc. Hence

v1 = a1 X + b1 Y + c1 Z,

v2 = a2 X + b2 Y + c2 Z,
v3 = a3 X + b3 Y + c3 Z.
Let ¯ ¯ ¯ ¯
¯ a1 b1 c1 ¯ ¯ v1 b1 c1 ¯
¯ ¯ ¯ ¯
D = ¯¯ a2 b2 c2 ¯ , D1 = ¯ v2 b2 c2 ¯ ,
¯ ¯ ¯
¯ a3 b3 c3 ¯ ¯ v3 b3 c3 ¯
¯ ¯ ¯ ¯
¯ a1 v1 c1 ¯¯ ¯ a 1 b 1 v1 ¯
¯ ¯ ¯
D2 = ¯¯ a2 v2 c2 ¯¯ , D3 = ¯¯ a2 b2 v2 ¯¯ ,
¯ a3 v3 c3 ¯ ¯ a 3 b 3 v3 ¯
By elimination, it can be shown that

X = D1 /D, Y = D2 /D, Z = D3 /D.

33
11.12. (a) If v = X(b × c) + Y (c × a) + Z(a × b), then the scalar multiplication of v by a, b and
c leads to
a · v = Xa · (b × c),
b · v = Y b · (c × a) = Y a · (b × c),
c · v = Zc · (a × b) = Za · (b × c).
In these results we have used (11.10d) and (11.10b).
(b) a · (b × c) = −6, v · v = 1, b · v = 4, c · v. Hence X = − 16 , Y = − 23 , Z = − 56 .
11.13. (a) The line L1 is in the direction u and L2 is in the direction v. The vector product of
these vectors is perpendicular to both. Therefore, let w = u × v.
(b) The position vector (a + λu) is a point on L1 and (b + µv) is a point on L2 . The difference is
a vector joining the two points, which will be perpendicular to both if it is in the direction of w,
that is, if
(b + µv) − (a + λu) = νw.
This represents three equations for the three unknowns λ, µ, ν.
(c) The equation in (b) becomes

(1, −1, 0) + µ(1, 1, 1) − (−1, 0, 0) − λ(0, 0, 1) = ν(0, 0, 1) × (1, 1, 1) = ν(−1, 1, 0).

Equating components and solving: λ = − 12 , µ = − 12 , ν = − 32 . Hence, the end-points of the


perpendicular line are

a1 = a + λu = (−1, 0, 0) − 12 (0, 0, 1) = (−1, 0, − 21 ) on L1 ,

and
b1 = b + µv = (1, −1, 0) − 12 (1, 1, 1) = ( 12 , − 23 , − 12 ) on L2 .
A vector equation for L3 is

r = a1 + νu × v = (−1, 0, − 21 ) + ν(−1, 1, 0).



The perpendicular distance is |b1 − a1 | = 32 2.
11.14. If the position vector of P is r then the moment M of the force F about the point Q with
position vector a is given by M = (r − a) × F.
(a) ¯ ¯
¯ î ĵ k̂ ¯
¯ ¯
F = ¯¯ 0 3 0 ¯¯ = −6k̂.
¯ 2 0 0 ¯

(b) ¯ ¯
¯ î ĵ k̂ ¯
¯ ¯
F = ¯¯ 0 3 −3 ¯ = −6ĵ − 6k̂.
¯
¯ 2 0 0 ¯

(c) ¯ ¯
¯ î ĵ k̂ ¯
¯ ¯
F = ¯¯ 0 −3 −3 ¯ = − − 6ĵ + 6k̂.
¯
¯ 2 0 0 ¯

11.15. Since the magnitude of F is 4, and a unit vector in the given direction is 31 (î − 2ĵ − 2k̂),

F = 43 (î − 2ĵ − 2k̂).

(a) M = (1, −1, 2) × 43 (1, −2, −2) = 43 (6, 4, −1).


(b) M = (3, −2, 0) × 34 (1, −2, −2) = 43 (4, 6, −4).

34
(c) The component is ĵ · (R × F) = (0, 1, 0) · 43 (6, 4, −1) = 16
3 in case (a).
11.16. The moment about an axis in the direction ŝ is ŝ · (R × F).
(a) ŝ = k̂. Hence, using (11.10a),
¯ ¯
¯ 0 0 1 ¯¯
¯
ŝ · (R × F) = ¯¯ 0 3 0 ¯¯ = −6.
¯ 2 0 0 ¯

(b) ŝ = −k̂. Hence ¯ ¯


¯ 0 0 −1 ¯
¯ ¯
ŝ · (R × F) = ¯¯ 0 3 0 ¯ = 6.
¯
¯ 2 0 0 ¯

(c) ŝ = î. Hence ¯ ¯


¯ 1 0 0 ¯¯
¯
ŝ · (R × F) = ¯¯ 0 3 0 ¯¯ = 0.
¯ 2 0 0 ¯
(d) ŝ = ĵ. Hence ¯ ¯
¯ 0 1 0 ¯
¯ ¯
ŝ · (R × F) = ¯¯ 0 3 0 ¯=0
¯
¯ 2 0 0 ¯

(e) ŝ = √1 î + √1 ĵ + √1 k̂. Hence


3 3 3
¯ ¯
¯ 1 1 1 ¯¯
1 ¯¯ √
ŝ · (R × F) = √ ¯ 0 3 0 ¯¯ = −2 3.

2 0 0 ¯

11.17. The axis AB is in the direction


√ (1, 1, 1) − (2, 3, 2) = (−1, −2, −1). The unit vector in the
direction AB is ŝ = (−1, −2, −1)/ 6. Let

R1 = AP = (2, −3, 1) − (2, 3, 2) = (0, −6, −1),

R2 = BP = (2, −3, 1) − (1, 1, 1) = (1, −4, 0).


Then the moment about AB using point A is
¯ ¯
¯ −1 −2 −1 ¯
1 ¯¯ ¯
¯ = √7 .
ŝ · (R1 × F) = √ ¯ 0 −6 −1 ¯
6¯ ¯ 6
1 1 2

The answer can be checked using the alternative point B:


¯ ¯
¯ −1 −2 −6 ¯
1 ¯ ¯
¯ = √7 .
ŝ · (R2 × F) = √ ¯¯ 1 −4 0 ¯
6¯ ¯ 6
1 1 2

Any point on point X on AB is

r = −OA + λAB = (2, 3, 2) + λ(−1, −2, −1).

A vector in the direction P X is

R3 = (2, 3, 2) + λ(−1, −2, −1) − (2, −3, 1) = (0, 6, 1) + λ(−1, −2, −1).

The component of F in this direction will be a multiple of R3 , and its vector product with R3 will
be zero which means that its contribution to the moment will be zero.

35
11.18. The moment of F at r about an axis through the origin in the direction ŝ is

M = ŝ · (r × F).

In this formula r × F is fixed. Taking the magnitude of M,

|M| = |ŝ · (r × F)| = |ŝ||r × F| cos θ = |r × F| cos θ.

This is a maximum when θ = 0, which occurs when ŝ is in the direction of r × F.


11.19. (a) A point P on the lamina has position vector

v = r = r cos θî + r sin θĵ,

where r is constant. The velocity of P is

ṙ = −r sin θ θ̇î + r cos θ θ̇ĵ = −îωr sin θ + ĵωr cos θ.

(b) If ω = ω k̂, then


¯ ¯
¯ î ĵ k̂ ¯
¯ ¯
v = ¯¯ 0 0 ω ¯ = −îωr sin θ + ĵωr cos θ.
¯
¯ r sin θ r cos θ 0 ¯

(c) Let OP = r and OQ = q. We are given that R = QP , so that, by the triangle law,

r = OP = OQ + QP = q + R.

The velocity of P relative to Q is

Ṙ = ṙ − q̇ = ω × r − ω × q = ω × R.

11.20. Let a point P of the body have position vector r. Let Q be the foot of the perpendicular
from P on to the axis of ω, and let θ (0 ≤ θ ≤ π) be the angle between r and ω. Since OP
is constant, the point P must move perpendicular to OP (follows since r · ṙ = 0), whilst since
QP is constant P must move perpendicular to QP . To satisfy both conditions P must move
perpendicular to ω.
Since v is perpendicular to both ω and r, it follows that v = kω × r for some value of the
constant k. Taking the magnitude of v,

|v| = |k||ω||r| sin θ = |k|ωP Q.

Since ω is the angular rate, it follows that |k| = 1. in order that v, ω, and r form a right-handed
system of vectors, we must put k = 1.
Let ω = (ω1 , ω2 , ω3 ). From the vector product expansion,

v = ω × r = (zω2 − yω3 , xω3 − zω1 , yω1 − xω2 ).

This can be expressed as the matrix product


  
0 z −y ω1
v =  −z 0 x   ω2  or Sω,
y −x 0 ω3

interpreting v and ω as column vectors. Note that S is a skew-symmetric matrix (see Section 7.3).
Still using matrix notation
|v|2 = vT v = ω T S T Sω,

36
(use the rule for transpose of a product given in Section 7.3). Finally
    2 
0 −z y 0 z −y y + z2 −xy −zx
T
S S=  z 0 −x   −z 0 x  =  −xy z 2 + x2 −yz  .
−y x 0 y −x 0 −zx −yz y + z2
2

11.21. By the definition of the vector product a × b is perpendicular to both a and b. Also
a × (a × b) is perpendicular to both a and a × b, and must therefore lie in the plane of a and b.
A similar argument can be applied to b × (a × b).
11.22. Use the identity (11.18) for the vector triple product in two different expansions. In the
first
v = (a × b) × (c × d) = [(a × b) · d]c − [(a × c) · c]d,
which confirms that v lies in the plane of c and d The coefficients are

m = (a × b) · d, n = (a × b) · d.

Similarly
v = −(c × d) × (a × b) = −[(c × d) · b]a + [(c × d) · a]b,
which confirms that the vector also lies in the plane of a and b.The coefficients are

p = (c × d) · b, q = (c × d) · a.

Suppose that the vectors a, b, c, d are all drawn from the origin. Then v lis in the direction of the
line of intersection of the plane through a and b, and the plane through c and d.
11.23. Use (11.18) repeatedly:

a × (b × c) = b × (c × a) + c × (a × b) =
(a · c)b − (a · b)c + (b · a)c − (b · c)a + (c · b)a − (c · a)b = 0.

11.24. (a) The vector n × b is perpendicular to n and b. The vector n × (n × b) is perpendicular


to n and n × b and is also in the plane of n and b.
(b) The straight line
r = b + µn × [(a − b) × n]
passes through r when µ = 0.
Do the lines intersect? Can we find values of λ and µ for which this occurs? Consider the
difference of the position vectors of the two lines:

b + µn × [(a − b × n] − [a + λn]
= b + µn · n(a − b) − µ[n · (a − b)]n − a − λn
= (µn · n − 1)(a − b) − [λ + µn · (a − b)]n
= 0

if µ = 1/(n · n) and λ = −µn · (a − b). We can find solutions for λ and µ which means that the
lines intersect. The lines must be at right angles since the scalar product of their directions is

n · {n × [(a − b) × n]} = 0,

by (11.10d).
11.25. Let N be the closest point to the origin on the line of intersection. The direction of the
line of intersection is perpendicular to both normals n1 and n2 , that is in the direction n1 × n2 .
Also ON is perpendicular to the line of intersection so that ON · (n1 × n2 ) = 0. The two vectors

37
(n1 × n2 ) × n1 and (n1 × n2 ) × n2 are not parallel, and ON can be expressed in terms of these
two vectors:
ON = α[(n1 × n2 ) × n1 ] + β[(n1 × n2 ) × n1 ].
To obtain α and β, substitute this position vector into the equations for the planes:

β[(n1 × n2 ) × n2 ] · n1 = d1 ,

α[(n1 × n2 ) × n1 ] · n2 = d1 .
Alternative forms can obtained by using formula (11.18) for the vector triple products.
11.26. Given H = (r − q) × (mv),
· ¸
dH d dv
= (r − q) × (mv) + (r − q) × m
dt dt dt
= [(v − u) × mv] + (r − [q) × F]
= −[mu × v] + [(r − q) × F],

using v × v = 0 and Newton’s law F = m(dv/dt). If u = 0, then

dH
= (r − q) × F = M,
dt
the moment of F about Q.
Chapter 12: Linear algebraic equations

12.1. Generally Cramer’s rule (Section 12.1) is not a recommended way of solving linear equations
but it can occasionally be useful as a formula. For some problems just the answer is given.
(a) Given
x1 + x3 = 1
x2 − x3 = 3 .
2x1 + x2 = −1
Let ¯ ¯ ¯ ¯
¯ 1 0 1 ¯ ¯ 1 1 1 ¯¯
¯ ¯ ¯
D1 = ¯¯ 3 1 −1 ¯ = 5, D2 = ¯ 0
¯ ¯ 3 −1 ¯¯ = −9,
¯ −1 1 0 ¯ ¯ 2 −1 0 ¯
¯ ¯ ¯ ¯
¯ 1 0 1 ¯ ¯ 1 0 1 ¯¯
¯ ¯ ¯
D3 = ¯¯ 0 1 3 ¯ = −6, D = ¯ 0
¯ ¯ 1 −1 ¯¯ = −1.
¯ 2 1 −1 ¯ ¯ 2 1 0 ¯
Hence the solution is
D1 D2 D3
x1 = = −5, x2 = = 9, x3 = = 6.
D D D

(b) The solution is x1 = 188 21 36


5 , x2 = − 5 , x3 = − 5 .
(c) The solution is x1 = 1, x2 = −1, x3 = −5.
(d) Given
x1 + x2 + x3 = 1
ax1 + bx2 + cx3 = d .
a2 x1 + b2 x2 + c2 x3 = d2
Let ¯ ¯
¯ 1 1 1 ¯
¯ ¯
D1 = ¯¯ d b c ¯ = (b − c)(c − d)(d − b),
¯
¯ d2 b2 c2 ¯

38
¯ ¯
¯ 1 1 1 ¯
¯ ¯
D2 = ¯¯ a d c ¯ = (d − c)(c − a)(a − d),
¯
¯ a2 d2 c2 ¯
¯ ¯
¯ 1 1 1 ¯
¯ ¯
D3 = ¯¯ a b d ¯ = (b − d)(d − a)(a − b),
¯
¯ a2 b2 d2 ¯
¯ ¯
¯ 1 1 1 ¯
¯ ¯
D = ¯¯ a b c ¯ = (b − c)(c − a)(a − b).
¯
¯ a2 b2 c2 ¯
The solution is
(c − d)(d − b) (d − c)(a − d) (b − d)(d − a)
x1 = , x2 = , x3 = ,
(c − a)(a − b) (b − c)(a − b) (b − c)(c − a)
provided a, b, c are all different.
(e) The solution is x1 = 2, x2 = −1, x3 = 2, x4 = 2.
12.2. Given the equations
4i1 − i2 − i3 = 12 (i)
−i1 − Ri2 = 24 (ii) ,
i1 + 5i3 = −12 (iii)

i2 = (24 + i1 )/R from (ii), and i3 = (−12 − i1 )/5, from (iii). Substitute i1 and i2 into (i):
1 1
4i1 + (24 + i1 ) − (−12 − i1 ) = 12.
R 5
Hence
24(2R − 5) 552 12(5R − 1)
i1 = , i2 = − , i3 = − .
21R + 5 21R + 5 21R + 5
If i2 = 2 amps, then R = 1/51.
12.3. (a) The augmented matrix is
   
1 2 1 3 1 2 1 3
 1 −3 2 4  →  0 −5 1 (r20 = r2 − 5r1 )
1 
(r30 = r3 − 5r1 )
5 5 6 1 0 −5 1 −4
 
1 2 1 3
→  0 −5 1 1  (r30 = r3 − r2 ) .
0 0 0 −15
By row 3 the equations are inconsistent.
(b) The augmented matrix is
   
1 1 1 0 2 1 1 1 0 2
 1 0 1 2 3   0 −1 0 2 1  0
  →   (r20 = r2 − r1 )
 1 1 0 1 4   0 0 −1 1 2  (r3 = r3 − r1 )
0 −1 2 0 2 0 −1 2 0 2
 
1 1 1 0 2
 0 −1 0 2 1 
→ 
 0
 (r40 = r4 − r2 )
0 −1 1 2 
0 0 2 −2 1
 
1 1 1 0 2
 0 −1 0 2 1 
→ 
 0
 (r40 = r4 + 2r3 )
0 −1 1 2 
0 0 0 0 5

39
By row 4 the equations are inconsistent.
(c) The augmented matrix is
   
1 1 0 0 0 1 1 1 0 0 0 1
 0 1 1 0 0 1   0 1 1 0 0 1 
   
 0 0 1 1 0 1 → 0 0 1 1 0 1 ,
   
 0 0 0 1 1 1   0 0 0 1 1 1 
1 3 5 7 4 1 0 0 0 0 0 −9

after the following sequence of row operations

r50 = r5 − r1 , r50 = r5 − 2r2 , r50 = r5 − 3r3 , r50 = r5 − 4r4 .

From row 5 the equations are inconsistent.


12.4. The equations are
x + y − z = 2
2x + 3y + z = 3 .
5x + 7y + az = b
Using row operations on the augmented matrix,
   
1 1 −1 2 1 1 −1 2
 2 3 (r20 = r2 − 2r1 )
1 3  →  0 1 3 −1 
(r30 = r3 − 5r1 )
5 7 a b 0 2 a+5 b − 10
 
1 1 −1 2
→  0 1 3 −1  (r30 = r3 − 2r2 )
0 0 a−1 b−8

(i) The equations have a unique solution if a 6= 1, in which case z = (b − 8)/(a − 1) and x and y
can be found by back substitution.
(ii) The equations have no solutions if and only if a = 1 and b 6= 8.
(iii) There is an infinite set of solutions if a = 1 and b = 8, in which case

z = λ, y = −1 − 3λ, x = 3 + 4λ.

12.5. Row operations on the augmented matrix give


   
1 −1 2 1 1 −1 2 1
 1 (r20 = r2 − r1 )
 1 3 2 
 →
 0
 2 1 1  (r30 = r3 − r1 )
 1 2 −1 3   0 3 −3 2 
(r40 = r4 − r1 )
1 −2 6 0 0 −1 4 −1
 
1 −1 2 1
 0 2 1 1  0 3
→   (r30 = r3 − 21 r2 )
 0 0 −29 1  (r4 = r4 + r2 )
2 2
9 1
0 0 2 − 2
 
1 −1 2 1
 0 2 1 1 
→ 
 0
 (r40 = r4 + r3 )
0 − 29 21 
0 0 0 0

Row 4 is consistent, and the solution can be found by back substitution:

z = − 19 , y = 59 , x= 16
9 .

40
12.6. The augmented matrix is
   
1 1 1 −1 10 1 1 1 −1 10
 1 −1 −1 0 1  →  0 −2 −2 1 −9  ,
4 −2 −2 −1 5 0 0 0 0 −5

after the sequence of row operations

r20 = r2 − r1 , r30 = r3 − 4r1 , r30 = r3 − 3r2 .

By row 3, the equations are inconsistent.


12.7. Transform the augmented matrix by row operations into echelon form:
   
0 1 2 −1 11 1 1 1 1 1
 1 1 1 1 1   0 1 2 −1 11 
  →   (r1 ↔ r2 )
 2 1 −1 4 0   2 1 −1 4 0 
1 −1 1 −2 2 1 −1 1 −2 2
 
1 1 1 1 1
 0 1 2 −1 11  0
→   (r30 = r3 − 2r1 )
 0 −1 −3 2 −2  (r4 = r4 − r1 )
0 −2 0 −3 1
 
1 1 1 1 1
 0 1 2 −1 11  0
→   (r03 = r3 + r2 )
 0 0 −1 1 9  (r4 = r4 + 2r2 )
0 0 4 −5 23
 
1 1 1 1 1
 0 1 2 −1 11 
→   0
 (r40 = r4 + 4r3 )
0 −1 1 9 
0 0 o −1 59

By back substitution x4 = −59, x3 = −68, x2 = 88, x1 = 40.


12.8. After interchanging rows 1 and 3 (to cover the possibility that a may be zero), the augmented
matrix is
   
4 1 −2 2 4 1 −2 2 0 1
 1 2 −a 2  →  0 7 1−a 3  (r02 = r2 − 14 r1 )
4 2 2 (r3 = r3 − 4 ar1 )
a −1 2 1 0 − a+44
a+4
2
2−a
2
 
4 1 −2 2
→  0 74 1−a
2
3
2
 (r40 = r4 + 1 (a + 4)r2 )
7
16−a2 13−2a
0 0 7 7

Row 3 is inconsistent if a = ±4. If a is not equal to either of these values then the solution is
3 2(a + 1)(a − 5) 2a − 13
x= , y= , z= .
a+4 a2 − 16 a2 − 16

12.9. The method of Section 12.3 should be used. The inverses are
(a)  
1 1
18 0 − 18
 −5 1 1 
− 54 .
54 9
7 1 5
108 18 108
(b)  
1
5 0 − 15
 6
− 25 1 1 .
9 25
7 1 3
25 5 25

41
(c)  
0 1 −1 0
 −1 4 −4 2 
 .
 0 1 −1 1 
1 1
0 2 0 2

(d)  
1 −1 0 0 0 0
 0 1 0 0 0 0 
 
 0 0 1 0 0 0 
 .
 0 0 0 1 0 0 
 
 0 0 0 0 1 −1 
0 0 0 0 0 1
(e)  
1 0 0 0 0
 −1 1 0 0 0 
 
 0 −1 1 0 0 .
 
 0 0 −1 1 0 
0 0 0 −1 1

12.10. A matrix A is singular if det A = 0. For the matrix


 
1 0 1 0 1
 0 1 0 1 0 
 
A=  1 0 1 0 1 ,

 0 1 0 1 0 
1 0 1 0 1

rows 1 and 3 have the same elements. Hence det A = 0 (Section 8.2) and the matrix is singular.
12.11. We need to find the points of intersection every set of three planes chosen from

Figure 6: Problem 12.11

6x − 3y − z = −3
2x − y + 5z = 15
.
y + z = 1
2x + y − z = 1

This is equivalent to solving 4 sets of linear equations, which can be solved either by elimination
or by row operations. The 4 sets of solutions are

(−1, −2, 3), (1, 2, 3), (0, 1, 0), (2, −1, 2),

which are the vertices of the tetrahedron. The tetrahedron is shown in the Figure 6.

42
12.12. The equations of the straight lines P A, P B, P C are:
x−3 y−2 z−2
= = = λ,
2 1 1
x−3 y−2 z−2
= = = µ,
2 2 1
x−3 y−2 z−2
= = = ν.
1 1 1
The lines P A, P B, P C meet the plane x = 0 where λ = − 32 , µ = − 32 , ν = −3. Hence the corners
of the triangle projected on to the plane x = 0 are

(0, 12 , 12 ), (0, −1, 12 ), (0, −1, −1).

The lines P A, P B, P C meet the plane y = 0 where λ = −2, µ = −1, ν = −2. Hence the corners
of the triangle projected on to the plane y = 0 are

(−1, 0, 0), (1, 0, 1), (1, 0, 0).

The lines meet the plane z = 0 where λ = −2, µ = −2, ν = −2. Hence the corners of the triangle
projected on to the plane z = 0 are

(−1, 0, 0), (−1, −2, 0), (1, 0, 0).

12.13. The parabola y = α + βx + γx2 passes through the points (x1 , y1 ), (x2 , y2 ), (x3 , y3 ) if
solutions of
α + βx1 + γx21 = y1 ,
α + βx2 + γx22 = y2 ,
α + βx3 + γx23 = y3 ,
for α, β, γ can be found. Leaving aside existence of solutions for the moment, elimination or row
reduction gives
x2 x3 y1 (x3 − x2 ) + x3 x1 y2 (x1 − x3 ) + x1 x2 y3 (x2 − x1 )
α= ,
(x2 − x3 )(x3 − x1 )(x1 − x2 )

y1 (x22 − x23 ) + y2 (x23 − x21 ) + y3 (x21 − x22 )


β= ,
(x2 − x3 )(x3 − x1 )(x1 − x2 )
y1 (x2 − x3 ) + y2 (x3 − x1 ) + y3 (x1 − x2 )
γ= .
(x2 − x3 )(x3 − x1 )(x1 − x2 )
There are various exceptional cases. For example if x1 = x2 and y1 6= y2 then there are no solutions.
A similar result holds for the other pairs of x’s.
12.14. The augmented matrix for

x + y + z = 4
x − y + z = 2 ,
2x + y − λz = µ

is
   
1 1 1 4 1 1 1 4
 1 (r20 = r2 − r1 )
−1 1 2  →  0 −2 0 −2 
(r30 = r3 − r1
2 1 −λ µ 0 −1 −λ − 2 µ−8
 
1 1 1 4
→  0 −2 0 −2  (r30 = r3 − 12 r2 )
0 0 −λ − 2 µ−7

43
(a) There is just one solution if λ 6= −2.
(b) No solutions if λ = −2 and µ 6= 7.
(c) An infinite set of solutions if λ = −2 and µ = 7.
12.15. The augmented matrix is
   
1 1 1 3 1 1 1 3 0
 3 5 1 −1  →  0 2 −2 −10  (r20 = r2 − r3 )
(r3 = r3 − r1 )
1 2 0 0 0 1 −1 −3
 
1 1 1 3
→  0 2 −2 −10 
0 0 0 2

By row 3 the equations are inconsistent.


(b) The augmented matrix is
   
0 1 1 1 1 1 2 3
 1 1 2 3  →  0 1 1 1  (r1 ↔ r2 )
1 −1 0 1 1 −1 0 1
 
1 1 2 3
→  0 1 1 1  (r30 = r3 − r1 )
0 −2 −2 −2
 
1 1 2 2
→  0 1 1 1 .
0 0 0 0

The equations are consistent with solution z = λ, y = 1 − λ, x = 2 − λ.


(c) The augmented matrix is
   
1 2 1 4 1 2 1 4
 1 1 0 −1  →  0 −1 −1 −5  (r20 = r2 − r1 )
3 4 −1 12 0 −2 −4 0
 
1 2 1 4
→  0 −1 −1 −5  (r30 = r3 − 2r2 )
0 0 −2 10

There is a unique solution given by z = −5, y = 10, x = −11.


12.16. The determinant simplifies to
¯ ¯ ¯ ¯
¯ 1−k 2 −1 ¯ ¯ 4−k 4−k −4 + k ¯
¯ ¯ ¯ ¯
∆ = ¯¯ 2 1−k −1 ¯ =
¯
¯ 2
¯ 1−k −1 ¯ (r10 = r1 + r2 − r3 )
¯
¯ −1 −1 2 − k ¯ ¯ −1 −1 2−k ¯
¯ ¯
¯ 0 0 −1 ¯¯ 0
¯ (c = c1 + c3 )
= (4 − k) ¯¯ 1 −k −1 ¯¯ 10
¯ 1−k (c = c2 + c3 )
1−k 2−k ¯ 2
= (4 − k)(k − 1)(1 + k)

Hence the determinant is zero when k = −1, 1, 4.


The equations
(1 − k)x + 2y − z = 0
2x + (1 − k)y − z = 0
−x − y + (2 − k)z = 0
have non-trivial solutions (that is, not all zero) ,if and only if, ∆ = 0, that is, if k = −1, k = 1, or
k = 4.

44
Case k = −1. The equations are

2x + 2y − z = 0
2x + 2y − z = 0 .
−x − y + 3z = 0

The solution is x = λ, y = −λ, z = 0.


Case k = 1. the equations are

+ 2y − z = 0
2x + − z = 0 .
−x − y + z = 0

The solution is x = λ, y = λ, z = 2λ.


Case k = 4.
−3x + 2y − z = 0
2x + −3y − z = 0
−x − y − 2z = 0
The solutions is x = −λ, y = −λ, z = λ.
12.17. Simplifying the determinant:
¯ 2 ¯
¯ a +t ab cd ¯
¯ ¯
∆ = ¯¯ ab b2 + t bc ¯
¯
¯ ca bc c2 + t ¯
¯ ¯
¯ 1 1 1 ¯
¯
2 ¯ 2
¯
2 2
= (t + a + b + c ) ¯ b b2 + t b2 ¯ (r10 = ar1 + br2 + cr3 )
¯
¯ c2 c2 c2 + t ¯
¯ ¯
¯ 0 0 1 ¯¯
¯
2 ¯ (c01 = c1 − c3 )
2 2
= (t + a + b + c 0 ¯ 0 t b2 ¯¯
¯ −t (c02 = c2 − c3 )
−t c + t ¯
2

= t2 (t + a2 + b2 + c2 ).

The equations
(1 + t)x + 2y + 3z = 0
2x + (4 + t)y + 6z = 0
3x + 6y + (9 + t)z = 0
have non-trivial solutions, if and only if,
¯ ¯
¯ 1+t 2 3 ¯
¯ ¯
∆ = ¯¯ 2 4+t 6 ¯ = 0.
¯
¯ 3 6 9+t ¯

Put a = 1, b = 2, and c = 3 in the first determinant above. Then

∆ = t2 (t + 12 + 22 + 32 ) = t2 (t + 14) = 0

when t = 0 or t = −14.
Case t = 0. The equations are

1x + 2y + 3z = 0
2x + 4y + 6z = 0 .
3x + 6y + 9z = 0

There is effectively just one equation, so the solution will contain two parameters:
x = −2µ − 3λ, y = µ, z = λ.

45
Case t = −14. The equations become

−13x + 2y + 3z = 0
2x − 10y + 6z = 0
3x + 6y − 5z = 0

The solution is x = 23 λ, y = 13 λ, z = λ.
12.18. The equations have non-trivial solutions if, and only if,
¯ ¯ ¯ ¯
¯ k 4 −1 3 ¯¯ ¯ k+6 4 −1 3 ¯¯
¯ ¯
¯ 4 k −1 3 ¯¯ ¯ k+6 k −1 3 ¯¯
¯ ¯ (c01 = c1 + c2 + c3 + c4 )
¯ 4 −1 k 3 ¯ = ¯ k + 6 −1 k 3 ¯¯
¯ ¯ ¯
¯ 4 −1 3 k ¯ ¯ k + 6 −1 3 k ¯
¯ ¯
¯ 1 4 −1 3 ¯¯
¯
¯ 1 k −1 3 ¯¯
= (k + 6) ¯¯
¯ 1 −1 k 3 ¯¯
¯ 1 −1 3 k ¯
= (k + 6)(k + 1)(k − 3)(k − 4) = 0,

after further row operations. The equations have non-trivial solutions if k = −6, −1, 3, or 4.
12.19. The augmented matrix is
   
1 2 3 0 4 1 2 3 0 4
 2 3 8 −1 20  →  0 −1 (r20 = r2 − 2r1 )
2 −1 12 
(r30 = r3 − 2r1 )
2 5 4 1 5 0 1 −2 1 −3
 
1 2 3 0 4
→  0 −1 2 −1 12  (r30 = r3 + r2 )
0 0 0 0 9

By row 3 the equations are inconsistent.


12.20. Denote the matrices by A and B respectively. By row operations reduce A to the identity
matrix I3 , and then perform the same operations on I3 . Thus
   
1 λ 0 1 0 0
(r20 = r2 − λr3 )
A= 0 1 λ → 0 1 0 
(r10 = r1 − λr2 )
0 0 1 0 0 1

whilst
   
1 0 0 1 −λ λ2
 0 1 (r20 = r2 − λr3 )
0  →  0 1 −λ 
(r10 = r1 − λr2 )
0 0 1 0 0 1
= A−1

Similarly  
1 0 0
B −1 =  −µ 1 0 .
2
µ −µ 1
Let λ = 3 and µ = 4. Then  
13 3 0
 4 13 3  = AB.
0 4 1

46
Using the inverse formula (AB)−1 = B −1 A−1 ,
 
1 −λ λ2
(r20 = r2 − λr3 )
(AB)−1 =  0 1 −λ 
(r10 = r1 − λr2 )
0 0 1
 
1 0 0
=  −µ 1 0 
µ2 −µ 1
 
1 −3 9
=  −4 13 −39  .
16 −52 157

12.21. Denote the determinant by ∆. Then


¯ ¯ ¯ ¯
¯ 1 1 1 ¯ ¯ 1 0 0 ¯
¯ ¯ ¯ ¯
∆ = ¯¯ a2 b2 c2 ¯
¯ = ¯
¯ a2
b2
− a2 c2 − a2 ¯
¯
¯ a(b + c) b(c + a) c(a + b) ¯ ¯ a(b + c) c(b − a) b(c − a) ¯
(c02 = c2 − c1 ), (c03 = c3 − c1 )
¯ ¯
¯ a+b c+a ¯
= −(a − b)(c − a) ¯¯ ¯
c b ¯
= −(b − c)(c − a)(a − b)(a + b + c).

The equations have non-trivial solutions if, and only if, ∆ = 0. From the expansion above this
occurs if b = c, or c = a, or a = b or a + b + c = 0.
Case a + b + c = 0. The equations can be expressed as

x + a2 y − b2 z = 0,

x + b2 y − a2 z = 0,
x + (a + b)2 y − (a + b)2 z = 0.
The solution is
x = 0, y = λ, z = λ.

12.22. Write the equations as


x1 = 13 (−x2 − x3 + 5),
x2 = 16 (2x3 − 3x4 + 6),
x3 = 14 (−x1 + 2x4 + 1),
x4 = 14 (x2 + 2x3 − 2).
(0) (0) (0) (0) (1) (1) (1)
Start with the values x1 = 0, x2 = 0, x3 = 0 and x4 = 0, and compute x1 , x2 , x3 , . . .
(1)
in sequence, but update variables at the first opportunity. For example, x1 = 35 from the first
(0 (1)
computation and this value should replace x1 in the equation for x3 . The first two iterations
and the final numerical solutions are shown in the table:
x1 1.6667 1.3611 ... 1.3981
x2 1.0000 1.1181 ... 1.0900
x3 -0.0833 -0.2661 ... -0.2844
x4 -0.2917 -0.3385 ... -0.3697

The matrix of coefficients in the equations is


 
3 1 1 0
 0 6 −2 3 
 .
 1 0 4 −2 
0 1 2 −4

47
The system is diagonally dominant since 3 ≥ 1 + 1 = 2 in the first row, 6 ≥ | − 2| + 3 = 5 in the
second row, 4 ≥ 1 + | − 2| = 3 in the third row, and | − 4| ≥ 1 + 2 = 3 in the fourth row.
12.23. The matrix of coefficients is
 
1 −2 1
 1 −1 −1  .
2 3 −4

Looking at the diagonal elements, 1 < | − 2| + 1 = 3 in the first row, | − 1| < 1 + | − 1| = 2 in


the second row, | − 4| < 2 + 3 = 5 so that the system of equations is not diagonally dominant. In
(0) (0) (0)
this case the Gauss-Seidel scheme may or may not converge. Let x1 = 0, x2 = 0, x3 = 0. The
equations can be written as
x1 = 2x2 + x3 + 4.
x2 = x1 − x3 − 1.
x3 = 14 (2x1 + 3x2 − 4).
As the following table indicates, the sequences do not converge.

x1 4 6.75 4.75 2 ...


x2 3 2.5 -0.5 0 ...
x3 3.25 4.25 1 0 ...

However, the equations do have the unique solution x = 0.45, y = −0.55, z = 0.75 found by
elimination.
12.24. The matrix of coefficients is  
6 −1 1
 3 2 1 .
1 −1 4
Row 2 fails the dominant diagonal test.
The equations can be written as
x1 = 16 (x2 − x3 + 2),
x2 = 12 (−3x1 − x3 + 1),
x3 = 14 (−x1 + x2 + 5).
(0) (0) (0)
Let x1 = 0, x2 = 0, x3 = 1. Then the table shows the convergence of the Gauss-Seidel scheme:

x1 0.1667 0.1007 ... 0.1000


x2 -0.2500 -0.2240 ... -0.2333
x3 1.1458 1.1688 ... 1.1667

12.25. Equations (12.7)-(12.9) are


x1 = 13 (−x2 − x3 − 1),
x2 = 14 (x1 − x3 − 8),
x3 = 15 (−2x1 − x2 − 14).
(0) (0) (0)
Let x2 = 0, x3 = 0. Using the Jacobi scheme we have to specify additionally x1 . We have
(0) (1)
chosen, for comparison purposes, x1 = − 13 which is the first value x1 .

x1 -0.3333 1.2500 0.8889 ...


x2 -2.0833 -1.4167 -1.1250 ...
x3 -2.2500 -2.2500 3.0167 ...

This table can be compared with the one following eqn (12.12).

48
Chapter 13: Eigenvalues and eigenvectors

13.1. (a) The eigenvalues are given by


¯ ¯
¯ 2−λ 3 ¯¯
¯ = (2 − λ)(6 − λ) − 12 = λ(λ − 8) = 0.
¯ 4 6−λ ¯

Hence the eigenvalues are λ1 = 0, λ2 = 8.


The eigenvector si is given by [A − λi I2 ]si = 0, (i = 1, 2). If
· ¸
ai
si = ,
bi

then, in this example,


· ¸· ¸ · ¸
2 3 a1 0 2a1 + 3a2 = 0
= or .
4 6 b1 0 4a1 + 6a2 = 0

A particular convenient solution is · ¸


3
s1 = .
−2
Similarly, we can choose · ¸
1
s2 = .
2
(b) The eigenvalues and eigenvectors of
· ¸ · ¸ · ¸
6 3 −3 1
A= are λ1 = 4, λ2 = 9, s1 = s2 = .
2 7 2 1

(c) The eigenvalues and eigenvectors of


· ¸
2 1 √ √
A= are λ1 = 4 − 2 2, λ2 = 4 + 2 2,
4 6
· √ ¸ · √ ¸
−1 − 2 −1 + 2
s1 = , s2 = .
2 2
(d) The eigenvalues and eigenvectors of
· ¸
1 1 √ √
A= are λ1 = 3 − 2 2, λ2 = 3 + 2,
4 5
· √ ¸ · √ ¸
−1 − 2 −1 + 2
s1 = s2 = .
2 2
(e) The eigenvalues and eigenvectors of
· ¸
1 2 √ √
A= are λ1 = 3 − 4 2, λ2 = 3 + 4 2,
14 5
· √ ¸ · √ ¸
−1 − 2 2 −1 + 2 2
s1 = s2 = .
7 7
(f) The eigenvalues and eigenvectors of
· ¸
2 −2
A= are λ1 = 4 − 2i, λ2 = 4 + 2i,
4 6

49
· ¸ · ¸
−1 − i −1 + i
s1 = s2 = .
2 2

13.2. The eigenvalues of · ¸


a b
A=
b c
are given by the equation
¯ ¯
¯ a−λ b ¯
¯ ¯ = 0 or λ2 − (a + c)λ + ac − b2 = 0.
¯ b c−λ ¯

Hence √
λ = 12 {(a + c) ± [(a − c)2 + 4b2 ]}.
Since (a − c)2 + 4b2 ≥ 0, both eigenvalues are real.
13.3. The eigenvalues of · ¸
6 3
A=
2 7
are λ1 = 4 and λ2 = 9. The inverse of A is
· 7 1
¸
−1 36 12
A = 1 1 .
− 18 6

The eigenvalues of this matrix are given by


¯ ¯
¯ 6−µ 3 ¯¯
¯ = 0, or 36µ2 − 13µ + 1 = 0, or (4µ − 1)(9µ − 1) = 0.
¯ 2 7−µ ¯

Hence the eigenvalues are µ1 = 14 = 1/λ1 and µ2 = 19 = 1/λ2 . The eigenvalues of the inverse of a
matrix are their reciprocals of the eigenvalues of the matrix.
The square of A is · ¸
2 42 39
A = .
26 55
Its eigenvalues are λ21 = 16 and λ22 = 81.
13.4. (a) The eigenvalues of  
1 1 2
A= 1 2 1 
2 1 1
are given by
¯ ¯
¯ 1−λ 1 2 ¯
¯ ¯
¯ 1 2 − λ 1 ¯ = −λ3 + 4λ2 + λ − 4 = −(λ + 1)(λ − 1)(λ − 4).
¯ ¯
¯ 2 1 1−λ ¯

Hence the eigenvalues are λ1 = −1, λ2 = 1, λ3 = 4.


The eigenvector si associated with the eigenvalue λi , (i = 1, 2, 3) is given by any non-zero
solution of
[A − λi ]si = 0.
Thus s1 = [a1 b1 c1 ]T satisfies
  
2 1 2 a1 2a1 + b1 + 2c1 = 0
 1 3 1   b1  = 0, or a1 + 3b1 + c1 = 0
2 1 2 c1 2a1 + b1 + 2c1 = 0.

50
The general solution is
   
−k −1
s1 =  0  : a convenient choice is s1 =  0  .
k 1
Eigenvectors associated with λ2 and λ3 are
   
1 1
s2 =  −2  , s3 =  1  .
1 1

(b) Eigenvalues and eigenvectors are


     
−2 1 1
λ1 = 0, λ2 = 1, λ3 = 5, s1 =  −2  s2 =  −3  s3 =  1  .
3 1 1

(c) Eigenvalues and eigenvectors are


     
0 1 0
λ1 = −2, λ2 = 2, λ3 = 3, s1 =  −1  s2 =  0  s3 =  2  .
2 0 1

(d) Eigenvalues and eigenvectors are


     
−1 −1 1
λ1 = 1, λ2 = 1, λ3 = 16, s1 =  0  s2 =  1  s3 =  1  .
1 0 1

13.5. The eigenvalues are given by the solutions of


¯ ¯
¯ 1 2 0 0 ¯
¯ ¯
¯ 3 2 0 0 ¯
¯ ¯ 4 3 2 2
¯ 0 0 3 1 ¯ = λ − 9λ + 22λ − 32 = (λ + 1)(λ + 2)(λ − 4) = 0.
¯ ¯
¯ 0 0 1 3 ¯

Hence the eigenvalues are

λ1 = −1, λ2 = 2, λ3 = 4 (repeated).

The corresponding eigenvectors are


       
−1 0 0 2
 1   0   0   3 
s1 =   
 0  , s2 =  −1
,
 s3 =  
 1 , s4 =  
 0 .
0 1 1 0
Note that two independent eigenvectors are associated with the repeated eigenvalue λ3 = 4, since
  
−3 2 0 0 a3
 3 −2 0 0   
[A − λ3 ]s3 =    b3  = 0.
 0 0 −1 1   c3 
0 0 1 −1 d3

where s3 = [a3 b3 c3 d3 ]T . The linear equations become


−3a3 + 2b3 = 0,
3a3 − 2b3 = 0,
.
−c3 + d3 = 0,
c3 − d3 = 0.

51
The general solution contains two parameters, say α and β such that

a3 = 2α, b3 = 3α, c3 = β, d3 = β.

We can obtain two independent eigenvectors, one with β = 0 and α = 1, and one with α = 0 and
β = 1, which accounts for s3 and s4 above.
Note the relation between the eigenvalues of A and the eigenvalues of the two 2 × 2 submatrices
· ¸ · ¸
1 2 3 1
, .
3 2 1 3

13.6. The eigenvalues are given by


 
1−λ 0 0
 0 2 − λ 2  = −λ3 + 8λ2 − 13λ + 6 = −(λ − 1)2 (λ − 6) = 0.
0 2 3

Hence the eigenvalues are λ1 = 1 (repeated) and λ3 = 6. The eigenvector s1 = [a1 , b1 c1 ]T satisfies
  
0 0 0 a1
 0 1 2   b1  = 0, or b1 + 2c1 = 0 .
2b1 + 4c1 = 0
0 2 4 c1

The general solution can be expressed in terms of two parameters: a1 = α, b1 = −2β, c1 = β so


that  
α
s1 =  −2β  .
β
We can associate two independent eigenvectors with λ1 , one with α = 1, β = 0, and one with
α = 0, β = 1. Hence, three independent eigenvectors are
     
0 1 0
s1 =  −2  , s2 =  0  , s3 =  1  ,
1 0 2

where s3 is the eigenvector associated with λ3 .


13.7. The eigenvalues of A are given by
¯ ¯
¯ −1 − λ −1 a + 1 ¯¯
¯
¯ a + 1 −a − λ −1 ¯¯ = −λ(λ2 + (2a + 1)λ + 2a2 + 5a + 2) = 0.
¯
¯ −a a + 1 −a − λ ¯

Hence A as a zero eigenvalue. A second solution must be λ = 3 which means that a must satisfy

9 + 3(2a + 1) + 2a2 + 5a + 2 = 0, or 2a2 + 11a + 14 = 0, or (2a + 7)(a + 2) = 0.

The solutions are a = −2 and a = − 72 .


Case a = −2. det A = λ2 (3 − λ). The third eigenvalue is a repeated λ = 0.
Case a = − 72 . det A = −λ(λ − 3)2 . The third eigenvalue is a repeated λ = 3.
13.8. The eigenvalues of A occur where Ax = λx has non-trivial solutions. Since A2 = A,

Ax = λx ⇒ A2 x = λx ⇒ λAx = λx ⇒ λ2 x = λx.

Therefore
λ(λ − 1)x = 0,
which will only have non-trivial solutions for x if λ = 0 or λ = 1.

52
A is idempotent since
 2  
1 0 0 1 0 0
A2 =  0 3 6  = 0 3 6  = A.
0 −1 −2 0 −1 −2

The matrices A and A2 have the same eigenvalues 0, 1, 1. The corresponding eigenvectors of both
A and A2 are      
0 0 1
 −2  ,  −3  ,  0 .
1 1 0

13.9. Check that  2


1 1 1 1
1 1 1 −1 −1 
A = 
2  = I4 .
4 1 −1 1 −1 
1 −1 −1 1
Multiply Ax = λx on the left by A:

A2 x = λAx = λ2 x.

Since A2 = I4 , it follows that


I4 x = λ2 x, or (λ2 − 1)x = 0.
Non-trivial solutions for x only exist if (λ2 − 1) = 0, or λ = ±1. The eigenvectors are
       
−1 1 1 1
 1   0   0   1 
s1 =      
 1  , s2 =  0  , s3 =  1  , s4 =  0  .
 

1 1 0 0

Since A has 4 independent eigenvectors it can be diagonalized.


13.10. The eigenvalues of  
1 2 1
A= 2 1 1 
1 1 2
are λ1 = −1, λ = 1, λ3 = 4. Check that

trace A = 1 + 1 + 2 = 4 = λ1 + λ2 + λ3 ,

and that
det A = −4 = λ1 λ2 λ3 .

13.11. The determinant  


£ ¤ 1 2 4
s1 s2 s3 = 2 −1 3 
1 3 5
is zero, which means that the vectors are linearly dependent.
13.12. Use the method explained in Section 13.4. The eigenvalues and eigenvectors of
 
−4 1 −2
A =  2 −2 1 
0 1 0

53
are 
   

7 −1 −1
λ1 = −5, λ2 = −1, λ3 = 0, s1 =  −5  , s2  −1  , s3 =  0  .
1 1 2
Form the matrix  
£ ¤ 7 −1 −1
C= s1 s2 s3 =  −5 −1 0 .
1 1 2
Next find the inverse of C:  
2 −1 1
1 
C −1 = −10 −15 −5  .
20
4 8 12
Now evaluate the product C −1 AC:
   
2 −1 1 −4 1 −2 7 −1 −1
1  −10
C −1 AC = −15 −5   2 −2 1   −5 −1 0 
20
4 8 12 0 1 0 1 1 2
 
−5 0 0
=  0 −1 0 ,
0 0 0

which is a diagonal matrix of eigenvalues of A.


13.13. The eigenvalues and eigenvectors of
· ¸
1 8
A=
2 1
are · ¸ · ¸
−2 2
λ1 = −3, λ2 = 5, s1 = , s2 = .
1 1
Let · ¸
£ ¤ −2 2
C= s1 s2 = .
1 1
Then · ¸
−1 − 41 1
2
C = 1 1 .
4 2

Confirm that · ¸· ¸· ¸ · ¸
−1 − 14 1
2 1 8 −2 2 −3 0
C AC = 1 1 = ,
4 2 2 1 1 1 0 5
a diagonal matrix of the eigenvalues.
13.14. The eigenvalues of A are given by
 
2−λ 0 0
 0 2−λ 2  = −λ3 + 3λ2 + 4λ − 12 = −(λ + 2)(λ − 2)(λ − 3) = 0.
0 2 −1 − λ

Hence the eigenvalues are


λ1 = −2, λ2 = 2, λ3 = 3.
The corresponding eigenvectors are
    
0 1 0
s1 =  −1  , s2 =  0  , s3 =  2  .
2 0 1

54
Let  
£ ¤ 0 1 0
C= s1 s2 s3 =  −1 0 2 .
2 0 1
Its inverse is  
0 −1 2
1
C −1 =  5 0 0 .
5
0 2 1
Then
   
0 −1 2 2 0 0 0 1 0
1 
C −1 AC = 5
5 0 0  0 2 2   −1 0 2 
0 2 1 0 2 −1 2 0 1
 
−2 0 0
=  0 2 0  = D.
0 0 3

13.15. Use results outlined in Problem 8.15. Take determinants of both sides of C −1 AC = D:

det[C −1 AC] = det D, or det C −1 det A det C = det D,

or (1/ det C) det A det C = det D.


Hence det A = det D: the result follows, since

det D = λ1 λ2 . . . λn .

13.16. The eigenvalues and eigenvectors of


 1 1 1

4 2 4
A= 1
2
1
4
1
4

1 1 1
4 4 2

are      
−1 − 12 1
1 1
λ1 = − , λ2 = , λ3 = 1, s1 =  1  , s2 =  − 12  , s3 =  1  .
4 4
0 1 1
Let    1 
1
£ ¤ −1 − 12 1 −2 2 0
C= s1 s2 s3 = 1 − 12 1  , so that C −1 =  − 13 − 13 2
3
.
1 1 1
0 1 1 3 3 3
Let D be the diagonal matrix of eigenvalues:
 
− 14 0 0
D=  0 1
0 .
4
0 0 1

Then, by Section 13.5,

An = CD

n −1
C   1 
1
−1 − 12 1 (− 41 )n 0 0 −2 2 0
=  1 −1 1   0 ( 14 )n 0   − 13 − 31 23 
2
n 1 1 1
0 1 1 0 0 1 3 3 3
 1 1 n −2n−1 1 1 n −2n−1 1 −n

3 + 3 [1 + 3(−1) ]2 3 + 3 [1 − 3(−1) ]2 3 (1 − 4 )
=  1 + 1 [1 − 3(−1)n ]2−2n−1 1 + 1 [1 + 3(−1)n ]2−2n−1 1 (1 − 4−n )  .
3 3 3 3 3
1 −n 1 −n 1 2 −n
3 (1 − 4 ) 3 (1 − 4 ) 3 + 34

55
Therefore  
1 1 1
An → 13  1 1 1 ,
1 1 1
as n → ∞, since 2−2n−1 → 0 and 4−n → 0.
13.17. A matrix A is orthogonal if AT = A−1 . In this example
 −1  
1 0 0 1 0 0
A−1 =  0 cos α sin α  =  0 cos α sin α  = AT ,
0 sin α cos α 0 sin α cos α

and so A is orthogonal. Expanding X = Ax:

X = x. Y = y cos α, Z = z sin α,

or, inverting,
x = X, y = Y cos α, z = −Y sin α + Z cos α.
The x and X axes remain coincident and the OXY Z coordinate frame is rotated about the x axis
through an angle α relative to the Oxyz frame. The points on the x axis are unaffected by the
rotation.
13.18. The matrix is orthogonal since
 −1  
1 −1 1 −1 1 1 1 1
1  1 −1 −1 1  1  −1 −1 1 1 
A−1 =   =   = AT .
2 1 1 −1 −1  2 1 −1 −1 1 
1 1 1 1 −1 1 −1 1

13.19. The matrix · ¸


cos α − sin α
sin α cos α
is orthogonal, which implies that the X and Y axes are orthogonal. Expanding

X = x cos α − y sin α, Y = x sin α + y cos α,

or, inverting,
x = X cos α + Y sin α, y = −X sin α + Y cos α.
A sketch of the axes drawn from these relations shows that α is the angle between the x axis
and the X axis. The x axis becomes the line Y cos α = X sin α, and the y axis becomes the line
Y sin α = −X cos α.
13.20. The eigenvalues are given by
 
−λ a b
 −a −λ c  = −λ(λ2 + a2 + b2 + c2 ).
−b −c λ

Hence the eigenvalues are


√ √
λ1 = 0, λ2 = − [−a2 − b2 − c2 ], λ3 = [−a2 − b2 − c2 ].

The two eigenvalues λ2 and λ3 are imaginary assuming that a, b and c are not all zero.
13.21. Given  
1 2 1
A= 2 1 1 ,
1 1 2

56
then
¯ ¯
¯ 1−λ 2 1 ¯
¯ ¯
det(A − λI3 ) = ¯ 2 1 − λ 1 ¯
¯ ¯
¯ 1 1 2−λ ¯
= (1 − λ)[(1 − λ)(2 − λ) − 1] − 2[2(2 − λ) − 1] + [2 − (1 − λ)]
= −λ3 + 4λ2 + λ − 4.

The required powers of A are


   
6 5 5 21 22 21
A2 =  5 6 5 , A3 = AA2 =  22 21 21  .
5 5 6 21 21 22
Hence
   
21 22 21 6 5 5
−A3 + 4A2 + A − 4I3 = −  22 21 21  + 4  5 6 5 
21 21 22 5 5 6
   
1 2 1 1 0 0
+ 2 1 1  − 4 0 1 0 
1 1 2 0 0 1
= 0

Mutiply both sides of this equation by A−1 :

−A−1 A3 + 4A1 A2 + A−1 A − 4A−1 = −A2 + 4A + I3 + A1 = 0.

Therefore  
−1 3 −1
1 1
A−1 = [−A2 + 4A + I3 ] =  3 −1 −1  .
4 4
−1 −1 3

13.22. The eigenvalues and eigenvectors of


 
5 −1 −3 3
 −1 5 3 −3 
A=
 −3

3 5 −1 
3 −3 −1 5
are
λ1 = 0, λ2 = λ3 = 4, λ4 = 12,
       
−1 0 1 1
 1   0   1   −1 
s1 =  
 −1  , s2 =   
 1  , s3 =  0
,
 s4 =  
 −1  .
1 1 0 1
The matrix has 4 independent eigenvectors even though there is a repeated eigenvalue. Let
   
−1 0 1 1 −1 1 −1 1
 1 0 1 −1  1 2 2 
C=  −1  0 0 .
 −1 1 0 −1  , and C = 4  2 2 0 0 
1 1 0 1 1 −1 −1 1

We can then check that  


0 0 0 0
 0 4 0 0 
C −1 AC = 
 0
.
0 4 0 
0 0 0 12

57
Since A has a zero eigenvalue, det A = 0.
13.23. Let x = [x1 x2 x3 ]T .
(a) In matrix form
 
1 2 −2
x21 + x22 + x33 + 4x1 x2 − 4x1 x3 + 4x2 x3 = xT  2 1 2  x.
−2 2 1

The eigenvalues and eigenvectors of A are


     
1 −1 1
λ1 = −3, λ2 = λ3 = 3, s1 =  −1  , s2 =  0  , s3 =  1  .
1 1 0

The required matrix  


£ ¤ 1 −1 1
C= s1 s2 s3 =  −1 0 1 .
1 1 0

(b) In matrix form  


1
0 2 − 12
x1 x2 − x1 x3 + x2 x3 = xT  1
2 0 1
2
 x.
− 12 1
2 0
The eigenvalues and eigenvectors of A are
    
1 −1 1
1
λ1 = −1, λ 2 = λ3 = , s1 =  −1  , s2 =  0  s3 =  1  .
2
1 1 0

The required matrix is  


£ ¤ 1 −1 1
C= s1 s2 s3 =  −1 0 1 .
1 1 0

13.24. Express each quadratic form as xT Ax and find the eigenvalues of A. Then the quadratic
form is positive-definite if, and only if, all its eigenvalues are positive (Section 13.7).
(a) In matrix form · ¸
2 2 T 4 −2
4x1 + x2 − 4x1 x2 = x x.
−2 1
Hence · ¸
4 −2
A= ,
−2 1
and its eigenvalues are 0 and 5. Therefore the quadratic form is not positive-definite.
(b) In matrix form
 
1 2 1
x21 + x22 + 2x23 + x2 x3 + 2x3 x1 + 4x1 x2 = xT  2 1 1  x.
1 1 2

Hence  
1 2 1
A= 2 1 1 ,
1 1 2
and its eigenvalues are −1, 1, 4. Since it has a negative eigenvalue, the quadratic form is not
positive-definite.

58
(c) In matrix form  
6 0 − 12
6x21 + 2x22 − x3 x1 = xT  0 2 0  x.
− 12 0 0
√ √
Its eigenvalues are 2 and 12 [6 ± 37]. One eigenvalue is negative so that the quadratic form is
not positive-definite.
13.25. The equations of motion can be obtained as in Section 13.8 but with three particles. As in
a generalization of Fig. 13.2, the first spring has extension x and tension −kx, the second extension
y − x and tension k(y − x), the third extension z − y and tension k(z − y) and the fourth extension
−z and tension −kz. Applying Newton’s law to each particle:

mẍ = −kx + k(y − x) = k(−2x + y),

mÿ = −k(y − x) + k(z − y) = k(x − 2y + z),


mz̈ = −k(z − y) − kz = k(y − 2z).
Let    
x 2 −1 0
k 
x =  y , A= −1 2 −1  .
m
z 0 −1 2
Then the equations of motion can be expressed in the matrix form

ẍ + Ax = 0.

The eigenvalues and eigenvectors of A are


√ √
λ1 = 2k/m, λ2 = [2 − 2]k/m, λ2 = [2 + 2]k/m.
     
−1 √ 1 √ 1
s1 =  0  , s2 =  2  , s3 =  − 2  .
1 1 1
Let  
£ ¤ −1 √1 √1
C= s1 s2 s3 = 0 2 − 2 .
1 1 1
Introduce the normal coordinates X = [X Y Z]T where
 
−X
√ + Y√+ Z
x = CX =  2Y − 2Z  .
X +Y +Z

The equations of motion become

C Ẍ + ACX = 0 or Ẍ + DX = 0,

where  
2 0√ 0
k 
C −1 AC = D = 0 2− 2 0√  .
m
0 0 2+ 2

13.26. Calculating A2 and A3 :


 2  
0 1 0 0 0 1
A2 =  0 0 1  = 1 0 0 , A3 = AA2 = I3 .
1 0 0 0 1 0

59
Therefore
A4 = A3 A = A, A5 = A3 A2 = A2 ,
and, in general,

A3m = I3 , A3m+1 = A, A3m+2 = A2 , (m = 1, 2, 3, . . .).

The eigenvalues and eigenvectors of A are


1 √ 1 √
λ1 = 1, λ2 =(−1 − i 3), λ3 = (−1 + i 3),
2 2
   1 √   1 √ 
1 − 2 (1 + i√3) − 2 (1 − i√3)
s1 =  1  , s2 =  − 12 (1 − i 3)  , s3 =  − 12 (1 + i 3)  .
1 1 1
Let  √ √ 
£ ¤ 1 − 12 (1 + i√3) − 12 (1 − i√3)
C= s1 s2 s3 = 1 − 12 (1 − i 3) − 12 (1 + i 3) 
1 1 1
Its inverse is given by  
2 √ 2 √ 2
1
C −1 = −1 + i√3 −1 − i√ 3 2  .
6
−1 − i 3 −1 + 3 2
By (13.5)

An = CDn C −1
 √ √  n
1 − 12 (1 + i√3) − 12 (1 − i√3) 1 0 0
1
=  1 − 1 (1 − i 3) − 1 (1 + i 3)  0 −(−1) 3 n 0 
2 2 2
1 1 1 0 0 −(−1) 3 n
 
2 √ 2 √ 2
1
× −1 + i√3 −1 − i√ 3 2 
6
−1 − i 3 −1 + 3 2

 I3 n = 3m
= A n = 3m + 1
 2
A n = 3m + 2

where m = 1, 2, 3, . . ..
13.27. Multiply S on the right by A:

S = A + A2 + · · · + An
,
SA = A2 + · · · + An + An−1

and take the difference S − SA, so that

S − SA = A − An−1 , or S(I3 − A) = A(I3 − An ).

Therefore
S = A(I3 − An )(I3 − A)−1 .
The method fails if I3 − A is a singular matrix, which is equivalent to A having a unit eigenvalue.
The eigenvalues and eigenvectors of
· ¸
1 3
A=
2 2

60
are · ¸ · ¸
−3 1
λ1 = −1, λ2 = 4, s1 = , s2 = .
2 1
Then · ¸
n n −1 1 3(−1)n + 22n+1 −3(−1)n + 3 × 4n
A = CD C =
5 −2(−1)n + 22n+1 2(−1)n + 3 × 4n
Finally · ¸
1
− 12
[I2 − A]−1 = 6
− 13 0
so that

S = A(I2 − An )(I2 − A)−1


· ¸· ¸· 1 ¸
1 1 3 5 − 3(−1)n − 22n+1 3(−1)n − 3 × 4n 6 − 12
=
5 2 2 2(−1)n − 22n+1 5 − 2(−1)n − 3 × 4n − 13 0
· ¸
1 5 − 3(−1)n − 22n+1 −27[(−1)n − 4n ]
= .
30 −8[(−1)n − 4n ] 0

13.28. As in Example 13.5, the eigenvalues of


 
1 2 1
A= 2 1 1 
1 1 2
are
λ1 = −4, λ2 = 1, λ3 = −1.
We now choose different eigenvectors (compare Example 13.5):
     
2 −1 −3
s1 =  2  , s2 =  −1  , s3 =  3  .
2 2 0

Let  
£ ¤ 2 −1 −3
C= s1 s2 s3 =  2 −1 3 .
2 2 0
Its inverse is  
1 1 1
6 6 6
C −1 =  −6 1
− 16 1
3
.
− 16 1
6 0
We can now check that
 1 1 1
    
6 6 6 1 2 1 2 −1 −3 4 0 0
C −1 AC =  − 16 − 16 1
3
 2 1 1  2 −1 3 = 0 1 0 .
− 16 1
6 0 1 1 2 2 2 0 0 0 −1

The problem illustrates the point that the choice of eigenvectors and consequently the choice of C
does not affect the matrix C −1 AC: it always equals D.
13.29. The remaining 3 × 3 minors of
 
1 2 3 4
A =  10 11 12 5 ,
9 8 7 6

61
are ¯ ¯ ¯ ¯ ¯ ¯
¯ 1 2 3 ¯ ¯ 1 3 4 ¯ ¯ 2 3 4 ¯
¯ ¯ ¯ ¯ ¯ ¯
¯ 10 11 12 ¯ = 0, ¯ 10 12 5 ¯ = −160, ¯ 11 12 5 ¯ = −80.
¯ ¯ ¯ ¯ ¯ ¯
¯ 9 8 7 ¯ ¯ 9 7 6 ¯ ¯ 8 7 6 ¯
The matrix has 18 (2 × 2) minors. Their values are:
with elements from rows 1 and 2: −9, −18, −35, −9, −34, −33;
with elements from rows 1 and 3: −10, −20, −30, −10, −20, −10;
with elements from rows 2 and 3: −19, −38, 15, −19, 26, 37.
13.30. (a) Let  
1 2 3
A= 3 4 5 .
6 7 8
Then det A = 0 which means that rank A < 3. However, the 2 × 2 minor
¯ ¯
¯ 1 2 ¯
¯ ¯ = −2 6= 0.
¯ 3 4 ¯

Therefore the rank of A is 2.


(b) Let  
3 2 1
A= 1 2 3 .
2 1 3
In this case det A = 12, which means that the matrix is non-singular and has rank 3.
(c) The matrix  
1 2 3 2
A= 1 3 4 5 
2 3 5 1
is of rank 2.
13.31. Apply row operations to the matrix in Problem 13.30c:
   
1 2 3 2 1 2 3 2
 1 3 4 5  →  0 (r02 = r2 − r1 )
1 1 3 
(r03 = r3 − 2r1 )
2 3 5 1 0 −1 −1 −3
 
1 2 3 2
→  0 1 1 3  (r03 = r3 + r2 )
0 0 0 0

In echelon form the matrix has one row of zeros which means that its rank is 2.
13.32. The eigenvalues of  
1 2 −1
A1 =  1 2 −1 
2 2 −1
are λ1 = 0 and the repeated eigenvalue λ2 = 1. The rank of λ1 I3 − A1 is 2, and the rank of
λ2 I3 − A1 is also 2.
The eigenvalues of  
3 0 −1
A2 =  0 1 0 
2 0 0
are λ3 = 2 and the repeated eigenvalue λ4 = 1. The rank of λ3 I3 − A2 is 2, but the rank of
λ4 I3 − A2 is 1. The vector space associated with this eigenvalue is 2, which means that we can
define two independent eigenvalues.

62
The eigenvalues of  
2 1 0 0
 0 −1 0 2 
A=
 0

0 1 0 
0 0 2 1
are λ1 = −1, λ2 = 1 (repeated), λ3 = 2. The rank of A is 4 (since det A 6= 0). The ranks of
λ1 I4 − A and λ3 I4 − A are both 3. The rank of λ2 I2 − A is 3. Hence in this problem we can find
only three independent eigenvectors, which can be expressed as
     
−1 −1 1
 3   1   0 
s1 =    
 0  , s2 =  0  , s3 =  0  .
 

0 0 0

63
PART I: Elementary methods, differentiation, complex numbers
Chapter 1: Standard functions and techniques, 1
Chapter 2: Differentiation, 15
Chapter 3: Further techniques for differentiation, 23
Chapter 4: Applications of differentiation, 32
Chapter 5: Taylor series an approximations, 46
Chapter 6: Complex numbers, 58

Chapter 1: Standard functions and techniques

1.1. (a) y = x4 for −1.5 ≤ x ≤ 1:


y
5
4

1
x
-1.5 -1 -0.5 0.5 1

Figure 1: Problem 1.1a

(b) y = x(1 − x) for −1 ≤ x ≤ 2:


y

x
-1 -0.5 0.5 1 1.5 2
-0.5

-1

-1.5

-2

Figure 2: Problem 1.1b

(c) y = 1 + x + x2 for |x − 1| ≤ 2:
y
12
10
8
6
4
2
x
-1 1 2 3

Figure 3: Problem 1.1c

(d) y = |x − 1| for −3 ≤ x ≤ 3:
y
4

x
-3 -2 -1 1 2 3

Figure 4: Problem 1.1d

1
y
10

x
-3 -2 -1 1 2 3

Figure 5: Problem 1.1e


y
1
0.8
0.6
0.4
0.2
x
-2 -1 1 2

Figure 6: Problem 1.1f

(e) y = |x| + |x − 3| + |x + 2| for −3 ≤ x ≤ 4:


(f) y = ||x|
√ − 1| for −2 ≤ x ≤ 2:
(g) y = (x2 + 1) for |x| ≤ 2:
y
2
1.5
1
0.5
x
-2 -1 1 2

Figure 7: Problem 1.1g

1.2. (a) y = −2x + 3; (b) y = 1; (c) y =√32 x − 13 . The√intersections occur at A : (2, 1), B : ( 54 , 12 ),
C : (1, 1). The side lengths are: AB = 41 13, BC = 41 5, CA = 1.
y
H2,1L
1 H1,1L

0.8

0.6

0.4 H54,12L

0.2

x
0.5 1 1.5 2

Figure 8: Problem 1.2

1.3. (a) Slope is 1, and the line cuts the axes at (0, −1) and (1, 0).
(b) Slope is 13 , and the line cuts the axes at (0, − 23 ) and (2, 0).
(c) Slope is − 25 , and the line cuts the axes at (0, − 45 ) and (2, 0).
1.4. (a) y = x + 1; (b) y = −2x − 4; (c) y = 0.5x − 0.5; (d) y = 3x − 1; (e) the slope of the line
must be − 14 : y = − 14 x + 11
4 .
1.5. The products of the slopes in each case must be −1. The slopes are: (a) − 32 and 23 ; (b) 2 and
− 21 ; (c) 2 and − 12 ; (d) 1 and −1.
1.6. At the point of intersection, x + y + 1 = 0 and 2x − 3y − 2 = 0, so the line
(x + y + 1) + α(2x − 3y − 2) = 0

2
must pass through this point, which has coordinates (− 51 , − 45 ). The straight line joining this point
to (1, 1) is
2y = 3x − 1,
with α = 12 .

1.7. (a) Centre at (0, 0), radius √ 3; (b) centre at (1, 0), radius 2; (c) centre at (1, 1), radius 23;
(d) centre at ( 21 , − 12 ), radius 12 11.
1.8. (x − 1)2 + (y + 2)2 = 9.
1.9. Eliminate one of the variables in each case and solve the resulting quadratic equation.
(a) (2, 2) and (2, −2);
(b) Eliminate y, so that x satisfies the equation

x2 + (2x + 1)2 − 2x + 2(2x + 1) − 4 = 0, or 5x2 + 6x − 1 = 0.

The points of intersection are


√ √ √ √
( 15 (−3 − 14, −1 − 2 14) and ( 51 (−3 + 14, −1 + 2 14)
√ √
(c) ( 12 2, 12 2), one point only since the line is tangential to the circle.
1.10. To three decimal places, the distances of the points from the origin are

1.060, 0.993, 1.011, 0.896, 1.124.

The average value of these distances is r = 1.017. The equation of the circle is

x2 + y 2 = r2 = 1.034.

1.11. (a) x = H(t + 1) − H(t − 1).

x
1
0.8
0.6
0.4
0.2
t
-2 -1 1 2

Figure 9: Problem 1.11a

(b) x = sgn(1 + t) + sgn(1 − t).

x
2
1.5
1
0.5
t
-2 -1 1 2

Figure 10: Problem 1.11b

(c) x = tH(t − 1).


(d) (t2 − 1)[sgn(t + 1) + sgn(1 − t)].
1.12. (a) f (t) = H(2 − t) + H(t + 1) − 1; (b) f (t) = 2tH(t);
(d) f (t) = (3 − t)H(3 − t) + (t − 2)H(2 − t) + (t − 1)H[1 − t) − tH(−t).

3
x
2

1.5

0.5

t
0.5 1 1.5 2

Figure 11: Problem 1.11c


x
t
-2 -1 1 2
-0.5

-1

-1.5

-2

Figure 12: Problem 1.11d

1.13. (a) 16 π radians; (b) 23 π radians.


√ √ √ √ √
1.14. √(a) 1/ 2; (b) 1; (c) 0; (d) −1/ 2; (e) 3/2; (f) − 3/2; (g) − 3/2;
(h) − 3/2.
1.15. (a) Using the identity cos2 B = 12 (1 + cos 2B)

1 1
cos4 A = (1 + cos 2A)2 = (1 + 2 cos 2A + cos2 2A)
4 4
1 1 1
= (1 + 2 cos 2A + (1 + cos 4A)) = (3 + 4 cos 2A + cos 4A).
4 2 8
(b) Use the identities sin2 B = 12 (1 − cos 2B) and cos2 2B = 12 (1 + cos 4B):

1 3 1 1
sin4 A = (1 − 2 cos 2A + cos2 2A) = − cos 2A + cos 4A.
4 8 2 8

1.16. (a) cos(x + 12 π) = cos x cos 12 π − sin x sin 21 π = sin x; (b) cos x; (c) − cos x; (d) − cos x (for
both); (e) sin x (for both).
1.17. (a)
1 1 1 1
cos x + cos y = cos[ (x + y) + (x − y)] + cos[ (x + y) − (x − y)]
2 2 2 2
1 1 1 1
= cos[ (x + y)] cos[ (x − y)] − sin[ (x + y)] sin[ (x − y)] +
2 2 2 2
1 1 1 1
cos[ (x + y)] cos[ (x − y)] + sin[ (x + y)] sin[ (x − y)]
2 2 2 2
1 1
= 2 cos[ (x + y)] cos[ (x − y)]
2 2
(b) sin x − sin y = 2 sin[ 21 (x − y)] cos[ 12 (x + y)].
(c) cos x − cos y = −2 sin[ 21 (x + y)] sin[ 12 (x − y)].
1.18. (a) x = nπ, (n = 0, ±1, ±, 2, . . .);
(b) x = 12 (2n + 1)π, (n = 0, ±1, ±, 2, . . .);
(c) x = 2nπ, (n = 0, ±1, ±, 2, . . .);
(d) x = 16 (2n + 1), (n = 0, ±1, ±, 2, . . .);
(e) x = 12 nπ, (n = 0, ±1, ±, 2, . . .);
(f) x = 2n, (n = 0, ±1, ±, 2, . . .).

4
1.19.
amplitude angular frequency period phase
(a) 2 0.2 10π 3.2
(b) 1.5 0.2 10π −0.48
(c) 3.87 0.2 10π −0.135
(d) 1 1 2π π

1.20. (a) F (x) = 2 −x; (b) F (x) = 2 (x − 3); (c) F (x) = 21 arcsin x;
1 1
1
(d) F (x) = arcsin(£12 x); (e) F¤(x) = [arccos x] 2 ;
(f) F (x) = arccos π2 arcsin x ; √
(g) F (x) = x−4 ; (h) F (x) = − 12 + (x + 14 ).
1.21. The graph shows y = x3 − x + 1 (the dashed curve) and its inverse.
y
2

1.5

0.5

x
-2 -1.5 -1 -0.5 0.5 1 1.5 2

-0.5

-1

-1.5

-2

Figure 13: Problem 1.21

1.22. (a) x = 12 ln 3; (b) x = 13 e2 ; (c) x = e−3 ; (d) x = − 31 ln 3;


(e) the equation is the same as (ex√− 1)2 = 0: hence √ x = 0;
(f) x = 2; (g) x = 2/17; (h) x = ± 2; (i) x = ± (1 + ee ); √
(j) x = (ln 3)/(ln 2); (k) x = −(ln 2)/(2 ln 3); (l) x = 12 ln[4 + 17];
(m) no solutions.
1.23. 2x = ex ln 2 .
1.24. Consider two values of x, say x1 and x2 , where x1 > x2 . Then if 10x1 = 2 × 10x2 , it follows
that
ln 2
10x1 −x2 = 2, or x1 − x2 = ,
ln 10
an interval which is independent of x1 and x2 .
1.25. (a) (x − 1)2 + y 2 ≤ 9.
y
3

x
-2 -1 1 2 3 4

-1

-2

-3

Figure 14: Problem 1.25a

(b) x ≥ 0, y ≥ 0, and x + y ≤ 1.
(c) (x2 /4) + (y 2 /9) ≤ 1.
(d) x2 + y 2 ≤ 1 and x ≥ 0.
(e) |x| + |y| ≤ 1.
1.26. Let
sinh y ey − e−y
x = tanh y = = y ,
cosh y e + e−y

5
y
1

0.8

0.6

0.4

0.2

x
0.2 0.4 0.6 0.8 1

Figure 15: Problem 1.25b


y
3

x
-2 -1 1 2

-1

-2

-3

Figure 16: Problem 1.25c

where −1 < x < 1. Hence


· ¸
2y 1 1+x
(1 − x)e = 1 + x so that y = ln
2 1−x
as required.

1.27. From triangle ABC



AC = AB sin θ + (BC 2 − AB 2 cos2 θ)

= 2.5[sin ωt + (4 − cos2 ωt)] cm,
where θ = ωt. The angular frequency ω = 400π/3.

1.28. x = 5 cos(ωt − 0.927). The amplitude c = 5 and the phase angle is φ = −0.927.

1.29. f (0) = 2 implies C = 2 and f (1) = 0.5 implies 0.5 = Ce−α = 2e−α . Hence α = ln 4. Also
f (2) = 18 .

1.30. The tidal period is 2π/0.5 = 12.57 hours. We require the times when the depth is 2m in one
period, which are given by solutions of
2 = 5 + 4.5 sin 0.5t so that sin 0.5t = − 23 .

y
1

0.5

0.20.40.60.8 1 x

-0.5

-1

Figure 17: Problem 1.25d

6
y
1

0.5

x
-1 -0.5 0.5 1

-0.5

-1

Figure 18: Problem 1.25e

Two consecutive times are 11.11 hours and 7.74 hours. Hence the yacht can float free for 9.20
hours in each tidal period. The yacht floats when sin 0.5t > − 23 . It is helpful to sketch y = sin 0.5t
and y = − 32 and plot their intersections.
1.31. (a) The cardioid r = 0.5(1 + cos θ):

0.5

0.5 1 1.5 2

-0.5

-1

Figure 19: Problem 1.31a

(b) The folium r = (4 sin2 θ − 1) cos θ

0.75

0.5

0.25

-1 -0.75 -0.5 -0.25 0.25 0.5

-0.25

-0.5

-0.75

Figure 20: Problem 1.31b

(c) r = sin 2θ:


0.75

0.5

0.25

-0.75 -0.5 -0.25 0.25 0.5 0.75

-0.25

-0.5

-0.75

Figure 21: Problem 1.31c

(d) The Archimedean spiral r = 0.04θ:


]
(e) The equiangular spiral r = 0.1e0.1θ :
1.32. (a) sgn (sin x):
(b) sgn cos 2x:

7
0.4

0.2

-0.6 -0.4 -0.2 0.2 0.4 0.6

-0.2

-0.4

-0.6

Figure 22: Problem 1.31d


0.4

0.2

-0.4 -0.2 0.2 0.4 0.6

-0.2

-0.4

Figure 23: Problem 1.31e

(c) H(x) sin x:


(d) sin2 x:
(e) | sin x|:
(f) sin |x|:
(g) H(x − π) sin x:
1.33. Let the points be A : (−7, 3), B : (1, −3) and C : (4, 1). The slope of AB is − 34 and the
d is a right angle. Let D
slope of BC is 43 ; the product of the slopes is −1 which means that ABC
be the fourth vertex. Then the equations of the lines AD and DC are

y − 3 = 34 (x + 7) and y − 1 = − 34 (x − 4),

or
3y − 4x − 37 = 0 and 4y + 3x − 16 = 0.
These lines intersect at the point D : (−4, 7)
There is a general formula buried here, if you notice that the coordinates of D are (−7 + 4 −
1, 3 + 1 − (−3)).
1.34. (a) periodic, period 12 π; (b) periodic, period 2π; (c) periodic, period 2π; (d) not periodic;
(e) periodic, period 2π; (f) periodic, period π; (g) not periodic; (h) periodic, period π; (i) periodic,
period π; (j) periodic, period 8π; (k) periodic, period 23 , since sin 3t has period 32 π and cos 9t has
period 29 π but has the period of sin 3t; (l) not periodic.
1.35. (a) neither odd nor even; (b) even; (c) odd since sin x is odd; (d) odd since product of odd
and even functions; (e) even; (f) even; (g) neither odd nor even.
1 1 1 2 1 1
1.36. (a) 5(x−2) − 5(x+3) ; (b) − x+1 + x+2 ; (c) x + x−1 ;

y
1

0.5

x
-6 -4 -2 2 4 6
-0.5

-1

Figure 24: Problem 1.32a

8
y
1

0.5

x
-2 -1 1 2
-0.5

-1

Figure 25: Problem 1.32b


y
1

0.5

x
-2 2 4 6

-0.5

-1

Figure 26: Problem 1.32c

1 1 1 1
(d) 2x − x+1 + 2(x+2) ; (e) 2(x−1) − x1 + 2(x+1)
1
;
1 1 1 1 4
(f) 4x − 2(x+2)2 − 4(x+2) ; (g) x+1 = (x+2)2 ;
1 1
(h) 2(x−3) + 2(x+1) ; (i) x1 − x−1
1 1
+ (x−1)2;
1 1 1
(j) x2 − x + x+1 .
1 x+1 1 1−x 1 x+6
1.37. (a) x − x2 +x+1 ; (b) 2(x−1) + 2(x2 +1) ; (c) − 5(x+1) + 5(x2 +2x+6) .

1.38. (a) x12 − x21+1 ; (b) x − 3 − 1


x+1 + 8
x+2 ;
9 1 9
(c) 1 + 8(x−3) + 8(x+1) − 4(x+3) .
1 1 1 1
1.39. (a) 4 + 8 + 16 + 32; (b) 1 + 2 + 5 + 10 + 17 ;
(c) x + 2x2 + 3x3 + 4x4 .
1.40. For (a), (b), (c), (e) and (f) proceed as in Example 1.17.
(a) 2[1 − ( 12 )8 ] = 255 128 .
(b) 12 · 13 [1 − ( 13 )5 ] = 121
729 .
(c) (1 − e−12 )/(1 − e2 ).
(d) The sum is 642. More generally, let

x + 2x2 + · · · + nxn = T.

Then
T − xT = (1 − x)T = xS − nxn+1 ,
where S is the sum of the geometric series

1 + x + · · · + xn−1 .

y
1
0.8

0.6

0.4

0.2
x
-6 -4 -2 2 4 6

Figure 27: Problem 1.32d

9
y
1
0.8
0.6
0.4
0.2
x
-6 -4 -2 2 4 6

Figure 28: Problem 1.32e


y
1

0.5

x
-6 -4 -2 2 4 6
-0.5

-1

Figure 29: Problem 1.32f

For the given problem x = 2.


(e) − 12 32 [1 − (− 12 )10 = − 1024
341
.
7 7
(f) 2[ 1−(0.5)
0.5 ] + 3[( 1−(0.6)
0.4 ] = 11.258 . . ..
1.41. The series can be expressed as
10
X
x + x5 + x9 + · · · + x41 = x (x4 )n .
n=0

Using (1.33), the sum of the series is


1 − x44
x .
1 − x4
1.42. Let D be the foot of the perpendicular on to the side AC. Then

c2 = DB 2 + DA2 = DB 2 + (AC − DC)2 .

But DB = a sin C and DC = a cos C. Therefore

c2 = a2 sin2 C + (b − a cos C)2


= a2 sin2 C + a2 cos2 C + b2 − 2ab cos C
= a2 + b2 − 2ab cos C

1.43. The ratio of any pair of successive terms is

f (t0 + (n + 1)T ) Aec(t0 +(n+1)T )


= = ecT ,
f (t0 + nT ) Aec(t0 +nT )

y
1

0.5

x
2 4 6 8 10 12

-0.5

-1

Figure 30: Problem 1.32g

10
which is independent of n. The common ratio is ecT .
1 1 1
1.44. (a) 1.111 . . . = 1 + 10 + 100 + · · · is an infinite geometric series with common ratio 10 and
1
sum 1/(1 − 10 ) = 11/9. (b) The common ratio is 1/10, and as a fraction the sum is 1; (c) the
common ratio is 1/100, and as a fraction the sum is 1/99; (d) the common ration is 1/100, and
as a fraction the sum is 1/11; (e) the common ration is 1/10 and as a fraction the sum is 2/3; (f)
the notation means 2.7̇2̇ = 2.727272 . . .: the common ratio is 1/100 and the sum represents the
fraction 30/11.
1.45. The sum of the infinite geometric series is

X xm
xn = , |x| < 1.
n=m
1−x

(a) 2; (b) 10/9; (c) e/(e − 1); (d) 32 ; (e) 3/5.


1.46. (a) 24, 720, 5040; (b) 12; (c) 720; (d) 220; (e) 120; (f) 1, 3, 3, 1.
1.47. (a) (i) n(n − 1); (ii) (n + 1)n; (b) (i) 2m m!; (ii) (2m + 1)!/(2m m!).
¡ ¢
1.48. (a) (i) 120; (ii) 504; (iii) 120; (iv) 35; (v) 35; (vi) 252; (vii) 4950; (viii) 10
7 =10 C7 = 120.
n!
(b) n Pn = (n−n)! = n!0! = n! Also P
n n−1 = n!
(n−n+1)! = n!
1! = n!. Consider a collection of n
different letters. The number of different words of length n which can be made without repetition
is n Pn = n!. Suppose that the letters are A, B, C, . . . . Suppose that the first letter of an n
character word is A. Then the remaining n − 1 letters can be chosen in (n − 1)! ways. Repeat
the procedure for words with first letters B, C and so on. We obtain all words with n characters
again, and there are n(n − 1)! = n! of them.
1.49. (a) 4 P1 = 4!; (b) 4 C3 = 4; (c) 44 = 256; (d) 20; (e) 4 P4 +4 P3 +4 P2 +4 P1 = 4+12+24+24 = 64.
(f) Without repetitions the number of combinations is

4 C1 +4 C2 +4 C3 +4 C4 = 4 + 6 + 4 + 1 = 15.

With 2 letters the same there are 4 + 12 + 12 = 28 possibilities, and with 3 letters the same there
are 4 + 12 = 16. Hence the total number of combinations is 15 + 28 + 16 = 59
1.50. (a) With no E’s there are 4 P3 = 24 words, with 1 E there are 3 ×4 P2 = 36, and with 2 E’s
there are 3 ×4 P1 = 12. Hence there are 24 + 36 + 12 = 72 words.
(b) Label six letters A, B, C, D, E1 , E2 . Then the number of words treating E1 and E2 as distinct
is 6! = 720. The letters E1 and E2 can be interchanged in 2! = 2 ways. Hence the number of
six-letter words is 720/2 = 360.
1.51. (a) There are 5 P4 = 5!/1! = 120 distinct four-digit numbers.
(b) To be divisible by 5, the last digit must be 5. The preceding 3 digits can be chosen in 4 P3 = 24
ways. Hence there are 24 numbers divisible by 5.
(c) To be divisible by 2 the final digit must be 2 or 4. As in (b) the number of numbers is 24 P3 = 48.
(d) The numbers contain either 1, 2, 3 or 4 digits. There are 4 one-digit numbers (excluding
zero). For two-digit numbers we must exclude those starting with zero since they are the same
as the one-digit numbers. Hence there are 16 distinct two-digit numbers. Similarly there are
44 P2 = 48 three-digit numbers and 44 P3 = 96 four-digit numbers. Hence the total number is
4 + 16 + 48 + 96 = 164 words.
1.52. (a) Without restriction, the number of distinct combinations of personnel (no distinction
being made as to which particular post is assigned to each person) is 7 C4 = 7!/(4!3!) = 35.
(b) There is one selection with 4 females, 12 with 3 females and one male, 18 with 2 females and
2 males and 4 with one female and three males. (b) The posts can be filled in the following ways:
4 C4 = 1 with 4 females; 4 C3 3 C1 = 12 with 3 females and one male; 4 C2 3 C2 = 18 with 2 females
and 2 males; 4 C13 C3 = 4 with one female and 3 males. This confirms the 35 combinations of
personnel.

11
1.53. (a) We may model the problem by thinking of an ordered line of N pool balls, of various
colours (types) denoted by A, B, . . ., the number of each colour being NA , NB , . . . . The number
of possible orders (permutations) for the individual balls is N !, but we cannot distinguish visually
between balls having the same colour, so many of the N ! orders will look identical.
Suppose that the number of distinguishable arrangements is M . Each one of of these corresponds
to a possible NA !NB ! . . . permutations within the separate colours, so that
N!
N ! = M [NA !NB ! . . .], or M = .
NA !NB ! . . .
(b) We require the total number of different combinations, involving every number 1, 2, . . . , N of
balls. Consider any one of these: it contains 0 or 1 or 2. . . or NA (that is, (1 + NA ) possibilities)
of type A; 0 or 1 or 2. . . or NB of type B; and so on. The number of possible combinations is
therefore
(1 + NA )(1 + NB ) . . . − 1,
in which the term −1 is introduced to exclude the case of an all-zero ‘combination’.
1.54. (a) The national groups may be ordered (permuted) in 4! ways. By allowing for 5! permu-
tations possible within each group we obtain
5!5!5!5!4! = 5!4 4! = 2880
distinct line-ups.
(b) The number of distinct orderings of the 4 groups around a circular table is (4 − 1)! = 3! (see
Example 1.23). All possible permutations within the groups are then to be allowed for, so the total
number of arrangements is 5!3! = 720.

1.55. (a) (Prizes identical) The number of combinations of 3 distinct prizewinners out of 10
eligibles is 10 C3 = 120.
(b) (Prizes different) Call the Prizes P1 , P2 , P3 . P1 may go to any of 10 people; with each allocation
P2 may go to any of the remaining 9; then P3 to any of the remaining 8; all of these distributions
being distinct. The total number of possibilities is 10 × 9 × 8 = 720.
(c) (Prizes equal, distribution arbitrary) There are 3 types of distribution which can occur:
(i) One person gets all the prizes: there are 10 possibilities.
(ii) There are 10 persons who might get 2 prizes. With each of these there are 9 persons eligible
for the other prize. There are therefore 9 × 10 = 90 possibilities.
(iii) Three different people get prizes. Part (a) gives the number: there are 120 possibilities.
Therefore the total number of possibilities is
10 + 90 + 120 = 220
(d) P1 may go to any of the 10; similarly with P2 and P3 . Therefore the total number is 10×10×10 =
1000.

1.56. (a) The table shows the permissible numbers in the 3 categories. The number of combinations
possible within each category are given in brackets.

Accountants Lawyers Doctors Committees


- 1(2) 3(1) 2
- 2(1) 2(3 C2 ) 3
1(2) - 3(1) 2
1(2) 1(2) 2(3 C2 ) 12
1(2) 2(1) 1(3) 6
2(1) - 2(3 C2 ) 3
2(1) 1(2) 1(3) 6
2(1) 2(1) - 1
Check: 7 C4 = 35

12
Committees with exactly 1 accountant: 2 + 12 + 6 = 20.
Committees with exactly 1 doctor: 6 + 6 = 12.
(b) To locate the fallacy consider combinations of the 7 letters A, B, C, D, E, F, G, and take n = 4
and r = 3. Take, say, the r = 3-fold combination ABC and supplement it by, say, the unused letter
D, to form the combination ABCD. In the fallacious construction this will be counted several
times; for example, the same combination is counted again when it arises from supplementing
BCD by A.
The result is shown to be false by simply substituting the given numbers: only one contradiction
is sufficient to dispose of it.
1.57. (a) Refer back to (1.44).
(b) (1 − x)6 = 1 − 6x + 15x2 − 20x3 + 15x4 − 6x5 + x6 .
(c)

(x + x−1 )5 = x5 + 5x4 x−1 + 10x3 x−2 +


10x2 x−3 + 5xx−4 + x−5
= x5 + 5x3 + 10x + 10x−1 + 5x−3 + x−5

(x − x−1 )5 = x5 + 5x4 (−x)−1 + 10x3 (−x)−2 + 10x2 (−)x−3 +


5x(−x)−4 + (−x)−5
= x5 − 5x3 + 10x − 10x−1 + 5x−3 − x−5

1.58.

(1.01)10 = (1 + 0.01)10
= 1 + 10 × (0.01) + 45 × (0.01)2 + 120 × (0.01)3 + · · ·
= 1 + 0.1 + 0.0045 + 0.00012 + · · · = 1.105

to three decimal places.


Similarly

(0.99)8 = (1 − 0.01)8
= 1 − 8 × (0.01) + 28 × (0.01)2 − 56 × (0.01)3 + · · ·
= 1 − 0.08 + 0.0028 − 0.00056 + · · · = 0.923

to 3 decimal places.
1.59. Use the binomial theorem in the form

(1 + x)n = 1 +n C1 x +n C2 + · · · +n Cn xn .

(a) Put x = 2, so that


3n = 1 + 2n C1 + 22 n C2 + · · · + 2n n Cn xn .
For the second result put x = −1:

0 = 1 −n C1 +n C2 − · · · + (−1)n n Cn xn .

(b) Obtain two series with x = 1 and x = −1. Then add and subtract the series.
1.60. F (n, k) is defined for n = 0, 1, 2, . . ., and k = 0, 1, 2, . . . , n by

F (n, k) = n C0 + n+1 C1 + n+2 C2 + · · · + n+k Ck . (i)

A certain formula, namely


F (n, k) = n+k+1 Ck (ii)

13
is suggested for the sum in (i), and its truth for small values of k can be confirmed by calculation;
for example, from (i)
n!
F (n, 0) = n C0 = = 1 ≡ n+1 C0 , (iii)
0!n!
(n + 1)!
F (n, 1) = n C0 + n+1 C1 = 1 + = n + 2 ≡ n+2 C1 ;
1!n!
and so on.
To prove the truth of (ii) for all values of 0 ≤ k ≤ n, recast (i) into the form

F (n, k + 1) ≡ F (n, k) + n+k+1 Ck+1 , (iv)

a ‘recurrence formula’ enabling us to advance one step at a time in k, starting, for example, with
F (n, 0) and finding F (n, 1), F (n, 2), . . . , successively.
Now suppose we have verified the formula (ii) for any one particular value of k, say for k = K;
that is, we know somehow that

(n + K + 1)!
F (n, K) = n+K+1 CK = (v)
K!(n + 1)!

(for all n). Then from (iv)

F (n, K + 1) = F (n, K) + n+K+1 CK+1


= n+K+1 CK + n+K+1 CK+1 from (v)
(n + K + 1)! (n + K + 1)!
= +
K!(n + 1)! (K + 1)!n!
µ ¶
(n + K + 1)! 1 1
= +
K!n! n+1 K +1
(n + K + 2)!
= = n+K+2 CK+1
(K + 1)!(n + 1)!

We have proved that if (iv) is true for k = K, it is true for k = K + 1, where K may take any
value in 0 ≤ K < n.
But we verified in (iii) that (iv) holds good when k = K = 0. Therefore by (vi) it is true when
k = K + 1 = 1, so by (vi) again it is true when k = K + 2 = 2, and so on. It is therefore true for
all k.
1.61. Using partial fractions
1 1 1 1
= = − .
x2 + 3x + 2 (1 + x)(2 + x) 1+x 2+x

Write as
1 1 1
= (1 + x)−1 − (1 + x)−1 ,
x2 + 3x + 2 2 2
and expand both terms using (1.37) for infinite geometric series. Hence

1 1 x x2 x3
= (1 − x + x2 − x3 + · · ·) − (1 − + 2 − 3 + · · ·)
x2 + 3x + 2 2 2 2 2
1 1 1 1
= − (1 − )x + (1 − )x2 − (1 − )x3 + · · · .
2 2.2 2.22 2.23

1.62. V1 = A(1 + R), V2 = V1 (1 + R) = A(1 + R)2 , etc. In pounds: for 1000 @ 3% p.a.;

V5 = 1000(1 + 0.03)5 = 1159.27, V10 = 1343.92; V15 = 1557.97.

14
(b) Let the period start at an arbitrary time T0 . Then

VT0 +T (1 + R)t0 +T
= = (1 + R)T .
VT 0 (1 + R)T0

(c) Let T2 be the doubling period, so that from (b)

ln 2
(1 + R)T2 = 2 and T2 = .
ln(1 + R)

If R = 3%, T2 = 23.4 yr; if R = 6%, T2 = 11.9 yr; if R = 9%, T2 = 8.0 yr.


For the ten-times period, T10 = ln 10/ ln(1 + R). If R = 6%, then T10 + ln 10/ ln 1.06 = 39.6 yr.
1.63. If the income is withdrawn annually, it has been allowed to accrue to the fund through the
previous year with interest at the going rate R annually, or r monthly, the relation being

A(1 + R) = A(1 + r)12

where A is the value of the fund at the start of that year. By the binomial theorem,

(1 + r)12 = 1 + 12r + · · · · · · > 1 + 12r,

so R > 12r.
1.64. If the interest is payable monthly at the rate of rM per month, the interest on a fixed debt
D over any 12-month period is D(1 + rM )12 . This is equal to D(1 + R) where R is the annual
equivalent rate (AER). Therefore R = (1 + rM )12 − 1. If rM = 1%, R = 1.0112 − 1 = 0.126 (12.6%).
If rM = 3%, R = 0.425 (42.5%).
1.65. (a) After N complete years the initial payment A has drawn interest for N yrs, the second
payment for N − 1 yrs, and so on, and the (N − 1)th payment for 1 yr. The value VN of the fund
is then given by the geometric series

A(1 + R)N + A(1 + R)N −1 + · · · + A(1 + R)


= A(1 + R){1 + (1 + R) + · · · + (1 + R)N −1 }
= A(1 + R){(1 + R)N − 1}/R.

(b) N = 10, R = 5%. We obtain

100(1.05)(1.0510 − 1)
V10 = = £1320.68,
0.05
equivalent to a gain of 32% on the total investment of £1000.
(c) M investments of 2A, at 2-year intervals. Formula (a), with the fund value increasing by a
factor (1 + R)2 in each interval, becomes

(2A)(1 + R)2 ){[(1 + R)2 ]M − 1}


V2M = .
{(1 + R)2 − 1}

Using the data in (b) we obtain

200(1.05)2 (1.0510 − 1)
V10 = = £1352.88.
1.052 − 1

Chapter 2: Differentiation

2.1. Below are some sample values for three values of x on either side of the point where the
tangent is required. (The exact value of the slope is also given here.)

15
(a) y = x3 at (1, 1).
x 0.94 0.96 0.98 1.02 1.04 1.06
chord slope 2.82 2.88 2.94 3.06 3.12 3.18
The slope is 3.

(b) y = x at (1, 1).
x 0.85 0.90 0.95 1.025 1.10 1.15
chord slope 0.520 0.513 0.506 0.494 0.488 0.483
The slope is 0.5

(c) y = cos x at ( 14 π, 1/ 2).
x − 14 π −0.09 −0.06 −0.03 +0.03 0.06 0.09
chord slope 0.674 0.685 0.696 0.718 0.728 0.738

The slope is 1/ 2 = 0.707
(d) y = ex at (0, 1).
x −0.15 −0.10 −0.05 0.05 0.10 0.15
chord slope 0.929 0.952 0.975 1.025 1.052 1.079
The slope is 1.
(e) y = e2x at (0, 1).
x −0.15 −0.10 −0.05 0.05 0.10 0.15
chord slope 1.728 1.813 1.903 2.103 2.214 2.332
The slope is 2.
1
(f) y = x3 + x 2 at (1, 2).
x 0.94 0.96 0.98 1.02 1.04 1.06
chord slope 3.33 3.38 3.44 3.56 3.62 3.68
The slope is 3.5, the sum of the slopes in (a) and (b).
(g) y = ln x at (1, 0).
x 0.94 0.96 0.98 1.02 1.04 1.06
chord slope 1.031 1.0206 1.010 0.990 0.981 0.971
The slope is 1
2.2. (a) For y = 3x at (2, 6),
· ¸
dy δy 3(2 + δx) − 6
= lim = lim = 3.
dx δx→0 δx δx→0 δx

(b) For y = 3 − 2x at (1, 1),


· ¸
dy δy [3 − 2(1 + δx) − (3 − 2)
= lim = lim = −2.
dx δx→0 δx δx→0 δx

(c) For y = 3x2 at (1, 3),


· ¸
dy 3(1 + δx)2 − 3(1)2
= lim
dx δx→0 δx
= lim [6 + 3δx] = 6.
δx→0

(d) For y = x3 at (1, 1),


· ¸
dy (1 + δx)3 − 13
= lim
dx δx→0 δx
· ¸
3δx + 3(δx)2 + (δx)3
= lim
δx→0 δx
= lim [3 + 3δx + (δx)2 ] = 3
δx→0

16
(e) For y = 1/x at (2, 12 ),
· ¸
dy 1 1 1
= lim −
dx δx→0 δx 2 + δx 2
· ¸
−1 1
= lim =− .
δx→0 2(2 + δx) 4
(f) For y = 3x + 2x2 at (1, 5),
· ¸
dy 3(1 + δx) + 2(1 + δx)2 − 3 − 2
= lim δx → 0
dx δx
= lim [3 + 4 + 2δx] = 7.
δx→0

(g) For y = (1 + 2x) = 1 + 4x + 4x2 at (−1, 1),


2

dy 1
= lim [1 + 4(−1 + δx) + 4(−1 + δx)2 − 1 + 4 − 4]
dx δx→0 δx
= lim [4 + 4(−2 + (δx)2 ] = −4.
δx→0

2.3. (a) y = 3x2


dy 1
= lim [3(x + δx)2 − 3x2 ]
dx δx→0 δx
= lim [6x + 3δx]
δx→0
= 6x.
(b) y = x3 .
dy 1
= lim[(x + δx)3 − x3 ]
dx δx→0 δx
= lim [3x2 + 3xδx + (δx)2 ]
δx→0
= 3x2 .
(c) y = 1/x.

· ¸
dy 1 1 1
= lim −
dx δx→0 δx x + δx x
· ¸
−1
= lim
δx→0 x(x + δx)

= −1/x2 .
(d) y = x + 12 .

dy 1 1 1
= lim [(x + δx + ) − (x + )]
dx δx→0 δx 2 2
= 1.
(e) y = x + (1/x).
·µ ¶ µ ¶¸
dy 1 1 1
= lim x + δx + − x+
dx δx→0 δx x + δx x
· ¸
1
= lim 1 − 2
δx→0 x + xδx
1
= 1 − 2.
x

17
(f) y = 2x2 − 3.

dy 1
= lim [(2(x + δx)2 − 3) − (2x2 − 3)]
dx δx→0 δx
= lim [4x + 2δx]
δx→0
= 4x.

2.4. Let x = f (t) be the displacement function in each case. The average velocity over the interval
t to t + δt equals
[f (t + δt) − f (t)]/δt.
(a) x = f (t) = 3t. When t = 1
Interval δt 0.5 0.25 0.1 0.01
f (1 + δt) 4.5 3.75 3.3 3.03
Average velocity 3 3 3 3
(Since f (t) is linear in t the velocity 3 at all t.)
(b) x = f (t) = 5t2 . When t = 3.
Interval δt 0.5 0.25 0.1 0.01
f (3 + δt) 61.25 52.81 48.05 45.35
Average velocity 32.5 31.25 30.5 30.05
The values are approaching the limit 30.
(c) x = f (t) = 2t − 5t2 . When t = 1.
Interval δt 0.5 0.25 0.1 0.01
f (1 + δt) −8.25 3.75 −3.85 −3.08
Average velocity −10.5 −9.25 −8.5 −8.05
The limit is −8.
(d) x = 2t − 5t2 . When t = 0.2.
Interval δt 0.5 0.25 0.1 0.01
f (0.2 + δt) −1.25 −0.3125 −0.05 −0.0005
Average velocity −25 −1.25 −0.5 −0.05
In the limit the velocity is zero.
2.5. (a) dy/dx = 1 for all x; (b) dy/dx = 3x2 so that dy/dx = 27 at x = 3;
(c) dy/dx = 4x3 so that dy/dx = 32 at x = 2 and −32 at x = −2.
2.6. (a) y = x; dy/dx = 1: the graph is a straight line at 45◦ to the x axis.
y
2

x
-2 -1 1 2

-1

-2

Figure 31: Problem 2.6a

(b) y = x2 ; dy/dx = 2x: the slope is negative for x < 0 and positive for x > 0, and increases from
−∞ to +∞: the curve is a parabola.

18
y
1
0.8
0.6
0.4
0.2
x
-1 -0.5 0.5 1

Figure 32: Problem 2.6b


y
1

0.5

x
-1 -0.5 0.5 1

-0.5

-1

Figure 33: Problem 2.6c

(c) y = x3 ; dy/dx = 3x2 : the slope is positive except at x = 0 where it is zero.


(d) y = x4 ; dy/dx = 4x3 .

y
1
0.8
0.6
0.4
0.2
x
-1 -0.5 0.5 1

Figure 34: Problem 2.6d

(e) y = x5 ; dy/dx = 5x4 .


y
1

0.5

x
-1 -0.5 0.5 1

-0.5

-1

Figure 35: Problem 2.6e

2.7. For the displacement x = t3 , the velocity of the point is dx/dt = 3t2 and its acceleration is
d2 x/dt2 = 6t. The graph of acceleration against time is a straight line.
2.8. (a) If V = 43 πr3 then dV /dr = 4πr2 .
(b) If S = πd2 then dS/dd = 2πd.
(c) If E = kT 4 then dE/dt = 4kT 3 .
(d) If I = V /R then dI/dV = 1/R.
(e) If H = RI 2 then dH/dI = 2RI.
(f) If V = RT /P then dV /dT = R/P .
2.9.
d d d d
(a) (3x2 − 2x + 1) = 3 (x2 ) − 2 (x) + (1) = 6x − 2.
dx dx dx dx

19
d 7
(b) (x − 3x6 + x + 1) = 7x6 − 18x5 + 1.
dx

d
(c) (x + C) = 1.
dx

d d 2
(d) [x(x − 1)] = (x − x) = 2x − 1.
dx dx

d 2 2 d 4
(e) [x (x + 1) − 1] = [x + x2 + 1] = 4x3 + 2x.
dx dx

d
(f) (ax2 + bx + c) = 2ax + b.
dx

d d 2
(g). [(x − 1)2 ] = (x − 2x + 1) = 2x − 2.
dx dx

2.10. Let m1 and m2 be the slopes of the curves at the point of intersection, and check that
m1 m2 = −1. Then
(a) m1 = (d/dx)(1 + x − x2 ) = 1 − 2x = −1 at x = 1,
m2 = (d/dx)(1 − x + x2 ) = −1 + 2x = 1 at x = 1. Hence m1 m2 = −1 as required.
(b) m1 = −x = −1, m2 = 1 at x = 1.
(c) m1 = −x = −1, m2 = x = 1 at x = 1.
2.11. (a) The curves y = x2 and y = 1 − x2 intersect where x2 = 1 − x2 or where x2 = 21 . Hence
the points of intersection occur at A : ( √12 , 12 ) and B; (− √12 , 12 ).
The slopes of the curves at A are
√ √ √ √
m1 = 2x = 2/ 2 = 2 and m2 = −2x = −2/ 2 = − 2.

Let
√ 1 √ 1
tan α1 = 2 (0 < α1 < π) and tan α2 = − 2 (− π < α1 < 0).
2 2
Using the identity from (1.17a):
√ √
tan α1 − tan α2 2+ 2
tan(α1 − α2 ) = = √ √
1 + tan α1 tan α2 1− 2 2

= −2 2

We choose a positive value √for the angle (a sketch of the intersection of the curves is helpful).
Hence α1 − α2 = arctan(−2 2) = 109.47◦ .
The slopes of the curves at B are
√ √ √ √
n1 = −2x = −2/ 2 = − 2 and n2 = 2x = 2/ 2 = 2.

The two slopes at B are interchanged but otherwise the same. Hence the angle between the
tangents will also be 109.47◦ . (Note that in both these cases you might obtain the alternative
angles (180 − 109.47)◦ .)
(b) The curves y = 13 x3 and y = x2 − 2x + 43 intersect where

x3 = 3x2 − 6x + 4 or where (x − 1)(x2 − 2x + 4) = 0.

The only real root is x = 1. Hence the point of intersection is at (1, 31 ). The slopes of the curves at
this point are m1 = 1 and m2 = 0. Let tan α1 = 1 and tan α2 = 0. Then we can choose α1 = 14 π
and α2 = 0. The required angle is 41 π.

20
2.12. Use the limits in Section 2.6. Note that, in all the following, ε is never zero, so cancellation
is legitimate.
ε
(a) lim = lim 1 = 1.
ε→0 ε ε→0

ε 1 1
(b) lim = lim = .
ε→0 2ε ε→0 2 2

ε2
(c) lim = lim ε = 0.
ε→0 ε ε→0

e2ε − 1 eµ − 1
(d) lim = lim = 1, (µ = 2ε) (from (2.11)).
ε→0 2ε µ→0 µ

e2ε − 1 eµ − 1
(e) lim = lim 2 = 2, (µ = 2ε).
ε→0 ε µ→0 µ

sin 2ε sin µ
(f) lim = lim = 1, (µ = 2ε) (from (2.13).
ε→0 2ε µ→0 µ

sin 2ε sin µ
(g) lim = lim 2 = 2, (µ = 2ε).
ε→0 ε µ→0 µ

ln(1 + ε2 ) ln(1 + µ)
(h) lim 2
= lim 2 = 1, (µ = ε2 ) (from (2.14)).
ε→0 ε µ→0 µ
(i) Note that (2.13) is only true if ε is measured in radians. Therefore replace ε degrees by 180ε/π
radians. Hence
sin ε sin(πε/180) π sin µ
lim = lim = lim (µ = πε/180)
ε→0 ε ε→0 ε µ→0 180µ

= π/180 (from (2.13)).


(j)
tan ε sin ε 1 sin ε 1
lim = lim = lim lim
ε→0 ε ε→0 ε cos ε ε→0 ε ε→0 cos ε
= 1×1=1
(k)
sinh ε eε − e−ε e2²−1
lim = lim = lim lim e−ε
ε→0 ε ε→0 2ε ε→0 2ε ε→0
= 1×1=1

e−ε−1 eε − 1
(l) lim = lim lim [−e−ε ] = −1.
ε→0 ε ε→0 ε ε→0
2.13.
· ¸
d cos(x + δx) − cos x
(cos x) = lim
dx δx→0 δx
· ¸
−2 sin 12 (2x + δx) sin 12 (δx)
= lim
δx→0 δx
· ¸
1 sin( 12 δx)
= lim − sin(x + δx) 1
δx→0 2 2 δx
= − sin x

21
2.14.
· 2(x+ε) ¸ · 2ε ¸
d 2x e − e2x e −1
(a) (e ) = lim = 2e2x lim = 2e2x ,
dx ε→0 ε ε→0 2ε

(from (2.11)).
(b)
· ¸
d sin[2(x + ε)] − sin 2x
(sin 2x) = lim
dx ε→0 ε
· ¸
2 sin ε 1
= lim cos (4x + 2ε)
ε→0 ε 2
= 2 cos 2x

· −(x+ε) ¸ · −ε ¸
d −x e − e−x e −1
(c). (e ) = lim = −e−x lim = −e−x .
dx ε→0 ε ε→0 −ε

Thus
d 1 d x 1
(sinh x) = (e − e−x ) = (ex + e−x ) = cosh x.
dx 2 dx 2
d 1 d x 1
(cosh x) = (e + e−x ) = (ex − e−x ) = sinh x.
dx 2 dx 2
2.15.
d d d
(a) (2 sin x − 3 cos x) = 2 (sin x) − 3 (sin x) = 2 cos x + 3 sin x.
dx dx dx

d d 1
(b) (ln 3x) = (ln 3 + ln x) = .
dx dx x

d d 3
(c) (ln x3 ) = (3 ln x) = .
dx dx x

d
(d) (sin x − x) = cos x − 1.
dx

d x 1
(e) (e − 1 − x − x2 ) = ex − 1 − x.
dx 2

2.16. The required tangent lines are


(a) y = 3x − 2; (b) y = 24x − 39; (c) y = −x + 21 π; (d) y = x/e; (e) y = 1; (f) y = −x + 3.
2.17.
dy d2 y d3 y
(a) y = x6 , = 6x5 , = 30x4 , = 120x3 .
dx dx2 dx3

dy d2 y d3 y
(b) y = 3x2 − 2x + 2, = 6x − 2, = 6, = 0.
dx dx2 dx3

dy d2 y d3 y
(c) y = x6 − x2 , = 6x5 − 2x, = 30x4 − 2, = 120x3 .
dx dx2 dx3

22
dy
(d) y = 2 sin x − 3 cos x, = 2 cos x + 3 sin x,
dx

d2 y d3 y
= −2 sin x + 3 cos x, = −2 cos x − 3 sin x.
dx2 dx3

1 dy d2 d3 y
(e) y = ex − 1 − x − x2 , = ex − 1 − x, = ex − 1, = ex .
2 dx dx2 dx3

2.18. To prove that (dxN /dxN )(xN ) = N ! for all integers N ≥ 1!. We can confirm the formula
for the case N = 1:
d
(x) = 1 = 1! (i)
dx
as a starting-point in a step-by-step argument.
Suppose that we have somehow established that the result is true for any one particular value
of N , say for N = K, so that
dN (xN )
= K! when N = K. (ii)
dxN
Next, consider the transition to N = K + 1:
· K ¸ · ¸
dK+1 K+1 d d K+1 dK d K+1
(x ) = (x ) = (x )
dxK+1 dx dxK dxK dx
dK
= {(K + 1)xK } (by (2.9))
dxK
= (K + 1)K! (by (ii))
= (K + 1)!

In other words, if (ii) is true for some integer N = K, it follows that it is also true for
N = K + 1. Since we now know it is true for N = K + 1, the same argument implies that it is
true for N = (K + 1) + 1 = K + 2; and so on for all subsequent values of N .
But we have verified its truth in the case N = 1 (in equation (i)); therefore (ii) is true for all
values of N . This is proof by induction.
2.19. If y = x2 (x2 − 1), then

dy d2 y
= 4x3 − 6x, = 12x2 − 6.
dx dx2
√ √
(a) The slope of √
the curve is positive
√ where dy/dx > 0 or where x(2x + 3)(2x − 3) > 0. This
occurs where − 21 3 < x < 0 and 12 3 < x. √ √ √
of the curve is negative where x(2x + 3)(2x − 3) < 0, that is where x < − 12 3
(b) The slope √
and 0 < x < 12 3. √ √ √
second derivative is positive where 12(x + 12 2)(x − 12 2) > 0, that is where x > 21 2 and
(c) The √
x < − 12 2. √ √
(d) The second derivative is negative where − 12 2 < x < 12 2.
2.20. At x = x0 the tangent has slope m0 = 2x0 . Hence the slope of the normal is −1/m0 =
−1/(2ax0 ). The equation of the normal is therefore
1
y − ax20 = − (x − x0 ).
2ax0

Chapter 3: Further techniques for differentiation

23
3.1.
d d d
(a) (xex ) = x (ex ) + (x)ex = xex + ex .
dx dx dx

d
(b) (x sin x) = x cos x + sin x.
dx

d
(c) (x cos x) = −x sin x + cos x
dx

d x
(d) (e sin x) = ex cos x + ex sin x.
dx

d
(e) (x ln x) = 1 + ln x.
dx

d 2
(f) (x ln x) = x2 x−1 + 2x ln x = x + 2x ln x.
dx

d x ex
(g) (e ln x) = + ex ln x.
dx x

d 2 x
(h) (x e ) = x2 ex + 2xex .
dx

d
(i) (sin x cos x) = − sin2 x + cos2 x = cos 2x.
dx

d 2 3 d 5
(j) (x x ) = x2 .3x2 + 2x.x3 = 5x4 = (x ).
dx dx

3.2. All these problems illustrate the reciprocal and quotient rules given in (3.2).
(a)
d d ³ cos x ´ sin x(− sin x) − cos x(cos x)
(cot x) = =
dx dx sin x sin2 x
1
= − 2 = −cosec 2 x
sin x
µ ¶ µ ¶
d x 1 d d 1
(b) = (x + 1) (x) − x (x + 1) = .
dx x+1 (x + 1)2 dx dx (x + 1)2

µ ¶
d sin x 1
(c) = (x cos x − sin x).
dx x x2

µ ¶
d ex ex
(d) = (x − 1).
dx x x2

µ ¶
d x2 − 1 1 4x
(e) = [(x2 + 1)(2x) − (x2 − 1)(2x)] = 2 .
dx x2 + 1 (x2 + 1)2 (x + 1)2

24
µ ¶
d tan x 1 2 1
(f) = (x sec2 x − 2x tan x) = 3 (x sec2 x − 2 tan x).
dx x2 x4 x

(g)
µ ¶
d sin x + cos x
dx sin x − cos x
(sin x − cos x)(cos x − sin x) − (sin x + cos x)(cos x + sin x)
=
(sin x − cos x)2
2
= − .
(sin x − cos x)2

µ ¶
d d 1 sin x
(h) (sec x) = = = sec x tan x.
dx dx cos x cos2 x

µ ¶
d d 1 cos x
(i) (cosec x) = =− = − cot xcosec x.
dx dx sin x sin2 x

µ ¶
d x −2 − 3x2
(j) 2
= .
dx 3x − 2 (−2 + 3x2 )2

µ ¶
d 1 −1 − 4x3
(k) = .
dx x(x3 + 1) x2 (x3 + 1)

µ ¶
d 1 1
(l) =− .
dx ln x x(ln x)2

µ ¶
d n d 1
(m) (x ) = = nxn−1 (by the quotient rule).
dx dx x−n

µ ¶
d 1 1
(n) =− .
dx x+1 (x + 1)2

µ ¶
d −x d 1 1 d x
(o) (e ) = =− (e ) = −e−x .
dx dx ex e2x dx

µ ¶
d d 1
(p) (cot x) = −cosec 2 x (as in (a)).
=
dx dxtan x
µ ¶ µ ¶
d −2 d ln x 1 x2 1 − 2 ln x
(x ln x) = = − 2x ln x = .
dx dx x2 x4 x x3

3.3.
· ¸ · ¸
d 1 1 d2 1 2
(a) = 2
; 2
= ,
dx 1 − x (1 − x) dx (1 − x) (1 − x)2
· ¸
d3 1 6
= .
dx3 (1 − x) (1 − x)4

25
d d2
(b) (x sin x) = x cos x + sin x; (x sin x) = 2 cos x − x sin x;
dx dx2
d3
(x sin x) = −x cos x − 3 sin x.
dx3
µ ¶ µ ¶
d x 1 d2 x 2
(c) =− ; = ;
dx x−1 (x − 1)2 dx2 x − 1 (x − 1)3
µ ¶
d3 x 6
=− .
dx3 x − 1 (x − 1)4
(d) Let y = f (x)g(x). Then
dy df dg
=g +f ,
dx dx dx
d2 y df dg d2 f d2 g
= 2 + g + f ,
dx2 dx dx dx2 dx2
d3 y dg d2 f df d2 g d3 f d3 g
= 3 + 3 + g + f .
dx3 dx dx dx dx2 dx3 dx3

3.4. These problems use the chain rule (3.3) in the form

dy dy du
= .
dx du dx
(a) Let u = sin x. Then y = u2 , and

dy dy du
= = 2u cos x = 2 sin x cos x = sin 2x.
dx du dx
(b) Let u = cos x, y = u2 . Then (d/dx)(cos2 x) = −2 sin x cos x = − sin 2x.
(c) Let u = x2 , y = sin u. Then (d/dx)(sin x2 ) = 2x cos x2 .
(d) Let u = x2 , y = cos u. Then (d/dx)(cos x2 ) = −2x sin x2 .
(e) Let u = tan x, y = u2 . Then d/dx)(tan2 x) = 2 sec2 x tan x.
(f) Let u = x2 , y = tan u. Then (d/dx)(tan x2 ) = 2x sec2 x2 .
(g) Let u = 1/x, y = cos u. Then d/dx)[cos(1/x)] = 2 sin(1/x)/x2 .
(h) Let u = −x, y = eu . Then (d/dx)(e−x ) = −e−x .
(i) Let u = 1/(x + 1), y = u5 . Then (d/dx)(1/(x + 1)5 ) = −5/(x + 1)6 .
(j) Let u = x3 + 1, y = u4 . Then (d/dx)[(x3 + 1)4 ] = 12x2 (x3 + 1)3 .
(k) Let u = 3x, y = sin u. Then (d/dx)(sin 3x) = 3 cos 3x.
(l) Let u = 12 x, cos u. Then (d/dx)(cos 12 x) = − 12 sin 12 x.
(m) Let u = 12 x, y = tan u. Then (d/dx)(tan 12 x) = 12 sec2 x.
(n) Let u = −3x, y = eu . Then (d/dx)(e−3x ) = −3e−3x .
(o) Let u = 2x + 1, y = sin u. Then (d/dx)[sin(2x + 1)] = 2 cos(2x + 1).
(p) Let u = 3x − 2, y = cos u. Then (d/dx)[cos(3x − 2)]) = −3 sin(3x − 2).
(q) Let u = 1 − 2x, y = tan u. Then (d/dx)[tan(1 − 2x)] = −2 sec2 (1 − 2x).
(r) Let u = 1/x, y = eu . Then (d/dx)(e1/x ) = −e1/x /x2 .

3.5. All these problems use the result that (d/dx)xα = αxα−1 .

d −2
((a)) (x ) = −2x−3 .
dx

d −1 1
(b) (x ) = − 2 .
dx x

26
d 1 1 2
(c) x 3 = x− 3 .
dx 3

d −1 1 4
(d) (x 3 ) = − x− 3 .
dx 3

d 3 3 1
(e) (x 2 ) = x 2 .
dx 2

d √ d 1 1 1
(f) ( x) = (x 2 ) = x− 2 .
dx dx 2

d √ 3 d 3 3 1
(g) x = (x 2 ) = x 2 .
dx dx 2

µ ¶
d 1 d −1 1
(h) = (x ) = − 2 .
dx x dx x

· ¸
d 1 d −1 −1
(i) √ = (x 2 ) = 3 .
dx x dx 2x 2

3.6. (a)
d 1 1 sin x
(x 2 sin x) = x 2 cos x + 1 .
dx 2x 2

d 1 cos x
(b) (sin 3 x) = 2 .
dx 3 sin 3 x

d 1 x
(c) [(x2 + 1)− 2 ] = − 3 .
dx 2
(x + 1) 2

d
(d) [sin2 (3t + 1)] = 6 cos(3t + 1) sin(3t + 1).
dt

d −t
(e) (e cos t) = −e−t (cos t + sin t).
dt

d −t
(f) (e sin t) = e−t (cos t − sin t).
dt

d −2t
(g) (e cos 3t) = −e−2t (2 cos 3t + 3 sin 3t).
dt

d −3t
(h) (e cos 2t) = −e−3t (3 cos 2t + 2 sin 2t).
dt

d
(i) (sin x cos2 x) = cos3 x − 2 cos x sin2 x.
dx

27
d
(j) (sin2 x cos x) = 2 cos2 x sin x − sin3 x.
dx

"µ ¶2 #
d sin x 2 cos x sin x 2 sin2 x
(k) = − .
dx x x2 x3

d £ ¤
(l) x(sin3 x) = 3x cos x sin2 x + sin3 x.
dx

d £ ¤
(m) x(cos3 x) = −3x cos2 x sin x + cos3 x.
dx

3.7. (a)
d d 1
(cos2 x) = (1 + cos 2x) = − sin 2x.
dx dx 2
d d 1
(sin2 x) = (1 − cos 2x) = sin 2x.
dx dx 2
(b)
d d
(cos2 x) = (cos x cos x) = − cos x sin x − sin x cos x = − sin 2x.
dx dx
d d
(sin2 x) = (sin x sin x) = sin x cos x + cos x sin x = − sin 2x.
dx dx
(c) To apply the chain rule let u = cos x, y = u2 . Then

d d 2 d
(cos2 x) = (u ) (cos x) = −2u sin x = −2 cos x sin x = − sin 2x.
dx du dx
Let u = sin x Then
d d 2 d
(sin2 x) = (u ) (sin x) = 2u cos x = 2 sin x cos x = sin 2x.
dx du dx

3.8.
d2 x
(a) + 4x = (−4A cos 2t − 4B sin 2t) + 4(A cos 2t + B sin 2t) = 0
dt2

d2 x
(b) + n2 x = (−n2 A cos nt − n2 B sin nt) + n2 (A cos nt + B sin nt) = 0.
dt2

d2 x
(c) − 9x = (9Aent + 9Be−nt ) − 9(Aent + Be−nt ) = 0.
dt2

d2 x
(d) − n2 x = (n2 Aent + n2 Be−nt ) − n2 (Aent + Be−nt ) = 0.
dt2

dx
(e) = (−A + B)e−t cos t − (A + B)e−t sin t.
dt
d2 x
= −2Be−t cos t + 2Ae−t sin t.
dt2

28
Hence
d2 x dx
2
+2 + 2x =
dt dt
[−2Be−t cos t + 2Ae−t sin t] +
2[(−A + B)e−t cos t − (A + B)e−t sin t] + 2[Ae−t cos t + Be−t sin t] = 0.

(f) The fourth derivative of each term in y returns the same function in each case. Hence

d4 y
− y = 0.
dx4

3.9. Use the chain rule in the form


dy dy dv du
= .
dx dv du dx
The intermediate variables are defined in each problem.
(a) Let u = cos x, v = u2 , so that y = ev . Hence

dy 2
= ev .2u.(− sin x) = −2ecos x cos x sin x.
dx
(b) Let u = x2 , v = cos u, so that y = e−v . Hence

dy 2
= (−e−v ).(− sin u).2x = 2x sin(x2 )e− cos x .
dx
(c) Let u = x2 , v = cos u, so that y = ln v. Hence

dy 1
= .(− sin u).2x = −2x tan(x2 ).
dx v
(d) Let u = x2 , v = eu − 1, so that y = v 4 . Hence

dy 2 2
= 4v 3 .eu .2x = 8xex (ex − 1)3 .
dx

3.10. Use the result (3.7) which states that if y = u(x)v(x)w(x) then ln y = ln u + ln v + ln w
which when differentiated gives
dy du dv dw
= vw + wu + uv .
dx dx dx dx
(a) Let u = x, v = ex , w = sin x. Then

dy
= ex . sin x.1 + sin x.x.ex + x.ex . cos x = ex [sin x + x sin x + x cos x].
dx
(b) Different variables are used. Let x = tet cos t, and let u = t, v = et , w = cos t. Hence

dx
= et . cos t.1 + cos t.t.et + t.et .(− sin t) = et [cos t + t cos t − t sin t].
dt
1 1
(c) Let u = x 2 , v = e2x , w = sin 2 3x. Then

dy 1 1 1 1 1 1 3 3
= e2x . sin 2 3x.( x− 2 ) + sin 2 3x.x 2 .2e2x + x 2 .e2x . cos 3x sin− 2 3x
dx 2 2
e2x
= 1 1 [3x cos 3x + (4x + 1) sin 3x].
2x 2 sin 2 x

29
This function and its derivative will only be real for restricted values of x—for example, for 0 ≤
x ≤ 13 π.
3.11. (a) Treating y as a function of x, and using the chain rule for y(x),
d 2 dy
(x + y 2 ) = 0, or 2x + 2y = 0.
dx dx
Hence dy/dx = −x/y as required. Solving the equation x2 + y 2 = 4 for y, it follows that y =
1
±(4 − x2 ) 2 . Differentiating with respect to x, we have
dy 1 x
= ∓x(4 − x2 )− 2 = − ,
dx y
after substitution back in terms of y. This agrees with the answer obtained by the implicit method.
Note that will always be two points on a circle which have the same slope. The tangent is always
perpendicular to the radius to the point which has slope m = y/x.
(b) In this case for x > 0, y ≥ 0,
d 1 1 1 1 1 1 dy
(x 2 + y 2 ) = 0, or x− 2 + y − 2 = 0.
dx 2 2 dx
Hence r
dy y
=− .
dx x
(c) In this case
d 3 dy dy
(x + xy − y 3 ) = 0 or 3x2 + y + x − 3y 2 .
dx dx dx
Hence
dy y + 3x2
=− .
dx x − 3y 2
(d) In this case
d dy dy
(x sin y − y sin x) = 0 or sin y + x cos y − sin x − y cos x = 0.
dx dx dx
Hence
dy y cos x − sin y
= .
dx x cos y − sin x

3.12. The expression for dy/dx obtained from the implicit relation f (x, y) = c does not depend
on c. For example for x2 + y 2 = c, we always have dy/dx = −x/y. However, the value of dy/dx
will depend indirectly on c since x and y must always satisfy x2 + y 2 = c.
3.13. If xy 2 − x2 y = 1, then
dy dy
y 2 + 2xy − 2xy − x2 = 0. (i)
dx dx
Hence
dy 2xy − y 2
= . (ii)
dx 2xy − x2
Differentiate (i) again with respect to x:
µ ¶2
dy dy dy d2 y dy dy d2 y
2y + 2y + 2x + 2xy 2 − 2y − 2x − 2x − x2 2 = 0. (iii)
dx dx dx dx dx dx dx
Eliminate dy/dx between (ii) and (iii): the answer is
d2 y 6xy(−x3 + 2x2 y − 2xy 2 + y 3 )
= .
dx2 (2xy − x2 )3

30
3.14. (a) Let y = arcsin x. Then x = sin y. Differentiating with respect to y,
dx √ √
= cos y = (1 − sin2 y) = (1 − x2 ).
dy
Hence µ ¶−1
dy dx 1
= =√ .
dx dy (1 − x2 )
(b) Let y = arccos x so that x = cos y. Then
µ ¶−1
dy dx 1 1
= =− = −√ .
dx dy sin y (1 − x2 )

(c) Let y = arctan x so that x = tan y. Then


µ ¶−1
dy dx 1 1
= = 2
= cos2 y = .
dx dy sec y 1 + x2

(d) Let y = sinh−1 x so that x = sinh y. Then


µ ¶−1
dy dx 1 1 1
= = =√ =√ .
dx dy cosh y (1 + sinh2 y) (1 + x2 )

(e) Let y = cosh−1 x so that x = cosh y. Then


µ ¶−1
dy dx 1 1 1
= = =√ 2 =√ 2 .
dx dy sinh y (cosh −1) (x − 1)

(f) Let y = tanh−1 x so that x = tanh y. Then


µ ¶−1
dy dx 1 1 1
= = = = .
dx dy sech 2 y 1 − tanh2 y 1 − x2

3.15. (a) If r = sin 21 θ, then the (x, y) coordinates are

1 1
x = r cos θ = sin θ cos θ, y = r sin θ = sin θ sin θ.
2 2
Using parametric differentiation,
Á 1
dy dy dx 2 cos 12 θ sin θ + sin 12 θ cos θ
= = 1 .
dx dθ dθ 2 cos 12 θ cos θ − sin 12 θ sin θ

At θ = 12 π,
1
2 2 +0

dy 1
= =− .
dx 0 − √12 2

(b) If r = 1 + sin2 θ, then

x = r cos θ = (1 + sin2 θ) cos θ, y = r sin θ = (1 + sin2 θ) sin θ.

Hence
dx 1
= − sin θ − sin3 θ + 2 cos2 θ sin θ = − √ at θ = 14 π,
dθ 2 2
and
dy 1 3 5 1
= cos θ + 3 sin2 θ cos θ = √ + √ = √ at θ = π.
dθ 2 2 2 2 2 4

31
Hence Á
dy dy dx
= = −5
dx dθ dθ
.

3.16. (a) For x = t3 and y = t2 ,


Á
dy dy dx 2t 2 2
= = 2 = = 1 .
dx dt dt 3t 3t 3x 3
(b) For x = 2 cos t and y = 2 sin t,
Á
dy dy dx 2 cos t x
= = =± √ ,
dx dt dt −2 sin t 2 (4 − x2 )

assuming 0 ≤ t ≤ 12 π.
3.17. Elimination of t between x and y, using the identity cos2 t + sin2 t = 1, gives the ellipse

x2 y2
+ = 1.
a2 b2
The derivative is given by
Á
dy dy dx b cos t b
= = = − cot t.
dx dt dt −a sin t a

The speed of the point is


"µ ¶2 µ ¶2 # 12
dx dy √
+ = [a2 sin2 t + b2 cos2 t].
dt dt

3.18. In exponential form ax = ex ln a . Hence


d x
(a ) = (ln a)ex ln a = xa ln a.
dx

Chapter 4: Applications of differentiation

4.1. f (u) = u2 , so f 0 (u) = 2u and f 00 (u) = 2 for all arguments u.


(a) Let u = t. Then f 0 (t) = 2t.
(b) Let u = t2 . Then f 0 (t2 ) = 2u = 2t2 .
(c) (d/dt)f (t2 ) = (d/dt)t4 = 4t3 .
1 1 1
(d) Let u = t 2 . Then f 0 (t 2 ) = 2u = 2t 2 .
1
(e) (d/dt)f (t 2 ) = (d/dt)t = 1.
1 1
(f) Let u = t 2 . Then f 00 (t 2 ) = 2.
4.2. Denote the function in each case by f (x). The stationary points are given by f 0 (x) = 0. If
A : x = a is a stationary point, then A is a minimum if f 00 (a) > 0, or a maximum if f 00 (a) < 0. If
f 00 (a) = 0, then the stationary point can be a minimum, maximum or point of inflection depending
on the sign of f 0 (x) on either side of x = a.
(a) Since f (x) = x2 − x, then

f 0 (x) = 2x − 1, f 00 (x) = 2.

Stationary point: x = 12 .

32
Test: f 00 ( 12 ) = 2 > 0 so x = 12 is a minimum.
(b) Since f (x) = x2 − 2x − 3, then

f 0 (x) = 2x − 2, f 00 (x) = 2.

Stationary point: x = 1.
Test: f 00 (1) = 2 > 0 so x = 1 is a minimum.
(c) Since f (x) = x ln x, then
1
f 0 (x) = 1 + ln x, f 00 (x) = .
x
Stationary point: x = e−1 .
Test: f 00 (e−1 ) = e > 0 so x = e−1 is a minimum.
(d) Since f (x) = xe−x , then

f 0 (x) = (1 − x)e−x , f 00 (x) = (−2 + x)e−x .

Stationary point: x = 1.
Test: f 00 (1) = −e−1 < 0 so x = 1 is a maximum.
(e) Since f (x) = 1/(x2 + 1), then

−2x 2(3x2 − 1)
f 0 (x) = , f 00 (x) = .
(x + 1)2
2 (x2 + 1)3
Stationary point: x = 0.
Test: f 00 (0) = −2 < 0 so x = 0 is a maximum.
(f) Since f (x) = x2 − 3x + 2, then

f 0 (x) = 2x − 3, f 00 (x) = 2.

Stationary point: x = 32 .
Test: f 00 ( 32 ) = 2 > 0 so x = 32 is a minimum.
(g) Since f (x) = ex + e−x , then

f 0 (x) = ex − e−x , f 00 (x) = ex + e−x .

Stationary point: x = 0.
Test: f 00 (0) = 2 > 0 so x = 0 is a minimum.
(h) Since f (x) = x2 + 4x + 2, then

f 0 (x) = 2x + 4, f 00 (x) = 2.

Stationary point: x = −2.


Test: f 00 − 2) = 2 > 0 so x = −2 is a minimum.
(i) Since f (x) = x − x3 , then

f 0 (x) = 1 − 3x2 , f 00 (x) = −6x.



Stationary points:
√ x = ±1/
√ 3. √
00
Tests: f√ (1/ 3) √
= −6/ 3 < 0, so x√= 1/ 3 is a maximum.
f 00 (−1/ 3) = 6/ 3 > 0 so x = −1/ 3 is a minimum.
(j) Since f (x) = x2 (x − 1), then

f 0 (x) = 3x2 − 2x, f 00 (x) = 6x − 2.

Stationary points: x = 0 and x = 23 .

33
Tests: f 00 (0) = −2 < 0 so x = 0 is a maximum.
f 00 ( 23 ) = 2 > 0 so x = 23 is a minimum.
(k) Since f (x) = sin x − cos x, then

f 0 (x) = cos x + sin x, f 00 (x) = − sin x + cos x.

Stationary points: √ x = 43 π and 74 π for 0 < x < 2π.


00 3 3
Tests: f (√ 4 π) = − 2 < 0 so x = 4 π is a maximum;
00 7 7
f ( 4 π) = 2 > 0 so x = 4 π is a minimum.
(l) Since f (x) = sin x cos x = 12 sin 2x, then

f 0 (x) = cos 2x, f 00 (x) = −2 sin 2x.

Stationary points: x = − 43 π, x = − 14 π, x = 14 π, x = 43 π for −π < x < π.


Tests: f 00 (− 43 π) = −2 < 0 so x = − 34 π is a maximum;
f 00 (− 14 π) = 2 > 0 so x = − 14 π is a minimum;
f 00 ( 14 π) = −2 < 0 so x = 14 π is a maximum;
f 00 ( 34 π) = 2 > 0 so x = 34 π is a minimum.
(m) Since f (x) = e−x sin x, then

f 0 (x) = e−x (− sin x + cos x), f 00 (x) = −2e−x cos x.

Stationary points occur where tan x = 1, at x = (n + 14 )π, (n = 0, ±1 ± 2, . . .).


√ 1
Tests: f 00 [(n + 14 )π] = − 2(−1)n e−(n+ 4 )π < 0 or > 0 according as n is even or odd. Hence the
stationary point is a maximum x = (n + 14 )π is a maximum if n is even, and a minimum if n is
odd.
1
(n) Since f (x) = e− 3 x sin 2x, then
1 1
f 0 (x) = 13 e− 3 x (− sin 2x + 6 cos 2x), f 00 (x) = 19 e− 3 x (−12 cos 2x − 35 sin 2x).

Stationary points occur where tan 2x = 6 at x = α+ 12 nπ, (n = 0, ±1±2, . . .), where α = 21 arctan 6.
1 1 √
Tests: f 00 (α + 12 nπ) = −222e− 3 (α+ 2 nπ) (−1)n /[9 37] < 0 or > 0 according as n is even or odd.
Hence the stationary point x = α + 21 nπ is a maximum if n is even, and a minimum if n is odd.
(o) Since f (x) = x − cos x, then

f 0 (x) = 1 + sin x, f 00 (x) = cos x.

Stationary points occur where sin x = −1 at x = (2n − 12 )π, (n = 0, ±1, ±2, . . .).
Tests: f 00 [(2n − 12 )π] = 0 for all n. Hence the test fails. But f 0 (x) = 1 + sin x ≥ 0 for all x. Hence
all the stationary points must be points of inflection.
(p) Since f (x) = 2ex − 12 e2x , then

f 0 (x) = 2ex − e2x , f 00 (x) = 2ex − 2e2x .

Staionary point occurs where ex = 2 at x = ln 2.


Test: f 00 (ln 2) = −4 < 0 so x = ln 2 is a maximum.
(q) Since f (x) = x2 e−x , then

f 0 (x) = xe−x (2 − x), f 00 (x) = e−x (2 − 4x + x2 ).

Stationary points at x = 0 and x = 2.


Tests: f 00 (0) = 2 > 0 so that x = 0 is a minimum; f 00 (2) = −2e−2 < 0 so that x = 2 is a maximum.
(r) Since f (x) = (ln x)/x, then
1 1
f 0 (x) = (1 − ln x), f 00 (x) = (−3 + 2 ln x).
x2 x3

34
Stationary point where ln x = 1 at x = e.
Test: f 00 (e) = −e−3 < 0 so x = e is a maximum.
(s) Since f (x) = (1 − x)3 , then

f 0 (x) = −3(1 − x)3 , f 00 (x) = 6(1 − x)2 .

Stationary point: x = 1.
Test: f 00 (1) = 0 and the test fails. However, f 0 (x) ≤ 0 for all x. Hence x = 1 is a point of inflection.
(t) Since f (x) = sin3 x, then

f 0 (x) = 3 sin2 x cos x, f 00 (x) = 3 sin x(2 − 3 sin2 x).

Stationary points occur where sin x = 0, at x = nπ, and where cos x = 0, at x = (n + 21 )π for
n = 0, ±1, ±2, . . ..
Tests: f 00 (nπ) = 0 so that the test fails. However, if n is even, then f 0 (x) is positive in a small
interval including x = nπ, and if n is odd, then f 0 (x) is negative in a small interval including
x = nπ. In both cases, therefore, the stationary point is a point of inflection.
f 00 [(n + 12 )π] = −3(−1)n so that x = (n + 12 )π is a maximum if n is even, and a minimum if n
is odd.
2
(u) Since f (x) = e−x , then
2 2
f 0 (x) = −2xe−x , f 00 (x) = 2e−x (2x2 − 1).

Stationary point: x = 0.
Test: f 00 (0) = −2 < 0 so x = 0 is a maximum.
2
(v) Since f (x) = ex −x
, then
2 2
f 0 (x) = (2x − 1)ex −x
, f 00 (x) = (4x2 − 4x + 3)ex −x
.

Stationary point: x = 12 .
1
Test: f 00 ( 12 ) = 2e− 4 > 0 so x = 1
2 is a minimum.
(w) Since f (x) = x + x−1 , then
1 2
f 0 (x) = 1 − , f 00 (x) = .
x2 x3
Stationary points: x = ±1.
Tests: f 00 (1) = 2 > 0 so x = 1 is a minimum; and f 00 (−1) = −2 < 0 so x = −1 is a maximum.
(x) Since f (x) = x3 e−x , then

f 0 (x) = x2 e−x (3 − x), f 00 (x) = xe−x (6 − 6x + x2 ).

Stationary points: x = 0 and x = 3.


Tests: f 00 (0) = 0 and the test fails. In a small interval which includes the origin f 0 (x) > 0 which
means that x = 0 is a point of inflection.
f 00 (3) = −9e−3 < 0 so x = 3 is a maximum.
4.3. If y = f [u(x)], then by the chain rule

dy df du
= = f 0 (u)u0 (x),
dx du dx
and
d2 y d 0 d 0
= [f (u)u0 (x)] = [f (u)]u0 (x) + f 0 (u)u00 (x) (product rule)
dx2 dx dx
= f 00 (u)[u0 (x)]2 + f 0 (u)u00 (x)

35
Since f 0 (u) > 0 for all u, then dy/dx can only be zero if u0 (x) = 0 (by the chain rule (3.3)). Hence
f [u(x)] and u(x) have stationary points only at the same values of x.
In 4.2(v), f (u) = eu and u = x2 − x.
4.4. If the sides have lengths x > 0 and y > 0, then the given area A = xy. The length of the
perimeter is P = 2x + 2y. Eliminate y so that
2A
P = 2x + .
x
The first and second derivatives of P are
dP 2A d2 P 4A
=2− 2, 2
= 3.
dx x dx x

Hence the perimeter length
√ is stationary where dP/dx = 0: at x = ± A. Since x > 0, choose the
stationary value x = A. For this value

d2 P 4A
= 3 > 0,
dx2 A2

so that the perimeter is a minimum when x = y = A. The piece of ground must be a square.
4.5. Let the the base of the cross-section be x > 0 which will also be the diameter of the semicircle,
and let the height of the rectangle be y > 0. The given area A of the tunnel cross-section is

A = xy + 18 πx2 .

The length of the perimeter is P = x + 2y + 12 πx. Eliminate y, so that


µ ¶
2 1 2 1 2A 1
P = A − πx + x(1 + π) = + (1 + π)x.
x 8 2 x 4

This is stationary where


dP 2A 1
= − 2 + 1 + π = 0,
dx x 4
q
8A
which occurs at x = 4+π (choosing the positive root). The perimeter is a minimum since

d2 P 4A
= 3 >0
dx2 x
at the stationary point.
4.6. Let r be the radius of the base and h the height of the drum. The volume V of the drum is
given by V = πr2 h and its prescribed surface area by A = 2πr2 + 2πrh. We are given that A is a
constant, so eliminate h in the expression for V :
1
V = [Ar − 2πr3 ].
2
Differentiating
dV 1 d2 V
= [A − 6πr2 ], = −6πr.
dr 2 dr2
The volume is stationary where r
dV A
= 0, at r = ,
dr 6π
choosing the positive root. Obviously d2 V 2
√/dr < 0 which proves that this radius gives a minimum
volume. The height of this drum is h = [2A/(3π)] which is equal to its diameter.

36
4.7. Similar to 4.6: the volume is given by the same formula but the prescribed A is different:

V = πr2 h, A = πr2 + 2πrh.

Elimination of h leaves
1
V = r(A − πr2 ).
2
Differentiating
dV 1 d2 V
= [A − 3πr2 ], = 3πr.
dr 2 dr2
Hence the radius and height of the drum of minimum volume are
r
A
r=h= .

4.8. (a) y = 1/(x2 + 1):
y
1

0.8

0.6

0.4

0.2

x
-4 -2 2 4

Figure 36: Problem 4.8a


2
(b) y = ex :
y

0.8

0.6

0.4

0.2

x
-2 -1 1 2

Figure 37: Problem 4.8b

(c) y = x/(x − 1):


y
4

x
-3 -2 -1 1 2 3 4

-1

-2

-3

-4

Figure 38: Problem 4.8c

(d) y = xe−x :
(e) y = x2 e−x :
(f) y = x3 e−x :
(g) y = e2x − 4ex :
(h) y = (ln x)/x for x > 0:
(i) [ln(−x)]/x for x < 0:
(j) y = x ln x − x for x > 0:

37
y
0.5

x
-1 -0.5 0.5 1 1.5 2

-0.5

-1

-1.5

Figure 39: Problem 4.8d


y
1

0.8

0.6

0.4

0.2
x
-1 -0.5 0.5 1 1.5 2
-0.2

Figure 40: Problem 4.8e

(k) y = sin(1/x):
(l) y = (x2 − 1)2 :
(m) y = x(x2 − 1)2 :
(n) y = (sin x)/x:
4.9. (a) y = 1/(x2 − 1):
(b) y = x/(x2 − 1):
(c) 1/[x(x − 2)]:
(d) y = x3 /(1 − x):
(e) y = (x + 2)/(x − 1):
(f) y = 1/(x + 1) + 1/(x + 2):
4.10. The incremental formula given by (4.4) is

δy ≈ f 0 (a)δx at x = a.

The exact value is given by


δy = f (a + δx) − f (a).
(a) f (x) = x3 : δy ≈ 3x2 δx. With x = 2 and δx = 0.1, the approximate and exact values are given
by
δy ≈ 1.200, δy = (2.1)3 − 23 = 1.157 . . . .
1
(b) f (x) = x sin x: δy ≈ (sin x + x cos x)δx. With x = 2π and δx = −0.2 the approximate and
exact values are given by
1 1 1
δy ≈ (sin π + π cos π)(−0.2) = −0.2,
2 2 2
1 1 1 1
δy = ( π − 0.2) sin( π − 0.2) − π sin π = −0.227 . . . .
2 2 2 2

y
1
0.75
0.5
0.25
x
-1 -0.5 0.5 1
-0.25
-0.5
-0.75
-1

Figure 41: Problem 4.8f

38
y
10
7.5
5
2.5
x
-1 -0.5 0.5 1 1.5 2
-2.5
-5
-7.5

Figure 42: Problem 4.8g


y

x
0.5 1 1.5 2

-2

-4

-6

-8

Figure 43: Problem 4.8h

(c) f (x) = cos x: δy ≈ − sin xδx. With x = 14 π and δx = 0.1 the approximate and exact values
are given by
1
δy ≈ (− sin π)(0.1) = −0.0707 . . . ,
4
1 1
δy = cos( π + 0.1) − cos( π) = −0.0741 . . . .
4 4
2
(d) f (x) = (1 + x)/(1 − x): δy = 2/(1 − x) δx. With x = 2 and δx = −0.2 the approximate and
exact values are given by
2
δy ≈ (−0.2) = −0.4, δy = −0.5.
(1 − 2)2

(e) y = tan x: δy ≈ sec2 xδx. With x = 14 π and δx = 0.1 the approximate and exact values are
given by
1 1 1
δy ≈ (sec2 π)(0.1) = 0.2, δy = tan( π + 0.1) − tan π = 0.223 . . . .
4 4 4
2 0 2 2
(f) f (x) = 1/(1 − x ): f (x) = 2x/(1 − x ) . With x = 0.5 and δx = ±0.1 the approximate and
exact values are given by
1
δy ≈ (±0.1) = ±0.177 . . . ,
(1 − 0.52 )2
1 1
δy = 2
− = 0.229 . . . or − 0.142 . . . .
1 − (0.05 ± 0.1) 1 − (0.05)2

4.11. (a) With f fixed,


dv f2
=− .
du (u − f )2

y
6
5
4
3
2
1
x
-3 -2.5 -2 -1.5 -1 -0.5
-1

Figure 44: Problem 4.8i

39
y

0.5

x
1 2 3 4

-0.5

-1

Figure 45: Problem 4.8j

y
1
0.5
x
0.1 0.2 0.3 0.4 0.5 0.6 0.7
-0.5
-1

Figure 46: Problem 4.8k

Hence, with f = 0.75, u = 1.25 and δu = 0.05,

f 2 δu −(0.75)2 (0.05)
δv ≈ − 2
= = −0.112 . . . .
(u − f ) (1.25 − 0.75)2

(b) The voltage is given by


E(R1 R4 − R2 R3 )
v= .
(R1 + R2 )(R3 + R4 )
Its derivative with respect to R1 is
dv ER2
= .
dR1 (R1 + R2 )2
Hence
ER2 5
δv ≈ = δR1 .
(R1 + R2 )2 18
(c) With b and A constant in a = b sin A/(sin B),
da −b sin A cos B
= = −a cot B.
dB sin2 B
Hence
δa ≈ −a cot B δB.

(d) In terms of a, b, c,
1√
A= [(a + b + c)(−a + b + c)(a − b + c)(a + b − c)].
4

y
1.2
1
0.8
0.6
0.4
0.2
x
-1.5 -1 -0.5 0.5 1 1.5
-0.2

Figure 47: Problem 4.8l

40
y
1
0.8
0.6
0.4
0.2
x
-1.5 -1 -0.5 0.5 1 1.5
-0.2
-0.4

Figure 48: Problem 4.8m


y
1
0.8
0.6
0.4
0.2
x
-10 -5 5 10
-0.2
-0.4

Figure 49: Problem 4.8n

Logarithmic differentiation (see equation (3.7)) gives


· ¸
1 dA 1 1 1 1 1
= + + + .
A dc 2 a + b + c −a + b + c a − b + c a + b − c

The incremental formula for δA becomes, at a = 2, b = 4, c = 5


dA 25
δA ≈ δc = − √ (0.1) = 0.0822 . . . .
dc 2 231

4.12. Given C = P (1 + r)n .


(a) With n and P fixed,

dC
= P n(1 + r)n−1 , so that δC ≈ P n(1 + r)n−1 δr.
dr
(b) With r and P fixed,

dC d n ln(1+r)
=P e = P (1 + r)n ln(1 + r) see Problem 3.18.
dn dn
Hence
δC ≈ P (1 + r)n ln(1 + r)δn.
(c) Suppose that P = £100, r = 0.05 (5% ) and n = 10 years. The tables below show comparisons
between the approximate increments δC for decreasing values of δr (n fixed) and δn (r fixed).

y
12
10
8
6
4
2
x
-2 -1 1 2

Figure 50: Problem 4.9a

41
y

10

x
-2 -1 1 2

-5

-10

Figure 51: Problem 4.9b


y

10

x
-1 1 2 3

-5

-10

Figure 52: Problem 4.9c

δr approximate increment in C exact increment in C


P n(1 + r)n−1 δr P (1 + r + δr)n − P (1 + r)n
0.01 15.513 . . . 16.195 . . .
0.005 7.756 . . . 7.925 . . .
0.001 1.551 . . . 1.557 . . .
0.0001 0.155 . . . 0.155 . . .

δn approximate increment in C exact increment in C


P (1 + r)n ln(1 + r)δn P (1 + r)n+δn − P (1 + r)n
1 7.947 . . . 8.144 . . .
0.1 0.795 . . . 0.797 . . .
0.01 0.080 . . . 0.080 . . .

4.13. The iterations in Newton’s method are


f (xn )
xn+1 = xn − , n = 0, 1, . . . ,
f 0 (xn )

for a given initial value x0 . (a) Let f (x) = x4 + 2x2 − x − 1. Then

f 0 (x) = 4x3 + 4 − 1.

For example, if we start with x0 = 0.75, we obtain


f (0.75)
x1 = 0.75 − = 0.833 . . . .
f 0 (0.75)

y
5

2.5

x
-2 -1 1 2 3

-2.5

-5

-7.5

-10

-12.5

-15

Figure 53: Problem 4.9d

42
y
10

7.5

2.5

x
-4 -3 -2 -1 1 2 3

-2.5

-5

-7.5

-10

Figure 54: Problem 4.9e


y
10

7.5

2.5

x
-4 -3 -2 -1 1

-2.5

-5

-7.5

-10

Figure 55: Problem 4.9f

Repeat the process starting with x1 to obtain x2 , and so on. The solution is x = 0.825 . . ..
1
(b) Let f (x) = x4 + x 3 − 1. Then
1 2
f 0 (x) = 4x3 + x− 3 .
3
The solution is x = 0.619 . . ..
(c) Let f (x) = x ln x + 0.3. Then
f 0 (x) = 1 + ln x.
The solution is x = 0.168 . . ..
(d) Let f (x) = ex − 4x3 . Then
f 0 (x) = ex − 12x2 .
The solution is x = 0.831 . . ..
(e) Let f (x) = tan x − 2x. Then
f 0 (x) = sec2 x − 2.
By Newton’s method the solution is x = 1.165 . . . 4
(f) Let f (x) = ex sin x/(1 + x). Then

f (x) (1 + x)[2(1 + x)e−x − sin x]


x− = x + .
f 0 (x) (1 + x) cos x + x sin x

Let x0 = 1.85. Then


x1 = 1.663, x2 = 1.689, x3 = 1.690
to three decimal places.
4.14. Since f (x) = xe−x + 1 then f 0 (x) = (1 − x)e−x . The function has its only stationary point
at x = 1 which is a maximum. The slope of y = f (x) in the neighbourhood of the solution of
f (x) = 0 is therefore positive whilst that for any value of x greater than 1 will be negative. By
the geometrical construction of Newton’s method illustrated in Figure 4.15, any tangent which
starts for x > 1 will produce iterations which diverge from the required solution. The graph of
y = xe−x − 1 is shown in the figure.
4.15. (a) The graph shows a continuous function in which f (a) and f (b) have opposite signs.
(b) Let g(x) = ex − 3x. The table gives a sequence of values for g(x) at intervals 0.25.
x 0 0.25 0.5 0.75 1.0 1.25 1.5
g(x) 1 0.534 0.149 -0.133 -0.282 -0.260 -0.018

43
y

x
-2 -1 1 2
-1

-2

-3

-4

Figure 56: Problem 4.14


y

a fHbL
x
b

fHaL

Figure 57: Problem 4.15

x 1.75 2.0 2.25 2.5


g(x) 0.505 1.389 2.738 4.682

Evidently the solutions of the equation lie between x = 0.5 and x = 0.75, and between x = 1.5 and
x = 1.75. Note also that the function has a minimum value at x = ln 3 = 1.098 . . ., which means,
for example, that any initial value for the smaller solution must start at a value of x < ln 3 for the
reasons outlined in Problem 4.14. Similar conditions apply to the other solution.
The solutions are x = 0.6190 . . ., and x = 1.5121 . . ..
4.16. (a) Calculate f (a + nE) for n = 1, 2, . . .. Stop the program at n = N , when f (a + N E) and
f (a + (N − 1)E) have different signs.
(b) In the following table the interval is bisected four times with E = 0.125 and N = 8.

x 0 0.125 0.250 0.375 0.500 0.625


f (a + N E) -1 -0.983 -0.929 -0.829 -0.676 -0.457

x 0.750 0.875 1
f (a + N E) -0.162 0.224 0.718

The solution of the equation lies between x = 0.75 and x = 0.875. The computed solution is
x = 0.806 . . ..
(c) Four decimal accuracy is obtained after 10 iterations using the bisection method, whilst New-
ton’s method achieve the same accuracy after just 4 iterations
4.17. The slope of the normal at x = x0 is −1/f 0 (x0 ) and at x = x0 + δx0 is −1/f 0 (x0 + δx0 ).
Hence their equations are
1
y − f (x0 ) = − 0 (x − x0 ),
f (x0 )
1
y − f (x0 + δx0 ) = − (x − x0 − δx0 ).
f 0 (x0 + δx0 )
Solving these equations for x and y:

f 0 (x0 )[f 0 (x0 + δx0 ){f (x0 + δx0 ) − f (x0 )} + δx0 ]


x = x0 − ,
f 0 (x0 + δx0 ) − f 0 (x0 )

44
[f 0 (x0 + δx0 ){f (x0 + δx0 ) − f (x0 )} + δx0 ]
y = f (x0 ) + ,
f 0 (x0 + δx0 ) − f 0 (x0 )
Divide the numerators and denominators by δx0 and let the increment tend to zero, so that the
centre of curvature (xc , yc ) is located at
µ ¶
f 0 (x0 )[1 + f 0 (x0 )2 ] [1 + f 0 (x0 )2 ]
x0 − , f (x0 ) + .
f 00 (x0 ) f 00 (x0 )
The radius of curvature
3
√ 2 2 [1 + f 0 (x0 )2 ] 2
R= [(xc − x0 ) + (yc − y0 ) )] = .
f 00 (x0 )

For the parabola y = x2 ,

f (x) = x2 , f 0 (x) = 2x, f 00 (x) = 2.

Hence the centre of curvature of the point (x0 , x20 ) is located at


1
[x0 − x0 (1 + 4x20 ), x20 + (1 + 4x20 )],
2
3
and its radius of curvature is R = 21 (1 + 4x20 ) 2 .
4.18. We shall prove Leibniz’s formula by induction. For n = 1, the formula is true since

(f g)(1) = f (1) g + f g (1)

by the product rule: note that 1 C1 = 1. Assume that the given formula is true for n = k and all
x. Then
(f g)(k) = f (k) g + k C1 f (k−1) g (1) + k C2 f (k−2) g (2) + · · · + k Ck f g (k) .
Differentiate both sides with respect to x:

(f g)(k+1) =
(f (k+1) g + f (k) g (1) ) + (k C1 f (k) g (1) + k C1 f (k−1) g (2) )
+(k C2 f (k−1) g (2) + k C2 f (k−2) g (2) ) + · · · + (k Ck f (1) g (k) + k Ck f g (k+1) )
= f (k+1) g + (1 + k C1 )f (k) g (1) + (k C1 + k C2 )f (k−1) g (2 + · · · + k Ck f g (k+1) .

The coefficients can be written as


k!
1 + k C1 = 1 + = k + 1 = k+1 C1 ,
1!(k − 1)!
k! k! k(k − 1) k(k + 1)
k−1 C1 + k C2 = + =k+ = = k+1 C2 ,
1!(k − 1)! 2!(k + 2)! 2! 2!
and, in general,
k! k!
k Cr + k Cr+1 = +
r!(k − r)! (r + 1)!(k − r − 1)!
· ¸
k! 1 1
= +
r!(k − r − 1)! k − r r + 1
k!(k + 1) (k + 1)!
= =
r!(k − r − 1)!(k − r)(r + 1) (r + 1)!(k − r)!
= k+1 Cr+1

Hence

(f g)(k+1) = f (k+1) g + k+1 C1 f (k) g (1) + k+1 C2 f (k−1) g (2) + · · · + k+1 Ck+1 f g (k+1) .

45
Hence if the result is true for n = k, then it is true for n = k + 1. We have shown that it is true
for n = 1 (the product rule); therefore it is true for n = 2, 3, . . ..

Chapter 5: Taylor series and approximations


1
5.1. (a) For f (x) = e 2 x ,
1 1x 1 1x 1 1x
f 0 (x) = e2 , f 00 (x) = e2 , e2 ,
2 4 8
so that
1 1 1
f (0) = 1, f 0 (0) =
, f 00 (0) = , f 00 (0) = .
2 4 8
The Taylor polynomial approximation to four terms becomes
1 11 2 11 3
f (x) ≈ 1 + x + x + x .
2 2! 4 3! 8
We can estimate that the three term approximation will be accurate to two decimal places if for
the fourth term ¯ ¯
¯ 1 3¯
¯ ¯
¯ 8.3! x ¯ < 0.005, or |x| < 0.621 . . . .
1
(b) For f (x) = (1 + x) 2 , the Taylor approximation is
1 1 1 1
(1 + x) 2 ≈ 1 + x − x2 + x3 .
2 8 16
The three-term approximation will be accurate to two decimal places if
¯ 3¯
¯x ¯
¯ ¯ < 0.005 or |x| < 0.432 . . . .
¯ 16 ¯

1
(c) For f (x) = (1 + x)− 3 , the four-term Taylor polynomial is
1 ¡ ¢ ¡ ¢¡ ¢³ 2´ ¡ ¢¡ ¢¡ ¢³ 3´
(1 + x)− 3 ≈ 1 + − 13 x + − 13 − 43 x2! + − 13 − 34 − 73 x3!
1 2 14
≈ 1 − x + x2 − x3
3 9 81
The three-term approximation will be accurate to two decimal places if
14 3
|x| < 0.005 or |x| < 0.306 . . .
81
.
(d) The Taylor approximation to four terms for sin 2x can be obtained form the series for sin y
where y = 2x (use (5.4c)):
1 1 1
sin 2x ≈ (2x) − (2x)3 + (2x)5 − (2x)7
3! 5! 7!
4 3 4 5 8 7
≈ 2x − x + x − x
3 15 315
The three-term approximation will be accurate to two decimal places if
8 7
|x| < 0.005 or |x| < 0.196 . . . .
15

(e) Using the expansion for cos z, where z = 12 x (see (5.4d)):


1 1 1 4 1
cos x ≈ 1 − x2 + x + x6 .
2 8 384 46080

46
The three-term polynomial will be accurate to two decimal places if
1 6
46080 x < 0.005 or |x| < 2.475 . . . .

(f) The four-term expansion is (see(5.4e)):

ln(1 + x) ≈ x − 12 x2 + 13 x3 − 14 x4 .

The three-term polynomial will be accurate to two decimal places if


1 4
x < 0.005 or |x| < 0.376 . . . .
4
1
(g) Let f (x) = (1 + x2 ) 2 . Put u = x2 . Then, as in (b),
1 1 1 1 1
(1 + x2 ) 2 = (1 + u) 2 ≈ 1 + u − u2 + u3
2 8 16
1 1 1
≈ 1 + x2 − x4 + x6 .
2 8 16
The three-term polynomial will be accurate to two decimal places if
1 6
x < 0.005 or |x| < 0.656 . . . .
16

(h) The four-term Taylor polynomial for ln(1 + 3x) is (put u = 3x, etc. ),
1 1 1
ln(1 + 3x) ≈ (3x) − (3x)2 + (3x)3 − (3x)4
2 3 4
9 2 81
≈ 3x − x + 9x3 − x4 .
2 4
The three-term approximation will be accurate to two decimal places if
81 4
x < 0.005 or |x| < 0.125 . . . .
4

5.2. The Taylor expansion for f (x) about x = 0 is


1 00 1
f (x) = f (0) + f 0 (0) + f (0)x2 + f 000 (0)x3 + · · · .
2! 3!

(a) Let f (x) = ex . Then f 0 (x) = f 00 (x) = · · · = ex . Hence

f (0) = f 0 (0) = f 00 (0) = · · · = 1.

(b) Let f (x) = sin x. Then

f 0 (x) = cos x, f 00 (x) = − sin x, f 000 (x) = − cos x, f (4) (x) = sin x, etc.

so that
f (0) = 0, f 0 (0) = 1, f 00 (0) = 0, f 000 (0) = −1, f (4) (0) = 0, . . . ,
the Taylor coefficients being
1 1
1, , , ... .
1! 2!
(c) Let f (x) = cos x. Then

f 0 (x) = − sin x, f 00 (x) = − cos x, f 000 (x) = sin x, f (4) (x) = cos x, . . . ,

47
so that
f (0) = 1, f 0 (0) = 0 f 00 (0) = −1 f 000 (0) = 0, f (4) (0) = 1, . . . .

(d) Let f (x) = (1 + x)α . Then

f 0 (x) = α(1 + x)α−1 , f 00 (x) = α(α − 1)(1 − x)α−2 , . . . .

so that
f (0) = 1, f 0 (0) = α, f 00 (0) = α(α − 1), . . . .

(e) Let f (x) = ln(1 + x). Then


1 1
f 0 (x) = , f 00 (x) = − ,
1+x (1 + x)2
2×1 3×2×1
f 000 (x) = , f (4) (x) = , ....
(1 + x)3 (1 + x)4 )
so that
f (0) = 0, f 0 (0) = 1, f 00 (0) = −1, f 000 (0) = 2!, f (4) (0) = 3!, . . . .
Therefore the coefficient of xn for n ≥ 1 is

(n − 1)! (−1)n−1
(−1)n−1 = .
n! n

5.3. (a) The general term for ex is xn /n!. Hence, for four-decimal point accuracy we require n
such that, for |x| = 2,
xn 2n
= < 0.00005.
n! n!
For n = 11, 2n /n! = 0.000051 . . . and for n = 12, 2n /n! = 0.0000085 . . . < 0.00005. Hence terms
up to x11 are required.
(b) The general term for sin x is (−1)n x2n−1 /(2n − 1)!. Hence for four-decimal accuracy we require
the smallest n such that, for |x| = 2,
¯ 2n−1 ¯
¯ x ¯ 2n−1
¯ ¯= 2
¯ (2n − 1)! ¯ (2n − 1)! < 0.00005.

For n = 6, 22n−1 /(2n − 1)! = 0.000051 . . . and for n = 7, 22n−1 /(2n − 1)! = 0.0000031 . . .. Hence
terms up to and including x11 are required.
(c) The general term for cos x is (−1)n x2n /(2n)!. Hence for four decimal accuracy we require the
smallest n such that, for |x| = 2,
¯ 2n ¯
¯ x ¯ 2n
¯ ¯= 2
¯ (2n)! ¯ (2n)! < 0.00005.

For n = 5, 22n /(2n)! = 0.00028 . . . and for n = 6, 22n /(2n)! = 0.0000085 . . .. Hence terms up to
and including x10 are required.
1
(d) For (1 + x) 2 the general term is

α(α − 1)(α − 2) . . . (α − n + 1) n
x ,
n!
where α = − 12 . For four-decimal accuracy we require the smallest n such that, for |x| = 0.5,
¯ ¯
¯ α(α − 1)(α − 2) . . . (α − n + 1) ¯
¯ (0.5) n¯
¯ n! ¯ < 0.00005.

48
For n = 8 the last term has magnitude 0.000051 . . ., and for n = 9 the magnitude is 0.000021 . . . .
(e) For ln(1 + x), the general term in its Taylor series is (−1)n+1 xn /n. For four-decimal accuracy
we require the smallest n such that, for |x| = 0.5,
¯ ¯
¯ n¯ n
¯(−1)n+1 x ¯ = 0.5 < 0.00005.
¯ n¯ n
n
0.5n
For n = 10, 0.5
n = 0.000097 . . ., whilst for n = 11, n = 0.000044 . . .. Hence terms up to and
including x10 are required.
5.4. (a) Let f (x) = arcsin x. Then

1 x 1 + 2x2
f 0 (x) = √ , f 00 (x) = 3 , f 000 (x) = 5
(1 − x2 ) (1 − x2 ) 2 (1 − x2 ) 2

so that f (0) = 0, f 0 (0) = 1, f 00 (0) = 0, f 000 (0) = 1. Hence


1
arcsin x = x + x3 + · · · .
6

(b) Let f (x) = arccos x. Then


1 x
f 0 (x) = − √ , f 00 (x) = − 3
(1 − x2 ) (1 − x2 ) 2

so that f (0) = 12 π, f 0 (0) = −1, f 00 (0) = 0. The Taylor series starts with

1
arccos x = π − x + ···.
2

(c) Let f (x) = arctan x. Then

1 2x −2 + 6x2
f 0 (x) = , f 00 (x) = − , f 000 (x) = .
1 + x2 (1 + x2 )2 (1 + x2 )3

Hence f (0) = 0, f 0 (0) = 1, f 00 (0) = 0, f 000 (0) = −2. The Taylor series starts with
1
arctan x = x − x3 + · · · .
3

(d) Let f (x) = e−x sin x. Then

f 0 (x) = e−x (cos x − sin x), f 00 (x) = −2e−x cos x.

Hence f (0) = 0, f 0 (0) = 1, f 00 (0) = −2. The Taylor series starts with

e−x sin x = x − x2 + · · · .

(e) Let f (x) = e−x cos x. Then

f 0 (x) = −e−x (cos x + sin x).

Hence f (0) = 1, f 0 (0) = −1 so that the Taylor series starts with

e−x cos x = 1 − x.

5.5. (a) Let f (x) = 1/(1 + 3x). Then f 0 (x) = −3/(1 + 3x)2 , f 00 (x) = 18/(1 + 3x)3 so that

f (0) = 1, f 00 (0) = −3, f 00 (0) = 18.

49
The first three terms of its Taylor series are
1
= 1 − 3x + 9x2 + · · · .
1 + 3x
Alternatively, the binomial expansion (5.4f) can be used. Also from (5.4f) the expansion will be
valid for
1 1
−1 < 3x < 1 or − < x < .
3 3
(b) Adapting (5.4f),
h
³ x ´i−1 · ³ x ´ (−1)(−2) ³ x ´2 ¸
1 −1 1
= 2 1+ − = 1 + (−1) − + − + ···
2−x 2 2 2 2! 2
1 1 1
= + x + x2 + · · ·
2 4 8
the series is valid for −1 < 12 x < 1 or −2 < x < 2.
(c) Using (5.4f) again
1 1 1 £ ¤
(3 − x) 3 = 3 3 (1 − 13 x) = 3 3 1 − 91 x − 1 2
81 x − ··· .

The series is valid for −3 < x < 3.


(d) Using (5.4f) again

1 1
h ³ x ´i 13
(x − 3) 3 = (−3) 3 1 + −
· 3 ¸
1 1 1
= (3) 3 −1 + x + x2 + · · · .
9 81

The series is valid for −3 < x < 3.


(e) Adapting (5.4e),
· µ ¶¸ µ ¶
1 1 1 1 2
ln(9 − x) = ln 9 1 − x = ln 9 + ln 1 − x = 2 ln 3 − x − x − ···.
9 9 9 162

The series is valid for −9 < x < 9.


(f) From (5.4d),

1 1 ³ x ´2 1 ³ x ´4
cos( x) = 1− + − ···
2 2! 2 4! 2
1 1 4
= 1 − x2 + x − ···.
8 384
The series is valid for all x.
1
(g) Put u = x 2 for x > 0, and use (5.4c):
1 1 1 5 1 1 3 1 5
sin(x 2 ) = sin u = u − u3 + u − · · · = x2 − x2 + x2 − · · · .
6 120 6 120
1
Since x 2 is not real for x < 0 the series will be valid only for x ≥ 0.
1
(h) Put u = x 2 for x > 0, and use (5.4d):
1
cos(x 2 ) = 1 − 12 x + 1 2
24 x + ···.

The series is valid for all x ≥ 0.

50
5.6. Multiply standard expansions for
1 1
e−x = 1 − x + x2 + · · · and = (1 + x)−1 = 1 − x + x2 + · · · .
2 1+x
Hence
µ ¶
e−x 1 2 ¡ ¢
= 1 − x + x + · · · 1 − x + x2 + · · · .
1+x 2
5
= 1 − 2x + x2 + · · ·
2

(b) As in (a)
µ ¶µ ¶
1 1 1 1
(1 − x) 2 ex = 1 − x − x2 − · · · 1 + x + x2 + · · ·
2 8 2
1 1 2
= 1 + x − x + ···
2 8

(c) This time square the series for ln(1 − x):


· ¸2
1 1 1 2 1 3
[ln(1 − x)]2 = −x − x − x − · · ·
x2 x2 2 3
· ¸
1 2 3 11 4
= x + x + x + ···
x2 12
11
= 1 + x + x2 + · · ·
12

5.7. Start with the Taylor series


1
1 + ln(1 + x) = 1 + x − x2 + · · · .
2
Then, assuming that the expansion takes the form b0 + b1 x + b2 x2 + · · ·,
1 1
= = b0 + b1 x + b2 x2 + · · · .
[1 + ln(1 + x)] 1 + x − 21 x2 + · · ·

We now equate coefficients of powers of x in the identity


µ ¶
1 ¡ ¢
1 = 1 + x − x2 + · · · b0 + b1 x + b2 x2 + · · · ,
2
1
= b0 + (b1 + b0 )x + (b2 − b1 − b0 )x2 + · · ·
2
Hence b0 = 1, b1 = −b0 = −1, b2 = b1 + 12 b0 = 3
2 and the Taylor series starts with

1 3
= 1 − x + x2 + · · · .
[1 + ln(1 + x)] 2

(b) Write tan x = sin x/ cos x and use the series for sin x and cos x. Thus

x − 16 x3 + 120
1
x5 + · · ·
tan x = 1 1 3
1 − 2x + 6x + · · ·
= b1 x + b3 x3 + b5 x5 + · · · .

51
Note that the series will contain only odd powers of x. By cross-multiplying
µ ¶
1 3 1 5 1 1 3 ¡ ¢
x− x + x + · · · = 1 − x + x + · · · b1 x + b3 x3 + b5 x5 + · · · .
6 120 2 6

By matching the coefficients of x, x2 , . . . on either side we obtain


1 2
b1 = 1, b3 = , b5 = ,
3 15
so that
1 2
tan x = x + x3 + x5 + · · · .
3 15
(c) From (5.4b)
1 1
1 + ex = 2 + x + x2 + x3 · · · .
2 6
Assume that
1
= b0 + b1 x + b2 x2 + · · · :
1 + ex
then
µ ¶
1 2 1 3 ¡ ¢
1 = 2 + x + x + x · · · b0 + b1 x + b2 x2 + b3 x3 · · ·
2 6
µ ¶
1
= 2b0 + (b0 + 2b1 ) x + b0 + b1 + 2b2 + · · ·
2
Solving for b0 , b1 , b2 and b3 ,
1 1 1 1
x
= − x + x3 + · · · .
1+e 2 4 48
(d) Use the definition
sinh x ex − e−x
tanh x = = x ,
cosh x e + ex
where
1 1 5 1 1
sinh x = x + x3 + x + · · · , cosh x = 1 + x2 + x4 + · · · .
6 120 2 24
Hence, if the required series is b1 x + b3 x3 + b5 x5 + · · · (it must be an odd function), then
µ ¶
1 1 5 1 1 ¡ ¢
x + x3 + x + ··· = 1 + x2 + x4 + · · · b1 x + b3 x3 + b5 x5 + · · ·
6 120 2 24
1 1 1
= x + (b1 x + (b3 + b1 )x3 + (b5 + b3 + )x5 + · · ·
2 2 24
Comparing powers of x, it follows that b1 = 1, b3 = − 13 , b5 = 2
15 so that
1 2
tanh x = x − x3 + x5 + · · · .
3 15

(e) Since x/ sin x is an even function and sin x = x − 16 x3 + 120


1
x5 , then
µ ¶
1 1 5 ¡ ¢
x = x − x3 + x b0 + b2 x2 + b4 x4 + · · ·
6 120
µ ¶ µ ¶
1 1 1
= b0 x + b2 − b0 + b4 − b2 + b0 x5 + · · ·
6 6 120
1 7
Hence b0 = 1, b2 = 6 and b4 = 360 so that
x 1 7 4
= 1 + x2 + x + ···.
sin x 6 360

52
5.8. The following series provide approximations for large values of x.
(a) Let u = 1/x. Then
µ ¶1
1 2 1 1 1
1− = (1 − u) 2 = 1 − u − u2 + · · · (binomial series)
x 2 8
1 1
= 1− − + ···
2x 8x2
which will be valid for |u| < 1, or equivalently, |x| > 1.
1
(b) Let x > 0 and u = 1/x 2 > 0. Then
µ ¶
1 1 1
ln 1 + 1 = ln(1 + u) = u − u2 + u3 + · · ·
x2 2 3
1 1 1
= 1 − + + ···.
x2 x 3x 32
This series will be valid for 0 < u ≤ 1, or x ≥ 1.
(c) Let x > 0 and u = 1/x. Then
1
x2 1
1 = 1 = (1 + u)−1
(1 + x) 2 (1 + x1 ) 2
1 3 2
= 1− u+ u + · · · (binomial expansion)
2 8
1 3
= 1− + + ···
2x 8x2
The series is valid for 0 < u < 1 or x > 1.
(d) Let u = (1/x) + (1/x)2 . Then, using (5.4e),
1
ln(1 + x + x2 ) ln(x2 ) + ln(1 + u) = ln(x2 ) + u − u2 + · · ·
=
2
µ ¶ µ ¶2
1 1 1 1 1
= ln(x2 ) + + − + + ···
x x2 2 x x2
1 1
= ln(x2 ) + + 2 + · · ·
x 2x
√ √
The series is valid for −1 < (1/x)+(1/x2 ) < 1, that is, when x < − 12 ( 5−1) or when x > 21 (1+ 5).
(e) Let u = 1/x. Then
1 1 1
= = 1 3 1 5
sin(1/x) sin u u − 3! u + 5! u + ···
1 1
= · 1 2 1 4
u 1 − 3! u + 5! u ···
" µ ¶ µ ¶2 #
1 1 2 1 4 1 2 1 4
= 1+ u + u + u + u + ···
u 6 120 6 120
· ¸
1 1 7 4
= 1 + u2 + u + ···
u 6 360
1 7
= x+ + + ···
6x 360x3

5.9. (a) Using the two-term Taylor expansion for sin x,


1 1 1 1
≈ ≈
sin x x − 16 x3 x 1 − 61 x2

53
µ ¶
1 1 2
≈ 1+ x (using (5.4f))
x 6
1 1
≈ + x,
x 6
for small x. Note that for x = 0.5, the error is
¯ µ ¶¯
¯ ¯
¯sin x − 1 + 1 x ¯ = 0.0024 . . . ,
¯ x 6 ¯

that is, about 0.1%.


(b) Write as
µ ¶1 µ ¶
1 1 1 2 1 1
(1 + x) 2 = x2 1+ ≈ x2 1 +
x 2x
1 1
= x2 + 1 ,
2x 2
for large x.
(c) Using the result from (b)
1 1 1 1 1 1
(2 + x) 2 − (1 + x) 2 = 2 2 (1 + x) 2 − (1 + x) 2
·³ ´2 1 ¸ · ¸
1 x 2 1 1 1
≈ 22 + 1/(2x) 2 − x 2 + 1
2 2x 2
1
≈ 1 ,
2x 2
for large x.
(d) Using the three-term Taylor expansion for cos x,
· ¸− 1
1 x2 x4 2
1 ≈ −
(1 − cos x) 2 2 24
1 · ¸
22 1 2
≈ 1 + x (binomial expansion)
x 24
1 1
22 22 x
≈ + ,
x 24
for small x.
5.10. (a) Expanding about x = 1,
1 1
ln x = ln[1 + (x − 1)] = (x − 1) − (x − 1)2 + (x − 1)3 + · · · (using (5.4e)).
2 3
The series is valid for −1 < x − 1 < 1 or 0 < x < 2.
(b) For an expansion about x = 21 π, write

1 1 1
cos x = cos[ π + (x − π)] = − sin(x − π).
2 2 2
Now use the Taylor expansion for the sine:
1 1 1 1 1 1
cos x = − sin(x − π) = −(x − π) + (x − π)3 − (x − π)5 + · · ·
2 2 3! 2 5! 2
The series is valid for all x − 12 π, which means for all x.

54
(c) For a Taylor series centred at x = 1, write
1 1 1 1 1
(1 + x) 2 = [2 + (x − 1)] 2 = 2 2 [1 + (x − 1)] 2 .
2
Now expand the right-hand side using the binomial series (5.4f):
1 √ 1 1
(1 + x) 2 = 2 + √ (x − 1) − √ (x − 1)2 + · · · .
2 2 16 2

The series is valid for −1 < 12 (x − 1) < 1, that is for −1 < x < 3.
5.11. (a) Since f (x) has a stationary point at x = c, f 0 (c) = 0 and its Taylor series about x = c
will be
1 1
f (x) = f (c) + f 00 (c)x2 + f 000 (c)x3 + · · · .
2! 3!
Approximately
1
f (x) ≈ f (c) + f 00 (c)x2 ,
2
00
for |x − c| small. Hence if f (c) > 0, then f (x) > f (c) close to x = c excluding x = c. The
conclusion is that x = c is a minimum. Similarly if f 00 (c) < 0, then x = c is a maximum.
(b) If f 00 (c) = 0, then we must look at the signs of higher derivatives. Suppose that f (N ) (c) 6= 0 is
the first non-zero derivative (that is, f (r) (c) = 0 for r = 1, 2, . . . , N − 1). Hence, approximately,
1 N
f (x) ≈ f (c) + f (c)(x − c)N .
N!
If N is even and f (N ) (c) > 0 then x = c is a minimum, whilst if f (N ) (c) < 0 then x = c is a
maximum. If N is odd then the stationary point will be a point of inflection.
5.12. (Compare eqn (2.15).) Put (ex − 1)/x = f (x) for x 6= 0, and use the Taylor series (5.4b) for
ex :
1 1
f (x) = [(1 + x + x2 + · · ·) − 1]/x = 1 + x + · · · ,
2! 2!
for x 6= 0. Therefore limx→0 f (x) = 1, which is the ‘missing value’ at x = 0.
(b) Put (1 − cos x)/x2 = f (x) for x 6= 0. From (5.4d)
1 1 1
f (x) = [1 − (1 − + x4 − · · ·)]
x2 2! 4!
1 1 2 1
= ( x − x4 + · · ·)
x2 2! 4!
1 1
= − x2 + · · · (for x 6= 0)
2! 4!
Therefore limx→0 f (x) = 21 .
(c) Put [ln(1 + x) − x]/ sin x = f (x), x 6= 0. From (5.4c,e),

(x − 21 x2 + · · ·) − x − 12 x2 + · · ·
f (x) = 1 3 = 1 3
x − 3! x + ··· x − 3! x + ···
x(− 12 + · · ·
= 1 3 (for x 6= 0).
1 − 3! x + ···

Therefore limx→0 f (x) = 0.


(Alternatively, rewrite f (x) in the form

ln(1 + x) − x x
x sin x

55
and use the limits (2.13) and (2.14).)
(d) Put sin x/(1 − cos x) = f (x), x 6= 0:
1 1 2
x − 3! + ··· x(1 − 3! x + · · ·)
f (x) = 1 2 = 1 2 1 2
1 − (1 − 2! x + · · ·) 2 x (1 − 4! x + · · ·)
1 2(1 − · · ·)
= .
x (1 − · · ·)

This does not tend to a limit; it approaches ∞ as x → 0. Therefore this function does not possess
a fill-in value at x = 0 which would make it continuous.
5.13. (a)
(1 − x)12 − 1 −12x + higher powers 12
lim = lim = ,
x→0 (1 − x)10 − 1 x→0 −10x + higher powers 10
where the binomial theorem (4.7) was used to expand the powers of (1 − x).
(b)
1 3
sin x − x x − 3! x + · · ·) − x
lim = lim 1 3 1 2
x→0 sin x − x cos x (x − 3! x+ · · ·) − x(1 − 2! x + · · ·)
x→0
1
− 3! + higher powers
= lim 1
3 + higher powers
x→0
1
− 3! + higher powers
= lim
1
x→0
3 + higher powers
1
= − .
2

(c) Put x = π + u. Then

cos x + 1 cos(π + u) + 1
lim = lim
x→π x−π u→0 u
cos π cos u − sin π sin u + 1
= lim (from (1.17a))
u→0 u
1 2
− cos u + 1 −(1 − 2! u + · · ·) + 1
= lim = lim
u→0 u u→0 u
1
= lim u( + · · ·) = 0.
u→0 2

(d) Put x = u + 12 π: then

sin x − 1 sin( 12 π + u) − 1
lim1 = lim
x→ 2 π cos 5x u→0 cos( 5 π + 5u)
2
sin 12 π cos u + cos 12 π sin u − 1
= lim (from (1.17a))
u→0 cos 5 π cos 5u − sin 5 π sin 5u
2 2
1 2
cos u − 1 u − ···
= lim = lim 2
u→0 − sin 5u u→0 −5u + · · ·
= 0

5.14. Let
ex − 1 (ex − 1)/x
f (x) = = .
ex − 1 − x [({ex − 1)/x} − 1]
Since limx→0 [(ex − 1)/x] = 1 (see eqn (2.15) or Problem 5.12a), f (x) approaches infinity as x → 0

56
To determine the question of signs, return to the original form and take the following steps:
(i) ex − 1 is negative when x < 0 and positive when x > 0.
(ii) (d/dx)(ex − 1 − x) = ex , which is greater than zero for all x. Therefore ex − 1 − x is a steadily
increasing function for all x. Since also it is zero at x = 0, it must be negative when x < 0 and
positive for x > 0.
(iii) From (i) and (ii), f (x) is negative when x < 0 and positive when x > 0. Therefore f (x) → −∞
as x → 0 from the left, and f (x) → +∞ as x → 0 from the right.
5.15. (a)
· ¸
sin3 3x (3x + · · ·)3
lim = lim
x→0 1 − cos x x→0 1 − (1 − 1 x2 + · · ·)
2!
27x3 + · · ·
= lim
x→0 1 x2 + · · ·
2
27x(1 + · · ·)
= lim = 0.
x→0 1 (1 + · · ·)
2
1
(b) Let [(ex − 1)/x] 2 = g(x). We know from eqn (2.15) that
· x ¸
2 e −1
lim [g(x)] = lim = 1.
x→0 x→0 x
But also
lim [g(x)]2 = lim [g(x)g(x)] = lim g(x) lim g(x) = [ lim g(x)]2 .
x→0 x→0 x→0 x→0 x→0

Therefore √
lim g(x) = 1 = 1,
x→0

the positive square root being taken because g(x) is never negative.
(c)
· ¸ · ¸ · ¸
(2 + tan x) sin x 2 + tan x sin x
lim = lim lim
x→0 x(3 − tan2 x) x→0 3 − tan2 x x→0 x
2+0 2
= · 1 = (where we refer to eqn (2.13))
3−0 3

5.16. Let f (x) = 3x − sin x and g(x) = x. Then


¯
f (x) f 0 (x) 3 − cos x ¯¯
lim = lim 0 = ¯ = 2.
x→0 g(x) x→0 g (x) 1 x=0

5.17. In the following, S represents the required sum.


(a)
X∞ ∞
1 n X
S = x − (−x)n
n=0
n! n=0
1
= ex − (1 − x + x2 − · · ·) = ex − ,
1+x
the second term being a geometric series with common ratio (−x): see (5.4a).
(b) S = x3 + 21 x4 + 13 x5 + · · · = x2 (x + 12 x2 + 13 x3 + · · ·). From (5.4e),
1 1
ln(1 − x) = −x − x2 − x3 − · · · .
2 3
Therefore S = −x2 ln(1 − x).

57
1 2 1 4
(c) S = 1 + 2! x + 4! x + · · ·. But from (5.4b),

1 2 1 1 1
ex = 1 + x + x + x3 + · · · and e−x = 1 − x + x2 − x3 + · · · .
2! 3! 2! 3!
Therefore ex + e−x = 2S, and
1 x
S= (e + e−x ) = cosh x
2
from (1.26).
5.18. In the following, S represents the required sum.
(a) From (5.4b)
2 3
e2 = 1 + 2 + 22! + 23! + · · ·,
and
22 23
e−2 = 1 − 2 + 2! − 3! + · · ·,
so that ³ ´
23 25
e2 − e−2 = 2 2 + 3! + 5! + ··· ,
or
S = 12 (e2 − e−2 ).

1
(b) From (5.4b), S = e 2 .
(c) S is geometric with common ratio (− 14 ). Therefore, by (5.4a),

1 4
S= 1 = .
1+ 4
5

Chapter 6: Complex numbers

6.1. (a) x = −1 ± i; (b) x = 3 ± i; (c) x = i or −3i.


6.2. The equation is a quadratic equation in x2 . Hence x2 = −4 or 1. Taking square roots

x = ±1 or ± 2i.

6.3. The standard form of a complex number is a + ib, where a and b are real numbers. Thus the
answers are (a) 4 + 3i; (b) 3 − 5i; (c) −11 + 15i; (d) 9 + 3i; (e) 21 + 12 i; (f) 1 + 6i; (g) −3 − 4i: (h)
− 78 96
25 − 25 i; (i) −4 − 4gm.

6.4. The boundary between real and complex roots in the (p, q) plane is the parabola p2 = 4q:
the roots are real if p2 ≥ 4q and complex if p2 < 4q. The roots are both real and negative in the
quadrant p > 0, q < 0.
1
6.5. (a) 4 + i; (b) 5 + 5i; (c) 5 − 75 i; (d) − 13
25 −
9
25 i.

6.6. (a) −4i; (b) −7 + 4i; (c) − 15 + 85 i; (d) − 15 − 85 i.


6.7. (a) 1 − i; (b) 2i; (c) −2i; (d) 12 (1 + gm); (e) i.
6.8. Numerically to 3 decimal places the answers are: (a) 4.482 + 2.218i; (b) 16.233 − 0.167i; (c)
−1.248 + 2.728i; (d) 88.669; (e) 266.050 + 0.512i.

6.9. (a) |z1 | = 2 2, Arg z1 = 34 π; (b) |z2 | = 8, Arg z2 = − 13 π; (c) |z3 | = 5, Arg z3 = − 21 π; (d)
|z4 | = 3, Arg z4 = π; (e) |z5 | = 5, Arg z5 = arctan( 34 ).
6.10. The curves are: (a) the circle x2 + y 2 = 1; (b) the straight line y = 2; (c) The circle
(x − a1 )2 + (y − a2 )2 = 1 where a1 = Re(a) and a2 = Im(a); (d) the parabola y 2 = 4x; (e) the
ellipse 3x2 + 4y 2 = 12 (need to square twice to remove the square roots); the complex formula

58
expresses the well-known property of ellipses that the sum of the distances from any point on an
ellipse to the foci is a constant; (f) the straight line y = x for x ≥ 0; (g) the archimedean spiral
r = θ.

6.11. (a) 2 exp( 34 iπ); √ (b) 2 exp(iπ); (c) 3 exp(− 21 iπ);√ √
1
(d) 14√
exp(− 3 iπ); (e) 2 2 exp(iθ) where θ = arctan[( 3 − 1)/( 3 + 1)];
2
(f) 1+√ exp(− 41 iπ); (g) e2 exp(i);
√ 3
(h) √ 2 exp(iθ) where θ = arctan[(cos 2 + sin 2)/(cos 2 − sin 2)]; (i) 512 exp(iπ);
(j) 2 exp( 43 iπ).
6.12. Use the identity
ei(θ1 +θ2 ) = eiθ1 eiθ2 .
Hence

cos(θ1 + θ2 ) + i sin(θ1 + θ2 )
= (cos θ1 + i sin θ1 )(cos θ2 + i sin θ2 )
= cos θ1 cos θ2 − sin θ1 sin θ2 + i(sin θ1 cos θ2 + cos θ1 sin θ2 ).

Equating real and imaginary parts it follows that

cos(θ1 + θ2 ) = cos θ1 cos θ2 − sin θ1 sin θ2 ,

and
sin(θ1 + θ2 ) = sin θ1 cos θ2 + cos θ1 sin θ2 .
For the other identities use
ei(θ1 −θ2 ) = eiθ1 e−iθ2 .

6.13.
-
z1 -z2 z1 +z2
-
-z2
z2

-
z1 z1 +z2

-
z1 -z2 z1

-
-z2 z2

- -
z1 +z2

Figure 58: Problem: 6.13

6.14 For the general case with f (θ) = a cos θ + b sin θ,

f 0 (θ) = −a sin θ + b cos θ,

and
f 00 (θ) = −a cos θ − b sin θ = −f (θ).
The first case can be obtained by putting a = 1 and b = i.
6.15. Using exponential forms for cos and sin, it follows that
sin ia 2[exp(ai2 ) − exp(−ai2 )]
tan a = =
cos ia 2i[exp(ai2 ) + exp(−ai2 )]
1 exp(−a) − exp(a) exp(a) − exp(−a)
= · =i
i exp(−a) + exp(a) exp(a) + exp(−a)
= i tanh a.

59
6.16. (a) The equation cosh z = 1 implies
1 z
(e + e−z ) = 1 ⇒ e2z − 2ez + 1 = 0 ⇒ (ez − 1)2 = 0.
2
Hence ez = 1. If z = a + bi (a, b real), then ea eib = 1 = e2nπi , (n = 0, ±1, ±2, . . .). Thus a = 0 and
b = 2nπ. The roots are given by z = 2nπi, (n = 0, ±1, ±2, . . .).
(b) Similarly sinh z = 1 implies
e2z − 2ez − 1 = 0.

Hence ez = 1 ± 2. If z = a + bi, then

ea+bi = ea (cos b + i sin b) = 1 ± 2.

It follows that sin b = 0 so that b = nπ, n = 0, ±1, ±2, . . .). Hence



ea cos b = ea cos(nπ) = (−1)n ea = 1 ± 2,

so that √ √
a = ln[ 2 − 1], (n odd) a = ln[ 2 + 1] (n even) .
The complex roots are
√ √
z = ln[ 2 − 1] + inπ, (n odd) z = ln[ 2 + 1] + inπ (n even).

(c) ez = −1 = e2n+1 πi , (n = . . . − 2, −1, 0, 1, 2, . . .). It follows that the roots are

z = (2n + 1)πi (n = . . . − 2, −1, 0, 1, 2, . . .).



(d) cos z = 2 implies that
1 iz √ √
(e + e−iz ) = 2 ⇒ e2iz − 2 2eiz + 1 = 0.
2
Hence √ √
eiz = 2 ± 1 = ( 2 ± 1)e2nπi , (n = 0, ±1, ±2, . . .).
If z = a + ib, then √
e−b = ( 2 ± 1), and a = 2nπ.
Hence the roots are √
z = 2nπ − i ln[ 2 ± 1].
√ √ √
6.17. (a) Log(1 + i 3) = ln( (1 + 3)) + iArg (1 + i 3) = ln 2 + 13 iπ.
(b) We can write log z = Log|z| + i(Arg z + 2kπ) where k is an integer. Hence if log z = πi, then
|z| = 1 and Argz + 2kπ = π so that k = 0 and z = πi is the only solution.
(c) Log(ei) = ln(e) + 12 πi = 1 + 12 πi.
(d) elog z = eln r+iθ+2kπi = eln r eiθ = reiθ = z. Therefore log z defines the set of functions inverse to
ez , as is suggested by the notation.
6.18. (a) 2i = ei ln 2 = cos(ln 2) + i sin(ln 2).
(b)
1 1
ii = ei ln i = exp[i ln(e 2 πi )] = e− 2 π .
This number is real: hence Arg(ii ) = 0.
(c) The equation becomes

z i = ei log z ei[Log|z|+i(Arg z+2kπ0] = e−(Arg z+2kπ) eLog|z| = −1 = e(2n+1)πi ,

where n and k are integers. Hence

Log |z| = (2n + 1)π, Arg z = 0, k = 0.

60
Therefore, the solutions are given by z = e(2n+1)π , where n is any integer.
6.19. Write the equation as

z 5 = −1 = e(2n+1)πi , n, any integer.

The solutions are given by


· ¸
1
z = exp (2n + 1)πi , (n = 1, 2, 3, 4, 5).
5

Other values of n merely repeat these solutions. On the Argand diagram the solutions all lie on
the unit circle centred at the origin at polar angles 15 π, 35 π, π, 75 π, 95 π.
y
1

0.5

x
-1 -0.5 0.5 1

-0.5

-1

Figure 59: Problem: 6.19

6.20. Denote the complex number by z in each case.


(a) z = 2e3+2i = 2e3 [cos 2 + i sin 2]. Hence

|z| = 2e3 , Arg z = 2, Re z = 2e3 cos 2, Im z = 2e3 sin 2.

(b) z = 4ei = 4[cos 1 + i sin 1]. Hence

|z| = 4, Arg z = 1, Re z = 4 cos 1, Im z = 4 sin 1.


√ √ √
(c) z = 5 exp[cos( 14 π) + i sin( 14 π)] = 5 exp(1/ 2)[cos(1/ 2) + i sin(1/ 2)]. Hence
√ √
|z| = 5 exp(1/ 2), Arg z = 1/ 2,
√ √ √ √
Re z = 5 exp(1/ 2) cos(1/ 2, Im z = 5 exp(1/ 2 sin(1/ 2).
(d) z = e1+i = e(cos 1 + i sin 1). Hence

|z| = e, Arg z = 1, Re z = e cos 1, Im z = e sin 1.

6.21. Let z = ceα+iβ = ceα [cos β + i sin β]. Comparing with

x = 0.04e−0.01t sin 12t,

we can identify
1
c = 0.04, α = −0.01t, β = 12t + π.
2
6.22. We have to express the sine as a cosine. Hence
1
i(t) = ce−0.05t sin(0.4t + 0.5) = ce−0.05t cos(0.4t + 0.5 − π),
2
1
= Re[ce−0.05t ei(0.4t+0.5− 2 π) ],
1
= Re[ce−0.05t+i(0.4t+0.5− 2 π) ]

61
6.23. (a) z 2 = (x + iy)2 = (x2 − y 2 ) + i2xy. Hence
Re(z 2 ) = x2 − y 2 , Im(z 2 ) = 2xy.
(b) First
z 3 = (x + iy)(x2 − y 2 + i2xy) = (x3 − 3xy 2 ) + i(3x2 y − y 3 ).
Thus
z + 2z 2 + 3z 3 = (x + 2x2 − 2y 2 + 3x3 − 9xy 2 ) + i(y + 4xy + 9x2 y − 3y 3 ).
Hence
Re(z + 2z 2 + 3z 3 ) = x + 2x2 − 2y 2 + 3x3 − 9xy 2 ,
Im(z + 2z 2 + 3z 3 ) = y + 4xy + 9x2 y − 3y 3 .
(c)
1 i(x+iy)
sin z = sin(x + iy) = [e − e−i(x+iy) ]
2i
1 −y ix
= [e e − ey e−ix ]
2i
1 −y
= [e (cos x + i sin x) − ey (cos x − i sin x)]
2i
= e−y sin x.
Hence
Re(sin z) = e−y sin x, Im(sin z) = 0.
(d)
1 i(x+iy) 1
cos z = [e + e−i(x+iy) ] = [eix e−y + e−ix ey ]
2 2
= cos x cosh y − i sin x sinh y.
Hence
Re(cos z) = cos x cosh y, Im(cos z) = − sin x sinh y.
(e) Using (d)
1 x+iy
ez cos z = e [cos x cosh y − i sin x sinh y]
2
1 x
= e (cos y + i sin y)[cos x cosh y − i sin x sinh y]
2
1 x
= e [(cos y cos x cosh y + sin y sin x sinh y) +
2
(sin y cos x cosh y − cos y sin x sinh y)].
(f) exp(z 2 ) = exp(x2 − y 2 ) exp(2xyi) = exp(x2 − y 2 )[cos 2xy + i sin 2xy]. Hence
Re[exp(z 2 )] = exp(x2 − y 2 cos 2xy, Im[exp(z 2 )] = exp(x2 − y 2 sin 2xy.

6.24. w = u + iv = f (z) = z 2 = (x + iy)2 = x2 − y 2 + 2xyi. Hence, equating real and imaginary


parts
u = x2 − y 2 , v = 2xy.
The hyperbolas map into the straight lines u = 1 and v = 2 respectively in the w plane.
6.25. Substituting for z it follows that
c c(x − iy)
w = z + = x + iy + 2
z x + y2
µ ¶ µ ¶
cx cy
= x+ 2 +i y− 2 .
x + y2 x + y2

62
For the circle |z| = 1, x2 + y 2 = 1, so that
w = x(1 + c) + iy((1 − c).
Hence u = x(1 + c) and v = y(1 − c). Thus, on the circle |z| = 1
u2 v2
x2 + y 2 = 1 = + ,
(1 + c)2 (1 − c)2
which is the equation of an ellipse.
6.26. The derivation of the formula for cos6 θ is given in Example 6.20. For sin6 θ use the identity
µ ¶
1 1
sin nθ = zn − n ,
2i z
where z = cos θ + i sin θ. Then
µ ¶6
6 1
(2 sin θ) = − z−
z
µ ¶ µ ¶ µ ¶
6 1 4 1 2 1
= − z + 6 + 6 z + 4 − 15 z + 2 + 20
z z z
= 2 (− cos 6θ + 6 cos 4θ − 15 cos 2θ + 20) .
Finally
sin6 θ = 1
32 (− cos 6θ + 6 cos 4θ − 15 cos 2θ + 20).

6.27. The displacement is given by


x = Re z = e−0.2t cos 0.5t.
Hence x = 0 where cos 0.5t = 0. The required zeros of x are given by
1
0.5t = (2n + 1)π, for integer n .
2
Hence t = (2n + 1)π, (n = 0, ±1, ±2, . . .).
The velocity is given by
dx d £ −0.2t ¤
= e cos 0.5t = −e−0.2t [0.2 cos 0.5t + 0.5 sin 0.5t],
dt dx
or, alternatively, by
· ¸
dz d (−0.2+0.5i)t
Re = Re e
dt dt
£ ¤
= Re (−0.2 + 0.5i)e−0.2t (cos 0.5t + i sin 0.5t)
= −e−0.2t [0.2 cos 0.5t + 0.5 sin 0.5t].

6.28. If z = 2 + i is a solution then so is its conjugate 2 − i since the coefficients of the polynomial
are real. Therefore (z − 2 − i)(z − 2 + i) = z 2 − 4z + 5 is a factor. Hence
z 4 − 2z 3 − z 2 + 2z + 10 = (z 2 − 4z + 5)(z 2 + 2z + 2).
The other solutions are given by z 2 + 2z + 2 = 0, that is, z = −1 ± i.
6.29. (a)
1 1
S = 1 − sin θ + sin 2θ − sin θ + · · ·
"∞ 2! # 3! " ∞ #
X (−1)n eniθ X zn
= Im = Im ,
n=0
n! n=0
n!

63
where z = −eiθ . The infinite series is the Taylor series for the exponential function ez (see Section
5.4). Hence

S = 1 + Im[ez ] = 1 + Im[exp(−eiθ )]
= 1 + Im[exp(− cos θ − i sin θ)]
= 1 − e− cos θ sin(sin θ).

(b) In this case

22 23
T = 1 + 2 cos θ + cos 2θ + cos 3θ + · · ·
2! " 3! #
X∞ X∞
2n 2n niθ
= cos nθ = Re e
n=0
n! n=0
n!

Using the Taylor series for an exponential function;


X∞ X∞
2n niθ (2eiθ )n
e = = exp[2eiθ ] = exp[2 cos θ + 2i sin θ]
n=0
n! n=0
n!
= e2 cos θ [cos(2 sin θ) + i sin(2 sin θ)]

Hence
T = e2 cos θ cos(2 sin θ).

64

You might also like